español ap - East Aurora School District 131

Transcripción

español ap - East Aurora School District 131
Advanced
Placement
Spanish
Language
EAHS
2010 – 2011
Mr. Pedroni
Cuaderno de trabajo
1
NOMBRE:
_____________________________________
1. Consonants / Las consonantes
Spanish spelling is pretty consistent: most letters represent a single sound regardless of
their position in a word. Note the following peculiarities:
H - La hache is never pronounced. Thus, words like Honduras, ahora and alcohol have
no aspiration before the /o/ sound.
CH - la che is always pronounced as in "cheers": coche, ocho. La hache is not combined
with any other consonants: no th, sh, ph, gh, etc. (English "ph" may translate to "f":
filosofía, Filadelfia).
C - la ce is pronounced /k/ (as in "case") in most positions: caso, cosa, cuota, frecuente,
crisis.
Before -e, -i, it is pronounced /s/ (as in "sin") in America or /th/ (as in "thin") in Spain:
cielo, acento.
The /k/ sound (as in "kiss"), is spelled "qu" (mute "u") before -e,-i: queso, quince.
G - la ge is pronounced /g/ (as in "go") in most positions: gala, gota, guante, globo.
Before -e, -i, it is pronounced almost like /h/ (as in "hen"): general, gitano.
The /g/ sound (as in "get"), is spelled "gu" (mute "u") before -e,-i: guerra, guitarra.
If the letter "u" is to be pronounced in a "gue/gui" combination, it is marked with a
diaeresis (la diéresis): pingüino, bilingüe, nicaragüense.
Q - la cu is used only in the que/qui combinations, and the "u" is always mute in this
position. Therefore, English "quota" and "frequent" translate to cuota and frecuente, and
the word quinteto has no /u/ sound.
Z - la zeta is pronounced /s/ in America and /th/ in Spain. Spanish avoids the ze/zi
combination and prefers ce, ci: lápiz --> lápices; cebra, cenit.
Only four consonants can be duplicated to represent specific sounds:
• cc is used before "e" or "i" only and sounds /ks/ (/kth/ in Spain): acción and acceso but
acento, ocurrir.
• ll sounds /y/: calle, llama but ilegal, aludir.
• rr represents the famous "rolling r" between vowels only: perro, carro, vs. pero, caro.
• nn is used only when a prefix ending in "n" is added to a word beginning with "n":
innecesario, connotación.
No other consonants are duplicated in Spanish: diferente, común, oportunidad,
imposible.
2
Summary / Resumen
Hard C sound: /k/ as in kiss
ca que qui co cu
buscar, busqué
S/Z sound:/s/ or /th/ as in
sink/think
za ce ci zo zu
realizar, realicé
Hard G sound: /g/ as in get
ga gue gui go gu
pagar, pagué
Soft G sound: /h/ as in hen
ja je ji jo ju ge gi
hija, gitano
As you see, vowels e and i are exceptional in their combination with g and c.
The words que, quien, guerra and guitarra may help you remember these
spelling changes.
No double consonants except rr, ll, cc and nn
ph --> f : filosofía
PRÁCTICA 1. Escriba en español el equivalente de las siguientes palabras (consulte el
diccionario si es preciso), notando la ortografía (spelling):
dictionary
possible
photograph
necessary
connection
different
recommend
committee
difficult
chemistry
intellectual
quality
accuse
immigrant
occur
progress
bilingual
frequency
zeal
architect
2. Vowels / Las vocales
Spanish has five vowel sounds -a, e, i, o, u-, each of which is always pronounced the
same way regardless of their position in a word:
a. like the vowel sound in "hot" or "what": casa, alma
e. like the vowel sound in "net": Pepe, cena
i. like the vowel sound in "sheep": mil, millaje
o. like the vowel sound in "north" or "small": son, hoja
u. like the vowel sound in "roof": tú, Honduras
• i and u are called "weak" (débiles o cerradas) because, in combination with another
vowel, are generally pronounced as one syllable: Ruiz, fue, dio, have only one syllable.
These are considered true diphthongs in Spanish —two vowels in one syllable—, as in
a·gua or vien·to.
• a, o, and e are called "strong" (fuertes o abiertas). Two strong vowels are pronounced
3
as two syllables: po·e·ma has three syllables, ca·os has two. These are not true
diphthongs, since the vowels belong in different syllables.
PRÁCTICA 2. Pronuncie las siguientes palabras y subraye los diptongos:
mar, sea, leer, hacer, ayer, amiga, mente, china, siempre, actividad, caos, cooperar,
alcohol, ahora, serie, sociedad, hondureño, muerto, azul, sueños, agua, viento, actuar,
plus, feo, diez, fuerza, cierto, realizar, cuento, fui, previo, chocolate, bilingüe, cualidad,
ciudad, guerra.
3. Stress and written accent / Los acentos
Every word has a stressed syllable (sílaba tónica). For example, the stressed syllable in
the word español is the last one. The absence or presence of written accents (tildes)
provide a predictable guide to locate the stressed syllable in Spanish, allowing the reader
to pronounce any written word correctly, even if you have never seen it before.
A. Basic rules for the usage of accents / Reglas básicas de acentuación
S
E
G
A
S 4 -- SOBRE-ESDRUJULAS always had written accent on the four to the last syllable.
E 3 -- ESDRUJULAS always had written accent on the third to the last syllable.
G 2 -- GRAVES had a written accent when the ending in other that N,S, or ,A,E,I,O,U
A 1 -- AGUDAS had a written accent when the ending in N,S, or ,A,E,I,O,U
PRÁCTICA 3A. Pronuncie las siguientes palabras y subraye la sílaba tónica (stressed
syllable).
e li mi nar (cuatro sílabas) co ci na (tres sílabas)
gra cias (dos sílabas)
va rios (dos sílabas)
fies ta (dos sílabas)
al co hol (tres sílabas)
pa cien cia (tres sílabas) ai re (dos sílabas)
ciu dad (dos sílabas)
tie rra (dos sílabas)
cui da do (tres sílabas)
fre cuen te (tres sílabas)
cau sa (dos sílabas)
a ho ra (tres sílabas)
llue ve (dos sílabas)
eu ro pe os (cuatro sílabas)
ra cial (dos sílabas)
suer te (dos sílabas)
cue llo (dos sílabas)
so cie dad (tres sílabas)
cuo ta (dos sílabas)
cre e mos (tres sílabas)
cre a mos (tres sílabas)
i de a (tres sílabas)
Therefore, you need two criteria to decide whether a word needs an accent or not: a) its
ending, and b) what syllable is stressed. The word región needs an accent because it ends
4
with "n" but the penultimate syllable is not stressed. The word regiones does not need an
accent because it ends with "s" and the penultimate syllable is stressed.
PRÁCTICA 3B. La sílaba tónica está subrayada (underlined). Escriba la tilde si es
necesaria:
poema
poetico
fuerte
cancion
canciones
origen
origenes
util
sutil
natural
español
ingles
feliz
estan
Panama
hasta
panameño verdad
B. Other uses of the written accent / Otros usos de la tilde
1. The written accent also indicates stress on a weak vowel (i, u) that is followed or
preceded by a strong vowel (a, e, o): dí·a, Ma·rí·a, Ra·úl, o·í·do, a·hí, ra·íz, mí·o,
dú·o, ge·o·gra·fí·a
2. A written accent is not used for one-syllable words (fe, ti, tres, guion), except to
distinguish between certain words that are otherwise spelled identically, such as:
si quiero, puedo (if I want, I can)
vs. claro que sí (of course yes);
el problema (the problem)
vs. él intenta (he tries);
tu país (your country)
vs. tú creas (you create);
mi lectura (my reading)
vs. detrás de mí (behind me);
te buscaba (I was looking for you) vs. les gusta el té (they like tea);
se ven (they see themselves) vs. lo sé (I know that).
Another common group of such pairs are question words, which do not have written
accents when used as relatives:
¿cómo? how
como as
¿dónde? where?
donde where (as relative)
¿qué? what?
que that (as a relative pronoun)
PRÁCTICA 3C. Escriba A, B o C junto a las siguientes palabras, según la razón por la
que necesitan tilde (Write A, B, or C next to the following word, according to the reason
why they need a written accent).
A: termina en consonante -no ene ni ese- y la sílaba tónica no es la última.
B: termina en vocal, ene o ese, y la sílaba tónica no es la penúltima.
C: hay una vocal cerrada tónica junto a una vocal abierta.
así: ___ también: ___ pájaro: ___ economía: ___ aún: ___ cárcel: ___ día: ___
carácter: ____ nación: ____ país: ____ raíz: ____ túnel: ____ después: ____
PRÁCTICA 3D. La sílaba tónica está subrayada. Pronuncie las palabras y escriba la
tilde si es necesaria:
rio
verde
facil
transicion
5
alcohol
America
rio
sabana
mapa
transiciones
articulo
americano
pues
sabana
poema
caracter
veintidos
historia
Dios
Canada
democracia
caracteres
sociologia
emocion
enfasis
ingles
aleman
español
teoria
geografia
fe
ingleses
alemanes
españoles
teorias
imagen
radio
ultima
maximo
minimo
linea
imagenes
final
acento
cafe
arbol
arboles
poetico
papa
papa
pase
pase
tomo
tomo
4. Use of capital letters / El uso de las mayúsculas
1. Not used for days, months or religions: lunes, martes, enero, febrero, el budismo, un
católico.
2. Used for geographical names such as rivers, countries and cities (Bolivia, Los
Angeles), but not used for the adjectives: venezolano, norteamericano, panameña,
guatemalteco, caribeño.
3. In titles (of books, stories, movies, etc.): Used only for the first word and for any
proper nouns: La cabaña del tío Tom, Antología del cuento hispánico, Cien años de
soledad.
PRÁCTICA 4A. Escriba tres oraciones utilizando palabras que sí necesitan mayúsculas
en inglés y no en español.
PRÁCTICA 4B. Rescriba el siguiente texto, incluyendo las mayúsculas y los acentos
cuando sean necesarios:
"el sr. juan valdez esta claramente asociado
con el cafe colombiano a causa de la
publicidad que la federacion nacional de
cafeteros de colombia ha utilizado durante
años. recuerdo que, en mi niñez, lo veia
todos los miercoles durante mi programa
favorito, "plaza sesamo", y me preguntaba
como podia el mantener el delantal siempre
tan limpio".
"Mr. Juan Valdez is clearly associated with
Colombian coffee because of the publicity
that the National Federation of Coffee
Growers in Colombia has used for years. I
remember that, in my childhood, I used to
see him every Wednesday during my favorite
program, "Sesame Street," and I wondered
how he was able to keep his apron always so
clean."
5. Subject Pronouns /Los pronombres de sujeto
6
The verbs hablar (speak), comer (eat) and vivir (live) are "regular" or predictable in their
endings. Here are their forms in the present indicative with the corresponding subject
pronouns:
I
you (familiar, singular)
you (formal, singular)
he
she
we
you (fam. in Spain, pl.)
you (formal in Spain, pl.)
they
yo
tú
usted (Ud.)
él
ella
nosotros/as
vosotros/as
ustedes
ellos
ellas
hablo
hablas
como
comes
vivo
vives
habla
come
vive
hablamos
habláis
comemos
coméis
vivimos
vivís
hablan
comen
viven
Subject pronoun usage
1. Ya que las terminaciones verbales
1. Since the verb endings contain clues as to
indican de quién se habla, los pronombres
the speaker, subject pronouns are only used
de sujeto se usan solamente para mayor
when required for clarity or emphasis
claridad o énfasis (aunque "usted" se usa
(although usted is often used as a courtesy).
con frecuencia por cortesía).
2. When addressing friends, children, animals, and God, tú is used. The form usted
(commonly abbreviated to Ud.), which uses third-person verb forms, is used for polite
address to people you do not know well or wish to express respect for. Ustedes is used as
plural for both forms in America, while in Spain the plural of tú is vosotros/vosotras.
Note that Spanish does not capitalize yo but uses a capital for the abbreviation Ud.
PRÁCTICA 5A. Escriba la respuesta a la siguiente pregunta, y dé dos ejemplos en
ambos idiomas: "¿Por qué es que en español se usan los pronombres de sujeto con
menos frecuencia que en inglés?"
PRÁCTICA 5B. Escriba el presente de "yo" y de "Ud." para los siguientes verbos:
ayudar, asistir, distinguir, convencer, escoger, pagar.
7
6. Gender of Nouns / El género de los sustantivos
A. GENERAL GUIDELINES
1 Nouns referring to males and/or ending in
-o are masculine
2
Referring to females and/or ending in
-a -ción -sión -tad -dad -tud are feminine
3 Referring neither to males or females and
having different endings can be of either
gender
4 Many nouns change gender by changing
the last vowel or by adding -a to the last
consonant
el hombre, el amante, el teléfono
la mujer, la amante, la risa, la nación,
la tensión, la libertad, la ciudad, la
actitud
la clase, la parte, la paz, el lápiz, el reloj,
el golpe, el café, el cristal, el avión
el compañero, la compañera,
un profesor, la profesora
5 Many nouns have the same form and differ
only by the article
6 mano, foto(grafía) and moto(cicleta) are
FEMININE
7 día, mapa, sofá and planeta are
MASCULINE
el / la estudiante, el / la indígena,
el / la artista, deportista, turista, etc.
la mano, la foto, la moto
el día, el mapa, el sofá, el planeta
8 Many nouns ending in -ma (most of Greek
el programa, el problema, el sistema,
origin) are MASCULINE
el idioma, el clima, el poema, el tema, el
( BUT: la cama, la llama, etc)
dilema
9 The names of days, colors, languages and
cardinal points are MASCULINE:
el lunes, el rojo, el francés, el norte
10 Compound nouns, formed by combining a verb and a noun such as el salvavidas (life
saver), un paracaídas (parachute), los limpiaparabrisas (windshield wiper), el
abrelatas (can opener), etc. are also masculine.
B. CERTAIN GENDER PECULARITIES
B1. Feminine nouns that begin with stressed a or ha use a masculine article in the
singular only, but remain feminine in every respect.
but las aguas and el agua limpia
El agua
but las almas
El alma
but el hambre es eterna
El hambre
el águila, el área, el arma, el hacha, etc.
8
B2. The meaning of some nouns changes according to their gender:
el capital (money) vs. la capital (city)
el cuento (short story) vs. la cuenta (check, bill)
el mañana (future) vs. la mañana (morning)
el orden (order, not disorder) vs. la orden (command)
el policía (policeman) vs. la policía (police force, policewoman)
el Papa (Pope) vs. la papa (potato)
PRÁCTICA 6. Escriba el artículo correcto (masculino “el” o femenino “la”), y anote al
frente de cada sustantivo la "regla" correspondiente.
Ejemplo: _el_ barrio __A1: los sustantivos terminados en -o son masculinos__
____ problema ______ ; ___ mano ______ ; ____ fraternidad ______ ; ____ dama
_______ ; ___ agua ____ ; ___ nación _______ ; ___ dentista ______ ; ___ verde
_____ ; ___ orden _____
7. Plural Forms/ Formación del plural
1
ending in vowel, add -s
un libro , la mesa
unos libros, las mesas
2
ending in consonant or accented
vowel, add -es
un papel, la red, el
rubí
una imagen
papeles, redes, rubíes
las imágenes
3
ending in an unstressed vowel
+s:
no change
el lunes, la crisis
but: el interés
los lunes, las crisis
los intereses
4
ending in z --> ces
el lápiz, la vez
los lápices, las veces
According to the rules for the written accent, some words will lose it in the plural, while a
few will gain one. The rules must be applied to the new word formed in the plural:
reacción —> reacciones • alemán —> alemanes
examen —> exámenes • imagen —> imágenes
PRÁCTICA 7. Cambie al plural los siguientes sustantivos y añada el artículo correcto
(―los‖ masculino o ―las‖ femenino):
luz _las luces_
día ______
acción ________________
9
clase __________________
vez _______ lápiz ______________ alemán ___________ parte _________________
lección _________ imagen __________ alma ________ coche ________________
ciudad ___________ foto ________ invención __________ examen ________________
universidad _______________ parabrisas ____________ análisis _________________
8. The Indefinite Article / El artículo indeterminado
English "a" or "an" is expressed in Spanish by un/una: un cuaderno • una pluma
The plural, unos/unas, is used for symmetrical objects, meaning ―a set of‖ or ―a pair of‖:
Necesito unas tijeras.
I need a pair of scissors.
Bajas por unas escaleras.
You go down a stairway.
Spanish does not use the indefinite article before words denoting occupations, religions,
political affiliation or nationality; they are treated as adjectives:
He is English, an Englishman.
Él es inglés.
No soy budista.
I am not a Buddhist.
Mi hermano es demócrata.
My brother is a democrat.
Are you a student?
¿Eres estudiante?
Your neighbor is a lawyer.
Tu vecino es abogado.
But when such words are qualified by an adjective, the article is used:
He is a very traditional Englishman.
El es un inglés muy tradicional.
No soy un norteamericano típico. I am not a typical North American.
Spanish uses the indefinite article less frequently than English. Since un/una also means
one, the article would be redundant when referring to things normally encountered one at
a time:
I have a fever. Do you have a family?
Tengo fiebre. ¿Tienes familia?
Is a visa needed for traveling?
¿Se necesita visa para viajar?
The woman was wearing a hat.
La mujer llevaba sombrero.
He speaks with a foreign accent.
Habla con acento extranjero.
You cannot drive without a license.
No puedes manejar sin licencia.
Use this as a guide.
Use esto como guía.
Como buen gato, le encanta dormir. As a good cat, s/he loves sleeping.
Pay attention to these differences as you read more Spanish. Note that the indefinite
article is used to emphasize the individuality of the noun, which is then modified by an
adjective (as seen also in the case of professions and nationalities):
Tiene una fiebre terrible. Esa mujer lleva un sombrero muy elegante. ¿Se necesita una
visa especial?
Never use un/una before the word otro/a:
otro ejemplo
otra situación
10
otra vez
another example
another situation
again (another time)
Other common expressions not requiring the indefinite article in Spanish:
What a day! What a long year!
¡Qué día! ¡Qué año tan largo!
Medio kilo de patatas. Media vida. Half a kilogram of potatoes. Half a life.
Es para cierta persona. Cierto
It‘s for a certain person. A certain charm.
encanto.
PRÁCTICA 8. Añada el artículo indefinido cuando sea necesario. Si no es necesario,
escriba una X en el espacio.
Ejemplo: Hoy tengo _un_ examen difícil, y tengo _X_ otro examen mañana.
1. Como ___ buen budista, le encanta meditar, pero hoy no puede porque tiene ___
fiebre.
2. Mi padre es ___ arquitecto y hoy tiene ____ día muy ocupado. ¡Qué ___ vida tan
difícil!
3. Soy ___ amante de los libros porque tuve ___ buen profesor en la secundaria.
4. Este joven tiene ____ futuro como ___ abogado: sabe debatir con ____ cierta
creatividad.
5. Es ___ mujer responsable, pero no tiene ___ trabajo.
6. Tengo ___ otra pregunta: ¿La palabra "cárcel" necesita ___ acento?
9. The Definite Article / El artículo determinado
There are four forms: el, la, los, las:
la clase • el reloj • los cuadernos • las plumas
A. USAGE OF THE DEFINITE ARTICLE. Spanish uses it more often than English:
A1. Before nouns intended in a general sense and all abstract nouns:
Los mexicanos adoran las fiestas.
Mexicans love parties.
La gente piensa que el dinero es vital. People think that money is vital.
La libertad es esencial en la vida.
Liberty/freedom is essential in life.
Me gusta el pescado.
I like fish.
Me interesa la política.
I am interested in politics.
A2. Before languages, illnesses, sports, sciences and other fields of knowledge, except
after the verbs hablar, estudiar, tener and saber. It is often omitted after the prepositions
de and en:
El inglés es fácil. Así es la biología. Lucha contra el cáncer. Le interesa el
fútbol.
El libro está en español. Me gusta la clase de alemán. No hablo francés.
11
A3. Before titles when not used in direct address:
El escritor mexicano Octavio Paz ganó el premio Nóbel en 1992.
El señor García habla con el coronel Páez sobre la reina Isabel.
BUT:"Buenos días, señor García."
(sobre: about)
A4. Before the words cama, escuela, colegio, trabajo, guerra, cárcel, ciudad, iglesia,
and clase (except in stock phrases that need to be learned one by one, such as ―at war‖:
en guerra, ―in class‖: en clase).
No piensan en el trabajo. Están en la cárcel. Leen para la clase. Vivo en la
ciudad de México.
A5. Apart from the names of a few countries and cities (la República Dominicana, Los
Ángeles, La Paz, El Salvador, etc.) the article is also used when the name of a country is
qualified by an adjective:
La España turística, el Perú moderno.
PRÁCTICA 9A. Añada el artículo definido cuando sea necesario. Si no es necesario,
escriba una X en el espacio. Ejemplo: Me gustan_los_ libros en _X_ español.
1. ____ trabajo y ____ escuela son importantes en ____ sociedad.
2. ____ señor Pérez va a ____ iglesia todos ____ días.
3. Me gusta ____ poesía y leo ____ libros con frecuencia.
4. Está en ____ Italia porque quiere estudiar ____ Roma antigua.
5. Queremos tomar ___ clase que enseña ____ profesora Gómez.
6. ___ gente joven prefiere ____ vida activa.
7. ___ fútbol es ___ deporte nacional de ____ brasileños.
8. Estudio ___ griego porque me interesan ____ filósofos clásicos.
9. ___ español y ___ quechua son ___ idiomas oficiales de Perú.
10. Necesitamos mejorar ___ educación en todos ____ países de__ mundo.
B. CONTRACTIONS AL AND DEL
The prepositions a and de contract with the masculine singular article to form the only
two written contractions in Spanish, al and del.
al trabajo (a + el)
del profesor (de + el)
C. OTHER PECULARITIES IN THE USE OF ARTICLES
C1. Articles, either definite or indefinite, are not used when there is an idea of amount or
quantity (if the words ―any‖ or ―some‖ can be inserted in English):
There are (some) people who do not eat
Hay gente que no come carne.
(any) meat.
We ask for (some) wine and they give us
Pedimos vino y nos dan agua.
water.
Viven sin libertad, pero con dinero. They live without (any) freedom but with
12
Leo libros con frecuencia.
Quieren manzanas para el
desayuno.
(some) money.
I read (a number of) books frequently.
They want apples for breakfast.
Note that hay (there is/are) always indicates a certain quantity, so it is never followed by
the definite article: Hay conflictos y no hay libertad. Hay un problema.
C2. Prepositions such as con (with) and sin (without), when involving amount, omit the
article: sin interés, con esfuerzo.
C3. Occasionally, unos/as expresses "some" when it means "a few," ―a number of‖ or
―about‖ (but algunos/as is a more common way to express some. See §17). Use un poco
de (a little) before mass nouns:
I still have some (a few) dollars.
Todavía tengo unos dólares.
There are some (about) ten people.
Hay unas diez personas.
Quieren unas manzanas y un poco
They want a few apples and some water.
de agua.
C4. In addition to el/la/los/las, Spanish has a neuter article, lo, used with the masculine
singular form of an adjective to express abstract concepts, very much like the English
―the+adjective+thing‖:
Eso es lo interesante.
That is the interesting element.
Lo malo es que nos gusta.
The bad thing about it is that we like it.
Lo bueno, lo malo y lo feo de esta
The good, the bad, and the ugly in this work.
obra.
PRÁCTICA 9B. Complete el texto con los artículos necesarios. Entre paréntesis, escriba
el numeral de la sección A (Usage), B (contractions) o C (peculiarities) correspondiente a
cada caso.
Panamanian composer Rubén Blades, _El (A3)_ compositor panameño Rubén Blades,
born in 1948, is a multifaceted
nacido en 1948, es una celebridad multifacética
celebrity whose interests include
cuyos intereses incluyen ______ música, ______
music, film and politics. He grew up
cine y ______ política. Creció en Panamá, pero
in Panama, but immigrated to New
inmigró a ______ ciudad de Nueva York en
York City in 1974 with only a few
1974 con sólo ______ dólares en el bolsillo.
dollars in his pocket. As a singer, he
Como cantante, comenzó a experimentar con
began to experiment with salsa music, ______ música de salsa, añadiendo elementos de
adding elements of jazz and rock, and jazz y rock, y explorando ______ bueno, ______
exploring the good, the bad, and the
malo y ______ feo de ______ vida en ______
ugly of life in the barrios of New York. barrios de Nueva York. ______ diez millones de
Some ten million copies of his songs, copias de sus canciones, como "Plástico" y
like "Plástico" and "Pedro Navaja,"
"Pedro Navaja", se vendieron en los años 1970.
were sold in the 1970s. Later on, he
Posteriormente, asistió a ______ escuela de
attended Harvard Law School and
derecho en Harvard y se hizo abogado. En 1994,
became a lawyer. In 1994 he went
regresó a su patria y recibió muchos votos al
13
back to his homeland and made a
strong run for the presidency of the
country. Blades' music continues to
evolve with great success. "La rosa de
los vientos," one of his recent
productions, sends a message of hope
to modern Panama, emphasizing the
importance of freedom and dreams in
life.
postularse para ______ presidencia ______ país.
______ música de Blades continúa
evolucionando con gran éxito. "La rosa de los
vientos", una de sus recientes producciones,
envía un mensaje de esperanza ______ Panamá
moderno, enfatizando la importancia de ______
libertad y de ______ sueños en ______ vida.
10. Negation / La negación
To make a statement negative, place the particle no immediately before the verb:
Somos americanos —> No somos americanos
Tengo tiempo —> No tengo tiempo
PRÁCTICA 10. Añada el artículo definido o indefinido correcto cuando sea necesario.
Si no es necesario, escriba una X en el espacio. Después escriba la oración en forma
negativa.
Ejemplo: Quiero _X_ sopa para _el_ almuerzo --> No quiero sopa para el almuerzo.
1. Hay ___ gente con ___ talento en ___ programa.
2. Tenemos ____ problemas con ___ geometría.
3. Este es ____ otro tema que me interesa.
4. ___ sistema solar es ____ conjunto de ____ planetas alrededor de ____ estrella.
5. ___ portugués es fácil para ____ gente que habla ____ español.
6. ___ interesante de__ cuento es ___ final. ___ ridículo es el nombre ____ personaje.
7. ___ Señor Martínez es ___ abogado. Él habla mucho con ____ doctora Pérez.
8. Sin ___ duda, ___ ciudad tiene ___ futuro. ¡Qué ____ buena cosa!
9. Puedo dormir sin ___ cobija porque no tengo ___ frío.
10. Gloria es de ___ capital de ___ Bolivia. Es ___ artista.
11. Respetamos ___ libertad y ___ orden. Estamos interesados en ____ democracia.
12. Necesitamos pagar ____ otra cuenta si queremos ____ visa para entrar a__ país.
Repaso número uno.
Exprese en español, usando las formas de “tú”:
1. Do you have another exam? No, I don't have any exams today.
2. Are you a teacher? Yes, I am an excellent teacher.
14
3. Portuguese is easy.
Englishmen do not speak much.
4. Where is he from? Is he from Canada? No, he is Venezuelan.
5. Professor Vélez studies tourism in modern Spain.
6. There are some people without any money.
7. Another example is politics—there are certain people who hate the subject.
Práctica escrita
A. Exprese en español, usando las formas de “tú” cuando sea necesario:
1. Are you an architect? Do you have a license?
2. I am Peruvian; I'm from the city of Lima. Many tourists visit my country.
3. I don't have a fever. I have another problem: I have a boyfriend!
4. Friends are important, love is necessary.
5. The interesting thing about (de) the short story is the magic.
6. What a beautiful morning! A certain topic. Half a program.
11. Adjectives / Los adjetivos
A. FORMATION AND AGREEMENT / FORMACIÓN Y CONCORDANCIA
Adjectives must agree with the noun they refer to (that is, if the noun is feminine plural,
then the adjective must be too) whether they come next to the noun or are separated from
it:
Los españoles orgullosos. Los españoles están muy orgullosos.
Hay gente honesta. Hay gente que es siempre muy honesta.
Most adjectives ending in -a or -o have four forms to agree with a noun if it's masculine
(-o), feminine (-a), or plural (-os, -as): un día largo, una clase larga, dos días largos,
tres clases largas.
15
Adjectives which end in -sta, -ta, like idealista, egoísta, idiota, etc., are the same in the
masculine and the feminine (indígena, also):
un hombre idealista • las sociedades capitalistas
los políticos demócratas • un texto indígena
Most adjectives ending in -e or a consonant also have only two forms, singular and
plural:
una profesora inteligente • un poema difícil
dos ensayos útiles • muchas novelas importantes
Exception: a small group of adjectives ending in consonants do have separate feminine
forms. The commonest of these are adjectives of nationality and those ending in -ón or or:
una estudiante alemana • dos novelas españolas
una ciudad inglesa • una mujer trabajadora
Adjectives that qualify nouns of mixed gender are masculine:
El flamenco y la paella son españoles. • Las novelas y los poemas son artísticos.
The adjectives bueno and malo are shortened to buen and mal before masculine singular
nouns:
un buen año • un mal día (but: los buenos años, los malos días)
Grande shortens to gran before a singular noun, masculine or feminine:
un gran hombre • una gran ciudad (but: las grandes ciudades, los grandes
hombres)
PRÁCTICA 11A. Escriba la forma adecuada del adjetivo entre paréntesis:
las venezolanas _______________
los peruanos __ inteligentes__(inteligente)
(brillante)
la ciudad ________________(inglés)
la gente __________________(trabajador)
los
los poemas ___________________(sincero)
indígenas ___________________(amistoso)
una cuenta ___________________(ilegal) el agua _________________(sucio)
B. PLACEMENT / POSICIÓN EN LA ORACIÓN
• Descriptive adjectives that distinguish one noun from another tend to come after the
noun:
Es un médico sensato. El automóvil rojo y la casa verde.
• When modified by an adverb, adjectives almost always follow:
Un hombre muy bueno • Una clase increíblemente aburrida
• The adjective grande means "great" when used before the noun, "large" when used
after:
New York is a great city.
Barcelona es una gran ciudad.
It is also a large city.
Es también una ciudad muy grande.
• Adjectives of quantity (cantidad), which do not stress difference, precede the noun:
muchos amigos • tres hermanos • algunos individuos • demasiados problemas
16
• ambas manos • varios temas • ninguna letra
PRÁCTICA 11B. Reemplace el sustantivo subrayado en cada oración por el que
aparece entre paréntesis, haciendo todos los cambios necesarios:
1. En realidad es un problema fácil, pero parece complicado. (lección)
2. Todas las jóvenes españolas deben estar bien informadas. (niños)
3. Tiene el brazo muy largo aunque poco musculoso.
4. Prefiero un auto pequeño porque es más barato.
(pierna)
(familia)
5. Dicen que el artista francés es buenísimo, pero el alemán es muy simpático.
cantante)
(la
6. Hay varias regiones que son muy frías. (planetas)
C. NOMINALIZATION ("... the green one", "... the old ones")
• While English usually avoids repeating nouns by the use of the word "one", Spanish
simply leaves out the noun, keeping the article which in such case acts as a pronoun
(artículo pronominal).
The Spanish style and the English one.
El estilo español y el inglés.
Cold days and hot days.
Los días fríos y los calientes.
The house in the city and the one in the
La casa de la ciudad y la del campo.
countryside.
• Similarly, with the possessive expressions rendered in English with 's (John's books)
and in Spanish with de, the article is used to avoid repetition:
Los hermanos de Juan y los de Pedro. Juan's brothers and Pedro's.
PRÁCTICA 11C. Complete las oraciones con las terminaciones y los artículos
adecuados.
1 . Voy a comprar la casa barat___, no ___ car__.
2. Jorge adora ___ clase de biología porque es interesant___, pero detesta ____ de
economía porque es aburrid___.
3 . ___ mano de Pedro es muy suav__, pero ____ de María es más aristocrátic___.
17
4. Uso dos autos: ___ viej__ para ir al trabajo y ___ nuev__ para ocasiones especial___.
5. ____ policía de Nueva York es más estrict___ que ____ de Boston.
D. COMPOUND NOUNS (COFFEE CUP, SPANISH BOOK, WRITING PAPER)
Spanish cannot put two nouns together in this way. Link them with a preposition, usually
de:
papel para
writing paper
book fair
feria del libro
escribir
clase de español Spanish class
vacaciones de verano summer vacation
horario de
office schedule
film producer
productor de cine
oficina
PRÁCTICA 11D. Exprese en español:
1. a paper cup 2. the biology book 3. a delicious wine glass
4. a great film director
5. a metal detector
6. the train schedule 7. the meat market
8. spring vacation
9. the winter boots
team
10. the dog house 11. a one-way (una vía) street 12. a rescue
12. Questions / Las preguntas
A. WORD ORDER / EL ORDEN DE LAS PALABRAS
• The simplest way to form a question is to change the intonation:
Los brasileños no hablan español. --> ¿Los brasileños no hablan español?
Notice that written Spanish warns the reader that a question or exclamation is coming
up by using an inverted question or exclamation mark at the beginning of the phrase.
• Changes in word order are also common in Spanish: If you have only a subject and a
verb, reverse the order: Ellas estudian. --> ¿Estudian ellas?
• If no subject is expressed, only one word order is possible:
Estudian en la universidad. -->
¿Estudian en la universidad?
• If there is subject/verb/object, keep the verb and predicate together, putting the subject
at the end:
Los hijos del señor Pérez tienen dinero. --> ¿Tienen dinero los hijos del señor
Pérez?
• If the predicate is long, however, the subject follows the verb:
Los artistas crean obras magníficas. -¿Crean los artistas obras magníficas?
>
18
PRÁCTICA 12A. Convierta en pregunta las siguientes oraciones.
Ejemplo: México está en Norteamérica --> ¿Está México en Norteamérica?
1. Guatemala está en Centroamérica.
2. Miguel Ángel Asturias fue un famoso escritor guatemalteco.
3. Las novelas de Asturias hablan sobre los grupos indígenas.
4. Hay muchas culturas indígenas en Guatemala.
5. Los indígenas guatemaltecos participan en la política.
B. QUESTION WORDS / PALABRAS INTERROGATIVAS
¿Qué?
¿Quién? ¿quiénes?
¿Cuál? ¿Cuáles?
¿Dónde? ¿Adónde?
¿Cuándo?
¿Cómo?
¿Por qué?
¿Para qué?
¿Cuánto/a/os/as?
What, which?
Who? [notice plural option]
Which (one)?
Where? To where?
When?
How?, what... like?
Why? (For what reason?)
Why? (With what aim or intention?)
How much, how many?
• Interrogative words have a written accent because they are used, unaccented, in other
ways:
I study when I have classes.
Cuando tengo clases, estudio.
Ese es el edificio donde vivo.
That is the building where I live.
Habla como político.
He speaks as a politician.
• Accented forms are also used in indirect questions:
She asks me who I am.
Me pregunta quién soy.
I do not know where he lives.
No sé dónde vive.
It does not matter when you call.
No importa cuándo llames.
• The accented forms are also used in exclamations such as:
What a film!
¡Qué película!
How he(she) talks!
¡Cómo habla!
1) ¿QUÉ?
• Qué is used like English what? asking for a definition or explanation:
¿Qué es esto?
What is this?
What are you looking for?
¿Qué buscas?
¿Qué significa eso?
What does that mean?
• Qué is also used next to a noun (as an adjective) meaning which:
¿Qué libro quiere usted?
Which book do you want?
¿Qué palabra no entiende Ud.?
Which word don‘t you understand?
2) ¿CUÁL? ¿CUÁLES?
• Cuál is the pronoun (not an adjective) for "which (one)", and it has a plural form:
¿Cuál de los programas es bueno? Which of the programs is good?
There are several poems. Which (ones) do you
Hay varios poemas. ¿Cuáles
19
prefieres?
prefer?
• Instead of English "What is the capital of Perú?‖, Spanish says:
(Which of the cities is the capital?)
¿Cuál es la capital del Perú?
• Similarly, questions like "What are the best ideas?", which use the verb ser and
involve choosing from a number of possibilities, use cuál or cuáles:
¿Cuál es la autora? ¿Cuál es tu nombre?
¿Cuál es el significado de ―envolver‖? (¿Qué significa...?)
¿Cuál es la respuesta correcta? ¿Cuál es tu teléfono?
¿Cuáles son las películas más recientes?
To a question like ―¿Qué es el teléfono?‖ a speaker of Spanish would respond with a
definition: ―Es un aparato para hablar a distancia‖.
PRÁCTICA 12B. Complete con qué, cuál, o cuáles:
1. ¿_________ prefieres: ir al cine o ver la televisión?
2. Tengo dos candidatos posibles, pero no sé _______ escoger.
3. ¿_________ es tu opinión? No sé.
4. ¿_________ clases tienes ahora? Ninguna.
5. ¿_________ son las características de tu país? Es grande y diverso.
6. ¿_________ es tu dirección? Calle Colón, número veinte.
7. No sé _______ helado prefiero, si el de vainilla o el de chocolate.
8. Hay dos sabores: ¿_________ prefieres? Chocolate.
9. ¿_______ es el significado de "elegir"? Significa "to choose".
10. ¿_______ es "envolver"? Significa "to wrap".
13. Números: 0-100
0
1
2
3
4
5
6
7
8
9
cero
uno/a
dos
tres
cuatro
cinco
seis
siete
ocho
nueve
10
11
12
13
14
15
16
17
18
19
diez
once
doce
trece
catorce
quince
dieciséis
diecisiete
dieciocho
diecinueve
20 veinte
21 veintiuno, veintiún
22 veintidós
23 veintitrés
24 veinticuatro
25 veinticinco
26 veintiséis
27 veintisiete
28 veintiocho
29 veintinueve
30 treinta
31 treinta y uno /un
32 treinta y dos
43 cuarenta y tres
54 cincuenta y cuatro
65 sesenta y cinco
76 setenta y seis
87 ochenta y siete
98 noventa y ocho
100 cien
Note: uno shortens to un before a masculine singular noun (e.g. un dólar). All other
numbers ending in 1 act the same way (e.g. veintiún libros, treinta y un alumnos, etc.).
All numbers ending in 1 that refer to feminine nouns become feminine: noventa y una
mujeres (But: setenta y dos señoras).
14. Time of Day / La hora
20
• The basic formula is:
¿Qué hora es? Son las dos.
What time is it? It's two o'clock.
• Other cases:
Es la una de la mañana.
Son las tres y media de la tarde.
¿A qué hora tienen ustedes clases?
Estudiamos a las ocho menos
cuarto.
• For time after the hour, use y:
Son las dos y cuarto.
a la una y media
Son las cinco y veinticinco.
• For time before the hour, use menos:
It is 1:00 am.
It is 3:30 pm.
At what time do you have classes?
We study at a quarter to eight.
It's 2:15
at 1:30
It's 5:25
Son las cuatro menos veinte.
It's twenty to four (3:40).
• Spanish does not often use the literal translation of expressions like 5:45 except for
timetables.
Use las seis menos cuarto.
• For am. and pm. use de la mañana, de la tarde, and de la noche.
• Note also the following expressions:
por la
in the morning
at mid-day, at noon
mañana
al mediodía
in the afternoon
por la tarde
at midnight
a medianoche
at night
por la noche
• Note that the word tiempo refers to time as duration:
Do you have time now?
¿Tienes tiempo ahora?
We have little time left.
Nos queda poco tiempo.
He thinks he'll get better in time.
Cree que mejorará con el tiempo.
How long (How much time) do the batteries
¿Cuánto tiempo duran las pilas?
last?
Al mismo tiempo.
At the same time (simultaneously).
PRÁCTICA 14. Exprese en
español:
1. It's 4:30
2. It's 3:15
4. At 6:27
5. At 11:45
7. At three pm
8. At noon
3. It's 12:25
6. At 4:55
9. At 9:12 pm
10. 88 difficult problems 11. 91 brilliant ideas
12. 73 paper cups
13. There are 16 wonderful options, but I do not have time now.
Repaso número dos
A. Exprese en español, usando las formas de "tú" (cover the answers as you practice):
21
1. Do you have another exam? Yes, I have two long exams today.
2. Are you a writer? Yes, I am a good writer.
3. English pencils and pens are very expensive.
4. There are some sincere individuals with interesting ideas.
5. Is the new president a Buddhist? No, he is an idealistic atheist.
6. What is your favorite movie? Why?
7. Which car do you prefer? The blue one is fast, but I prefer the red one.
B. Complete las preguntas y respóndalas con una palabra o dos.
1 . ¿___________ es la capital de Cuba? _________________________
2 . ¿___________ se llaman las personas de Perú? _______________________.
3 . ¿___________ (#) países tienen como lengua oficial el español? (number:)________.
4 . ¿En __________ país está Tegucigalpa? _______________________.
5 . ¿De ___________ (origen) son los madrileños? _____________________.
Práctica escrita.
A. Exprese en español, usando las formas de “tú” cuando sea necesario:
1. Do they speak English in Canada? Yes, and they also make paper bags.
2. What do you have in your (the) hand? I have a few old coins.
3. Big cities and small ones have weird people.
4. How many good friends do you have? Which are honest?
5. What's the time? It's 4:30 pm. It's not 1:15 am!
6. Who are the noisy tourists? They are not tourists; they are politicians.
7. How does she learn French? What is her secret?
22
B. Complete el siguiente párrafo con las terminaciones y los artículos adecuados.
Después, escriba tres preguntas sobre el tema y respóndalas.
Pablo Neruda (1904-1973), ____ famos__ escritor
chilen__, es autor de numeros___ poemas que
much___ gente hispanohablant__ lee con intens___
interés. ____ poesía de Neruda es muy divers___, e
incluye temas romántic___, polític___, históric___ y
filosófic___. ____ obra muy conocid__ entre ____
estudiantes de español es ___ libro titulad__ Odas
elemental___, porque ____ vocabulario es bastante
sencill__ y ____ imágenes son ___ mismo tiempo
íntim____, profund____ y accesibl____.
Pablo Neruda (1904-1973), the famous
Chilean writer, is the author of numerous
poems that many Spanish-speaking people
read with intense interest. Neruda's poetry
is very diverse, and it includes romantic,
political, historical and philosophical
themes. A very well known work among
Spanish students is the book titled
Elementary Odes, because the vocabulary
is quite simple and the images are at the
same time intimate, profound and
accessible.
15. Common expressions with tener/ Expresiones
comunes con ―tener‖
How old are you?
I am twenty.
She is very cold.
Are you thirsty?
She is hungry.
I am warm.
We are sleepy.
They are afraid [of...]
I am not very lucky.
You are right.
Be careful!
I have to do a lot of things.
I have a lot of things to do.
¿Cuántos años tienes?
Tengo veinte años.
Ella tiene mucho frío.
¿Tenéis sed?
Ella tiene hambre.
Tengo calor.
Tenemos sueño.
Tienen miedo (de...)
No tengo mucha suerte.
Tienes razón.
¡Ten cuidado!
Tengo que hacer muchas cosas.
Tengo muchas cosas que hacer.
Since the literal meaning of these phrases is "I have hunger", etc., Spanish uses adjectives
where English needs adverbs such as "very":
Tenemos mucho sueño.
We are very sleepy.
Tengo demasiada hambre.
I am too hungry.
PRÁCTICA 15. Complete las oraciones lógicamente, con una de las expresiones
anteriores.
1. No juego en los casinos porque _______________________________________.
2. Mi hermana es muy joven; solamente _________________________ años.
3. Quiere una cobija adicional porque _________________________.
4. Necesito un poco de agua porque _________________________.
5. Vamos a dormir ahora porque __________________________.
6. Muchos niños ________________________ la oscuridad.
23
7. ¿_______________________? Prefiero no revelar mi edad.
8. No puede ir a la fiesta porque ____________________ trabajar.
9. Necesitas ____________________ al cruzar la calle.
10. No quiero comer ahora porque no _______________________.
11. ¿Puedes poner el aire acondicionado? _________________________.
16. Adverbs / Los adverbios
A. GENERAL USE
Adverbs provide more information about verbs or adjectives by describing how an action
is performed, how often it is done, how intense a quality is, etc.:
She speaks well.
Habla bien.
We do not study too much.
No estudiamos demasiado.
It is quite difficult.
Es bastante difícil.
Adverbs have no feminine or plural forms, and are placed as close to the verb as possible:
They're quite difficult.
Son bastante difíciles.
The pears are too old.
Las peras están demasiado viejas.
Me gustan mucho las fiestas.
I like parties a lot.
Some adverbs, however, are also adjectives. As such, adjectives must agree with the noun
in number and gender: Tiene muchos problemas, bastantes preocupaciones y
demasiadas deudas.
PRÁCTICA 16 A. Complete the following sentences with a logical form of bastante(s),
mucho(a/s), poco(a/s) or demasiado(a/s). Circle the instances in which these words are
adverbs:
1. Estudio _____________, pero tengo _____________ clases.
2. Mis clases son ______________ difíciles, pero hay ___________ gente que me ayuda.
3. _______________ de mis profesores dicen que me preocupo ___________________.
4. Quiero aprender __________ y ser _____________ profesional en mi trabajo.
B. FORMATION OF ADVERBS FROM ADJECTIVES
•
•
To form an adverb from an adjective add -mente to the feminine form of an adjective:
lento
lentamente
rápido
rápidamente
feliz
felizmente
difícil
difícilmente
The suffix –mente is rarely used with adjectives ending in –nte. Use adverbial
phrases instead:
interestingly
de manera interesante
frequently
con frecuencia
intelligently
con inteligencia, de forma inteligente
elegantly
con elegancia
24
•
•
importantly
de manera importante
When used in a series, only the last adjective will add the suffix –mente:
Trabaja rápida y eficazmente. He works quickly and efficiently.
The common adjectives bueno and malo have their own corresponding adverbial
forms:
(good) bueno
bien (well) (bad) malo
mal (badly)
Ella habla bien el italiano, pero canta mal.
PRÁCTICA 16B. Termine las oraciones con los adverbios o las expresiones adverbiales
correspondientes:
1. Tiene una mente rápida. Piensa ______________________.
2. Siempre tiene mucho cuidado. Actúa _______________________.
3. Es un hombre lento y cauteloso. Procede ______________ y _____________________.
4. Tiene un estilo muy elegante. Escribe _______________________________.
5. Es una persona interesante. Vive ________________________________.
6. La película tiene un final feliz. Termina _____________________.
7. Tenemos problemas sociales y políticos. Necesitamos mejorar __________ y
_________________.
17. Negative words and their opposites
Las palabras negativas y sus opuestos
A. NUNCA, NADA, NADIE
nunca
never
una vez
once
a veces
sometimes
muchas veces
often
siempre
always
nada
nothing
algo
something
cualquier cosa
anything
todo
everything
nadie
nobody
alguien
somebody
cualquiera
anybody
todo el mundo
everybody
• Negative pronouns and adverbs may precede the verb, but if they follow, no must
precede:
Nadie estudia geología.
Nobody studies geology.
We never go out.
Nunca salimos.
I never study.
No estudio nunca.
There is no one here.
Aquí no hay nadie.
• Negatives do not cancel each other in Spanish:
I never send anything to anyone.
Nunca mando nada a nadie.
No tengo dinero ni trabajo.
I do not have money or a job.
We want nothing.
No queremos nada.
• Spanish does not usually express "any" or "a" in negative expressions such as:
25
"I don't have any idea(s)" - no tengo idea(s) or "I don't need a car" no necesito coche.
PRÁCTICA 17A. Responda negativamente las preguntas.
Ejemplo: ¿Acepta cualquier cosa? No acepta nada.
1. ¿Todo el mundo quiere algo?
6. ¿Alguien tiene algo aquí?
2. ¿A veces habla por teléfono?
7. ¿Todo el mundo tiene problemas?
3. ¿Tiene algo en la mano?
8. ¿Alguien tiene dinero?
4. ¿Cualquiera tiene razón?
9. ¿Siempre comes cualquier cosa?
5. ¿Tenemos que hacer algo?
10. ¿Tiene que hacer todo varias veces?
B. NINGÚN, ALGÚN, CUALQUIER
Ningún tema es bueno.
No topic is good.
Algunos temas son buenos.
Some topics are good.
Cualquier tema es bueno.
Any topic is good
Ninguno/a (none, no) shortens to ningún before a masculine, singular noun. When it
follows the verb, a double negative is required:
There is no problem.
No hay ningún problema.
¿Tenéis amigos? No tenemos
Do you have any friends? We have none.
ninguno.
No society is perfect.
Ninguna sociedad es perfecta.
Ninguno/a is never used with plural nouns (with some unimportant exceptions):
They have no relatives. No tienen ningún pariente. No tienen parientes.
There are no difficulties. No hay ninguna dificultad. No hay dificultades.
Alguno (some, any) shortens to algún before a masculine, singular noun, and varies in
gender and number according to the noun it refers to: alguno, alguna, algunos, algunas:
Is there any museum in the city?
¿Hay algún museo en la ciudad?
I need a taxi. Do you see any?
Necesito un taxi. ¿Ve Ud. alguno?
Algunas sociedades son muy estrictas. Some societies are very strict.
Cualquiera (any at all) shortens to cualquier before any noun, and normally occurs in the
26
singular:
Llámenme si hay cualquier problema. Call me if there are any problems.
I need a taxi. Any one at all!
Necesito un taxi. ¡Cualquiera!
Any society has some government.
Cualquier sociedad tiene gobierno.
PRÁCTICA 17B. Escriba en español:
1. He has no friends.
2. Are there some difficult words? None.
3. Any city has some restaurants.
4. Which flavor do you want? Any.
5. Some regions have no mountains.
6. There are no possibilities for progress.
7. No problem is impossible.
8. Is there any bank in this city? Some.
C. (N)EITHER, (N)OR
ni nor
ni... ni... neither...nor
tampoco neither,
not...either
o or
o... o...
también
either...or
also, too
Ni (nor, not even) often translates into English "or" in negative sentences:
A esta hora no hay taxis ni buses. There are no taxis or buses at this time.
We don't want to study nor sleep.
No queremos estudiar ni dormir.
Not even the best society is perfect.
Ni la mejor sociedad es perfecta.
Ni siquiera is a stronger way to express "not even":
Nunca escribe, ni siquiera para
He never writes, not even for Christmas.
Navidad.
Ni...ni... is the Spanish equivalent for English "neither...nor...". Note the verb in plural
when there's more than one subject, as well as the usual need of a negative word before
the verb:
I'm neither cold nor hot.
No tengo ni frío ni calor.
Neither Juan nor Pedro is hungry.
Ni Juan ni Pedro tienen hambre.
27
In negative phrases, English "either...or..." translates into ni:
He's here without either family or friends.
Está aquí sin familia ni amigos.
When no negative particles are involved, "either...or" translate into o:
We can either go to a movie or watch
Podemos ir a cine o ver la
television.
televisión.
To emphasize compelling choices, o...o... may be used:
You have to choose: either you work or you
Tienes que elegir: o trabajas o
study.
estudias.
Tampoco (neither, not...either) is used in any negative phrases as the opposite of también
(also):
She wants some coffee and so do I.
Quiere café y yo también.
She doesn't want any coffee. Neither do I.
No quiere café. Yo tampoco.
No cantamos esta noche tampoco. We are not singing tonight either.
Also, the government does not have the
El gobierno tampoco tiene la
answer.
respuesta.
PRÁCTICA 17C. Haga negativas las siguientes afirmaciones:
1. Hay carne o pescado.
2. El doctor y su esposa tienen mucha hambre.
3. Tú y yo tenemos razón.
4. Come con cuchillo y tenedor.
5. Tiene que estudiar y yo también.
6. También tiene sueño siempre.
7. Hoy también tenemos que trabajar bastante.
8. Queremos sopa, arroz y ensalada también.
PRÁCTICA 17D. Cambie el orden de las siguientes oraciones incluyendo "no" antes del
verbo.
Ejemplo: Ni la madre ni el padre trabajan. —> No trabajan ni el padre ni la madre.
1. En esta familia nadie trabaja.
2. Tampoco estudia nadie.
28
3. Tampoco tienen ninguna habilidad ni inclinación artística.
4. Ni el hijo ni la hija leen nunca nada.
5. Ni siquiera los hijos practican ningún deporte.
18. Stem-changing verbs / Verbos que cambian de raíz
All Spanish verbs have a stem (la raíz) and an ending (-ar, -er, -ir): pensar, volver, pedir.
There is a large group of verbs that change their stem when the stem vowel is stressed.
There are three types: those that change -e to -ie-, those that change -o- to -ue - and those
that change -e- to -i-:
to think:
pensar (ie)
pienso
piensas
piensa
pensamos
pensáis
piensan
to come back:
volver (ue)
vuelvo
vuelves
vuelve
volvemos
volvéis
vuelven
to ask (for):
pedir (i)
pido
pides
pide
pedimos
pedís
piden
e-->ie: cerrar, empezar, comenzar, querer, preferir, perder, entender
o-->ue: encontrar, contar, volar, mostrar, poder, resolver, morir, dormir
e-->i: sentir, (son)reír, seguir, servir, elegir
Jugar has a different stem change, from u to ue in the same places:
juego, juegas, juega, jugamos, jugáis, juegan.
Note: All -ir stem-changing verbs change their stems in other tenses (see appendix A).
PRÁCTICA 18. Transforme las oraciones cambiando el sujeto indicado entre paréntesis
y añadiendo "también" o "tampoco". Ejemplos:
Nunca pienso en nada (nosotros) --->
Nunca pensamos en nada tampoco.
Siempre volvemos a tiempo (ustedes) ---> Siempre vuelven a tiempo también.
1. A veces cierran la iglesia. (nosotros)
7. Nunca encuentras nada. (Tú y yo)
2. No jugamos con nadie. (tú)
8. Ni tú ni ella entienden. (El doctor López)
3. Resuelven algunos problemas. (La señora Vélez)
9. Pedimos demasiado. (La profesora Pérez)
29
4. Sonríe por cualquier cosa. (nosotros)
10. Nunca elijo mis emociones. (ni tú ni ella)
5. Las águilas vuelan muy alto. (el avión)
11. Nunca seguimos las instrucciones. (él)
6. La clase siempre empieza a tiempo. (el concierto)
12. Nadie puede jugar hoy. (nosotros)
Repaso número tres
Escriba en español, usando las formas de “tú”:
1. Do you have another biology exam? No, I have no exams today.
2. Are you very afraid of ghosts? Yes, and there are some here at night.
3. Miss Gómez never plays with anybody after 10:45 p.m.
4. Do you have to work in the morning? No, and I don't have to study either.
5. Everybody is asking for something desperately and nobody ever finds the answer.
6. Frequently we do not solve problems adequately.
7. For some reason, they do not have any symptoms.
8. Third World countries are not independent either politically or economically.
Práctica escrita.
A. Escriba en español, usando las formas de “tú” cuando sea necesario:
1. She always works a lot, and very efficiently. But she doesn‘t have any money.
2. He is never too hungry. What is his secret?
3. I like sports a lot but I never play after 10 pm.
30
4. Do you have something? No, I have nothing either.
5. I am never right; not even in dreams. But sometimes I fly!
6. Neither you nor anybody ever understands my emotions.
7. You have to choose: either you come back early or you never come back!
B. Complete el siguiente párrafo con las terminaciones y los artículos adecuados. Después, escriba
tres preguntas sobre el tema y respóndalas.
Barcelona es ___ capital polític__ y cultural de ___
Comunidad Autónom__ de Cataluña. Tiene
aproximad__________ dos millones de habitantes y es
___ centro industrial y comercial de much___
importancia en ___ país. Algun___ de ____
características peculiar___ de Barcelona son ___ idioma
y ___ arquitectura. Aunque ___ régimen de__ dictador
Francisco Franco intentó suprimir ___ catalán, est__
lengua está presente por tod__ ___ ciudad, y es un
símbolo de ___ ric__ tradición cultural de ___ región.
Pero ning___ otr__ símbolo de ___ ciudad es más
representativo que ___ arquitectura de Antonio Gaudí
(1852-1926), especial_______ ____ famos__ Templo de
___ Sagrad__ Familia.
Barcelona is the political and cultural capital of the
Autonomous Community of Cataluña. It has
approximately two million inhabitants and it is an
industrial and commercial center of great
importance in the coutry. Some of Barcelona's
pecular characteristics are the language and the
architecture. Although dictator Francisco Franco's
regime attempted to supress Catalonian, this
language is present all over the city, and it is a
symbol of the rich cultural tradition of the region.
But no other symbol of the city is more
representative than Antonio Gaudí's architecture,
especially the famous Temple of the Sacred
Family.
19. Ir a... to express future / El futuro con "ir a..."
Just as English says, "I am going to...", Spanish uses the verb ir and the preposition a:
Voy a cantar mañana. ¿Cuándo vas a entenderme? ¿Qué va a pasar?
Vamos a decidir la próxima semana. No váis a ver nada. No van a ir.
Some useful terms to indicate future occurrences:
después
later
mañana
tomorrow
pasado mañana
the day after tomorrow
más tarde
later
esta tarde
this afternoon
esta noche
tonight
31
esta semana
this week
la próxima semana
next week
el mes, el año próximo
next month, year
este mes
this month
el mes que viene
next month
el año que viene
next year
PRÁCTICA 19. Cambie las oraciones usando ir a y diferentes expresiones del futuro:
1. Nunca pensamos en ese problema.
4. No piensa lúcidamente por la mañana.
2. Siempre resuelvo esos problemas por la noche.
5. A veces cierran temprano esta taberna.
3. Ni tú ni ella ven claramente esta situación.
6. Con frecuencia siento hambre por la tarde.
20. Demonstratives / Los demostrativos
near the speaker
this
este
esta
these
estos
estas
not too far, often near the
person being addressed
that
those
ese
esos
esa
esas
Voy a llevar estos libros a la biblioteca.
¿Por qué vas a leer ese poema?
¿Quién es aquel señor?
Aquellos árboles son pinos.
remote
that
aquel
aquella
those
aquellos
aquellas
I'm taking these books to the library.
Why are you going to read that poem?
Who is that gentleman?
Those trees over there are pine trees.
The same words are used as pronouns (accent marks can be used to indicate they are
pronouns in cases of potential misunderstanding):
This letter and that one
Esta carta y esa (ésa)
Ese artículo es bueno, pero prefiero este. That article is good, but I prefer this one.
Este restaurante está cerrado, pero aquel This restaurant is closed, but that one
isn't.
no.
The neuter pronouns esto, eso and aquello are invariable. They are used to refer to
unidentified objects (gender unspecified), ideas or situations in a general sense:
What's that (thing) you have in your
¿Qué es eso que tienes en la mano?
hand?
This? It's an ultramodern cell phone.
¿Esto? Es un celular supermoderno.
I do not agree with that.
No estoy de acuerdo con eso.
Spanish uses definite articles for expressions like "those who..."
32
Those who read a lot, learn.
That who seeks, will find.
Los que leen mucho, aprenden.
El que busca, encuentra.
English ―like this/that‖ is usually expressed by así: Un problema así. ¿Siempre habla
así?
PRÁCTICA 20. Escriba en español:
0. Those who serve are happy.
1. Are you going to buy this cell phone? No, I prefer that one.
2. Do you understand these poems? Yes, and I want to read those, too.
3. What do you think of the national economy? Oh, I do not understand much about (de) that.
4. What is that? This is an ultramodern electronic watch. It works (funciona) like this.
21. Personal a / La a personal
In part because of its flexibility in word order, Spanish labels direct objects by putting the
preposition a before them in the following cases:
A. With any definite person, direct object of any verb not using other prepositions:
I don't know former President Clinton.
No conozco al expresidente Clinton.
The rescue team is looking for the lost
El equipo de rescate busca a la niña perdida.
child.
We're going to visit my sister.
Vamos a visitar a mi hermana.
But: Busco un secretario nuevo. (an indefinite person)
Pienso en mi hermana con frecuencia. (Pensar already uses en)
B. Before the words alguien, alguno, nadie, ninguno, cualquiera and any other
pronoun referring to people:
No necesito a nadie. No quiero a ninguno de I need no one. I don't want any of
them.
ellos.
How many are you inviting? Not
¿A cuántos invitas? No invito a cualquiera.
anyone.
Call one of them. Which one do you
Llame a alguna de ellas. ¿A cuál conoce Ud?
know?
33
C. When there might be ambiguity about which is subject and which is object:
La represión responde a la rebelión.
Repression is responding to rebellion.
The personal a is normally not used after the verb tener, except for emphasis:
Tengo tres hermanos.
Siempre tengo a mi madre.
PRÁCTICA 21. Complete los espacios con a, al, o X si no es necesaria la a personal.
1. Admiro ___ estas personas porque saben
sonreír.
2. ¿___ quién vas a llamar? ___ doctor
Martínez.
3. Necesitamos ___ un buen abogado.
4. ¿Conoces ___ Madrid? Sí, tengo ___
amigos allá.
5. Van a entrevistar ___ algunos
candidatos.
6. El gobierno combate ___ la
discriminación.
7. El gobierno combate ___ sus
enemigos.
8. Nadie critica ___ nadie.
22. Conocer y Saber
Spanish thinks of knowing in two ways. The verb conocer indicates familiarity or
recognition and is mostly used for people and places:
Conozco un lugar magnífico en el barrio
I know a great place in China Town.
chino.
No conocemos al presidente. (personal "a") We don't know the president.
Do you know Acapulco?
¿Conoces Acapulco?
Saber means to be aware of facts or information, to know things by heart, and also to
"know how to".
They can't swim. I don't know what to say!
Ellos no saben nadar. ¡No sé qué decir!
I don't know the museum, but I know where it is.
No conozco el museo, pero sé dónde está.
I know the song, but I don't know the words (by heart).
Conozco la canción, pero no sé la letra.
Two useful rules: One cannot saber people and conocer cannot be immediately followed
by que.
I know that woman.
Conozco a esa mujer.
I know she lives in New York.
Sé que vive en Nueva York.
PRÁCTICA 22. Complete las oraciones con la forma correcta de "conocer" o "saber".
Añada también la "a" personal cuando sea necesaria:
4. Nadie ___________ porqué somos
1. Carmen _________ tocar el piano.
así.
2. Voy a __________ Cancún en mi próximo
5. Yo _____________ varios senadores.
viaje.
6. Ella __________ qué hacer en estos
3. Queremos ______________ doctor Vélez.
casos.
23. Relative Pronouns / Los pronombres relativos
34
Relative pronouns link a dependent clause to a main clause, providing a smooth transition
from one idea to another. As pronouns, they refer back to a noun in the main clause,
called the antecedent. In contrast to English, the relative pronoun can never be omitted in
Spanish:
Main clause
rel.
dependent clause
Celebramos la belleza que vemos en el mundo.
We celebrate the
(that) we see in the world.
beauty
"beauty" is the antecedent in this example
A. QUE (that, which, who)
The most frequently used relative pronoun is que, which can refer to people, places,
things, and abstract ideas. If you never used any other relative, you would be right most
of the time:
Conozco al niño que estudia aquí. I know the boy who studies here.
You are the person (who) I need.
Eres la persona que necesito.
Me gustan las novelas que escribes. I like the novels (that) you write.
La gente que habla bien, tiene éxito. People who speak well, succeed.
PRÁCTICA 23A. Escriba en español:
1. That is the case I know.
2. The people who practice succeed.
3. I need the book you have.
4. You are the person he needs.
5. They prefer the house we are going to see tomorrow.
B. RELATIVE PRONOUNS USED AFTER PREPOSITIONS
When a preposition is involved (to whom, by which, etc.), the relative que cannot refer to
people, and the following forms are used:
for people only, after prepositions
for people or things, after prepositions
el que forms
el cual forms
quien, quienes
el que
los que
el cual
los cuales
la que
las que
la cual
las cuales
35
QUIEN/QUIENES is used after a preposition when the antecedent is a person:
Ese es el médico de quien hablo.
That's the doctor I'm talking about.
The children I play with (with whom I
Los niños con quienes juego.
play).
EL QUE /EL CUAL forms must be used after a preposition when the antecedent is not a
person:
Los temas sobre los que hablan son tontos. The topics they talk about are silly.
The company you work for is very
La compañía para la cual trabajas es muy
famous.
famosa.
EL CUAL forms tend to be more formal, but are basically interchangeable with EL QUE
forms. Both forms can be used for people or things after a preposition.
QUE can be used after the simple prepositions de, con, en, only when the antecedent is
not a person:
La educación es el arma con que creamos la Education is the weapon with which we create peace.
paz.
PRÁCTICA 23B. Complete las oraciones con las terminaciones correctas y con un
pronombre relativo adecuado. Nota: A veces hay varias posibilidades correctas.
1. Est__ es la mesa sobre ________ escribo mis trabajos.
2. Es___ son los amigos con __________ juego (al) fútbol todos los sábados.
3. Est__ es el aula en __________ recibo algunas de mis clases. (aula es femenina)
4. Aquell___ son los profesores _________ leen mis trabajos.
5. Est___ es el libro sobre _________ discutimos mucho en la clase de filosofía.
6. Est___ son los lápices con ________ hago mis tareas de matemáticas.
7. Est__ es la persona sin _________ yo no puedo vivir.
C. CUYO, LO CUAL, LO QUE
CUYO (A, OS, AS) is the equivalent of whose used as a relative in English (not for
questions):
Es un autor cuyos libros conozco bien. It's an author whose books I know well.
BUT: ¿De quién es esta pluma?
Whose pen is this?
Both neuter forms LO CUAL and LO QUE are used in adjective clauses, set off by
commas, that refer back to a previously stated idea. In this usage they correspond to the
English which:
Dicen que soy inteligente, lo cual es
They say I'm intelligent, which is true.
cierto.
Hay nubes, lo que indica posible lluvia. There are some clouds, which indicates possible rain.
LO QUE means what referring to an action or idea still to be mentioned:
This is what he answers.
Esto es lo que contesta.
36
What we think is not important.
Lo que pensamos no es importante.
The English all that or everything that are expressed by todo lo que:
Everything (that) she writes is good.
Todo lo que escribe es bueno.
All (that) I ask for is tranquility.
Todo lo que pido es tranquilidad.
PRÁCTICA 23C. Una las oraciones utilizando correctamente lo que o lo cual. Ejemplos:
Juan dice que no. Estoy de acuerdo con eso.
Estoy de acuerdo con lo que dice
Juan.
Juan dice que no, con lo cual estoy
de acuerdo.
1. El profesor pide atención. Eso es correcto.
3. El perro quiere salir. Eso es
normal.
2. La niña contesta que tal vez. Acepto eso.
4. El vecino dice que sí. Me parece
bien eso.
PRÁCTICA 23D. Complete con un pronombre relativo correcto.
1. Me gusta la gente con _________
4. Soy una persona _________ quiere
trabajo.
aprender.
2. Eso es _________ tú piensas.
5. Trabaja mucho, _________ es admirable.
3. Esta es la amiga con _________ voy
6. _________ necesitamos es más tiempo.
al cine.
Repaso número cuatro
A. Exprese en plural todos los elementos de las siguientes oraciones. Después,
tradúzcalas al inglés.
1. Este niño, que nunca hace lo que le dicen, tampoco sabe nunca lo que quiere.
2. Esta legislación sobre la que hablo ahora no va a afectar a ningún ciudadano.
3. Ese pintor mexicano, cuyo mural muestra la situación del indígena, va a presentar su
obra hoy.
37
4. Esa universidad madrileña, que es famosa por su tradición académica, va a ofrecer
una beca.
5. No voy a protestar contra una opinión con la cual estoy de acuerdo.
6. Todo lo que pido es un trabajo en el cual expresar mi talento artístico.
B. ¿Qué personas, regiones o cosas curiosas conoce Ud.? Describa varias de ellas
usando pronombres relativos. Por ejemplo: Conozco una ciudad cuyas calles son de
piedra. Conozco a una mujer que solamente duerme tres horas diarias. Etc.
C. Escriba en español:
1. Do you have any questions about what I'm saying? I have no questions at this point.
2. Some people are afraid of what they do not understand.
3. This is another problem that I prefer to postpone for next week.
4. No one ever understands these young men who prefer to listen to noisy music.
5. Does she know Madrid? No, she knows no Spaniards either. But she does know how to
dance.
6. Neither you nor anybody understands what I am going to do.
D. Complete el siguiente párrafo con los pronombres relativos, las terminaciones y
los artículos adecuados. Escriba X en los espacios que no necesitan nada. Después,
escriba tres preguntas sobre el tema y respóndalas.
Sandra Cisneros (Chicago, 1945) es ___ poeta, Sandra Cisneros is a poet, novelist and
novelista y cuentista. Est__ autor__ chican__ short story writer. This Chicana author
normally writes in English, but uses a
escribe normal______ en inglés, pero utiliz__ style that incorporates many phrases in
___ estilo _____ incorpora much___ frases en Spanish. Several of Cisneros' poems
stories are well known for the
___ español. Vari__ de ___ poemas y cuentos and
humor and strength with which they
de Cisneros son bien conocid___ por ___
represent the experiences of Mexicans
humor y ___ fuerza con ______ represent____ on both sides of the border. The novel
The House on Mango Street, which
_____ experiencias de ____ mexicanos en
received the American Book Award in
amb___ lados de ___ frontera. ____ novela The 1985, is another example of her literary
Cisneros, whose readers
House on Mango Street, ________ recibió ____ successes.
expect with great enthusiasm her future
premio de__ "American Book" en 1985, es ___ works, currently resides in the city of
otro ejemplo de sus éxitos literari___. Cisneros, San Antonio, Texas.
38
_______ lector___ esperan con ___ gran
entusiasmo sus próxim___ obras, resid__
actual______ en ___ ciudad de San Antonio, en
Texas.
24. Possessives / Los posesivos
Short Form: Adjectives
Singular
Plural
mi
mis
tu
tus
su
sus
nuestro(a)
nuestros(as)
vuestro(a)
vuestros(as)
su
sus
Long Form: Adjectives and Pronouns
Singular
Plural
mío(a)
míos(as)
tuyo(a)
tuyos(as)
suyo(a)
suyos(as)
nuestro(a)
nuestros(as)
vuestro(a)
vuestros(as)
suyo(a)
suyos(as)
Possessives agree with the noun they qualify in gender and number:
Your family is from Perú; mine is from
Tu familia es de Perú; la mía, de Chile.
Chile.
Nuestros padres conocen a las vecinas
Our parents know your neighbor ladies.
tuyas.
Short forms are used more frequently and precede the noun. Long forms, used for
emphasis or contrast, follow the noun which is preceded by the article:
Dicen que el amor es solo un sueño
They say love is only a dream of ours.
nuestro.
The forms su, sus, suyo(a), suyos(as) have multiple meanings. This means that one
cannot distinguish except by context between his book, hers or theirs. To clarify the
meaning, it is necessary to use phrases such as el libro de él, de ellas, de usted, etc.:
La familia de él, como la de ustedes, vive
His family, like yours, lives in Rome.
en Roma.
Possessive pronouns replace a possessive adjective + noun: mi casa --> la mía. They use
the long forms, and are generally accompanied by the definite article:
My family lives in Madrid. And yours?
Mi familia vive en Madrid. ¿Y la tuya?
Mine lives near Toledo.
La mía vive cerca de Toledo.
The article can be omitted after the verb ser: Esta pintura es mía. • Esta casa no es
nuestra.
PRÁCTICA 24A. Llene los espacios con el adjetivo posesivo lógico, y termine cada
oración con el verbo y el pronombre posesivo adecuado. Ejemplo:
Quiero a _mi_ familia como casi todos nosotros... queremos a la nuestra.
39
1. Cerramos ________ oficina y tú
2. Nadie puede ver ____ futuro, pero yo
también...
sí...
3. Juegas con ______ hijos igual que
4. Él cuenta ____ historias, y vosotros
nosotros...
también...
5. Siempre hago ____ trabajo, en cambio tú6. Resolvemos ____________ problemas
nunca...
así como todo el mundo...
PRÁCTICA 24B. Exprese en español, eliminando la ambigüedad:
1. I look at her photos.
2. Do you need his pencils?
3. We do not understand their orders (the ladies‘).
4. They want your short story (formal, singular).
5. He plays with your (pl.) friends.
6. Where is his motorcycle?
7. I never see her husband.
25. Uses of ser and estar
The verb estar, associated with the words state and station, is used:
A. to express location (ubicación):
El profesor no está aquí. Ahí está el
problema. Panamá está en
Centroamérica.
BUT: Use ser to describe where an event is taking place: El concierto es aquí.
B. to describe states and conditions or a
Están tristes porque su perro está
change in a characteristic:
muerto.
Estamos interesados en el tema.
El país está en una situación difícil.
C. with a gerund to express a continuing
Estamos comiendo. Van a estar
action:
durmiendo.
D. with the past participle, which must agree in number and gender, to express a
resultant state:
Las ventanas están cerradas.
The windows are closed.
The watch is broken.
El reloj está roto.
For the formation of past participles, see §26.
The verb ser, from the Latin essere associated with the word essence, is used:
A. to identify, describe or define a subject: El hermano es ingeniero. Es a ella a
quien busco. Esto es un problema.
40
B. with de to denote origin, material, or
Es de madera. Es del Canadá. Es de
ownership:
Juan.
C. with adjectives that denote inherent, characteristic qualities, including (somewhat
surprisingly): pobre, rico, joven, viejo.
D. in expressions of time:
Son las tres y media. Hoy es jueves.
E. in impersonal expressions:
Es posible saber eso, es importante...
Note: For the use of ser with past participle (passive voice), see §41.
Some adjectives have quite different meanings when used with the two verbs, for
example:
estar rico (with food and drink), to be
ser rico implies the literal meaning, rich
delicious
Mi tío no es rico.
Esta paella está muy rica
estar listo/a to be ready
ser lista/o
to be bright, quick, intelligent
María es muy lista.
¿Estás lista, María?
estar aburrido to be bored
ser aburrido
to be boring or dull
Estoy aburrido con esta novela.
Esta novela es muy aburrida.
PRÁCTICA 25. Complete las oraciones con la forma apropiada de ser o estar:
1. Mi amigo
en Chile ahora.
estudiante. No _____ muy rico.
2. Esa puerta
verde, pero _ rota. Ahora _ abierta.
3. Todos los candidatos
hombres.
perdiendo (losing).
__ tristes.
4. ¿Qué
esto?
mi reloj. ¿
suizo? No,
de Texas.
5. ¿Dónde
Portland? ¿Cuál
la ciudad más grande de Maine?
6. Mi amiga
rubia. Tu amiga
morena, pero hoy
muy pálida.
7. Las ventanas aquí
muy grandes. Todas
abiertas.
8. Estos ejercicios
muy aburridos, pero ya
hechos.
9.
importante estudiar mucho. Yo
estudiando ahora. _____ muy listo/a.
10. Nosotros no ____________ listos para decidir porque __________ en problemas.
11. Este programa _____ para niños. ______ (yo) viendo el canal equivocado.
26. Past participle / El participio
To form the past participle (participio pasivo) of most verbs, change the infinitive
ending:
from -ar to -ado:
from -er or -ir to -ido
terminar > terminado
comer > comido ; vivir > vivido
-er and -ir verbs whose stems end in a strong vowel (a ,e ,o) add ído:
oír : oído • creer : creído • leer : leído • caer : caído • traer : traído •
reír : reído
BUT: construir : construido (because the stem ends in a weak vowel, u).
The following irregular forms exist:
abrir : abierto
poner : puesto ; suponer : supuesto
cubrir : cubierto
resolver : resuelto
41
decir : dicho ; predecir :
romper : roto
satisfacer : satisfecho
predicho
escribir : escrito ; describir:
ver : visto
volver : vuelto ; devolver : devuelto
descrito
hacer : hecho
morir : muerto
together with verbs derived from them: descubrir: descubierto, componer: compuesto,
absolver: absuelto, inscribir: inscrito, etc.
The past participle (such as English known, defeated, written, etc) is used:
with haber to form the perfect tenses. The participle is invariable after haber. (See
•
§53)
• wtih ser to form the passive voice (See §41)
• as an adjective: Voy a ver unos programas hechos en España.
with estar to describe a condition or state that has resulted from a previous action or
•
event:
The window is broken.
La ventana está rota.
Todavía no estamos preparados.
We're not ready yet.
Para 1492, la unidad española ya estaba consolidada.
By 1492, the unity of Spain was already consolidated.
PRÁCTICA 26A. Sustituya los verbos entre paréntesis usando la forma correcta del
participio:
1. La mesa está (cubrir) de platos. Las cartas están (escribir)
2. Las plantas están (poner) en el suelo. El libro está (abrir).
3. La puerta está (cerrar), pero las ventanas están (abrir).
4. Vamos a leer un cuento (escribir) en Japón. Es sobre una leyenda (olvidar).
PRÁCTICA 26B. Replique a las afirmaciones utilizando el pronombre posesivo, y estar
+ participio, como en el modelo.
MODELO: Voy a escribir mi carta esta noche. : Oh, la mía ya está escrita.
1. Voy a hacer la tarea de español mañana.
42
2. No puedo decidir cuál va a ser el título de mi ensayo.
3. Voy a devolver mis libros a la biblioteca más tarde.
4. No sé cuándo voy a elegir mi especialización.
5. Quiero resolver mis dudas lo antes posible.
6. Necesito pagar mis cuentas.
27. Gerund and infinitive / El gerundio y el infinitivo
To form the gerund of most verbs, change the infinitive ending:
from -ar to -ando:
from -er or -ir to -iendo:
hablar > hablando
comer > comiendo • vivir > viviendo
-ir stem-changing verbs are affected (see appendix A): pedir: pidiendo • dormir:
durmiendo
-ir and -er verbs with stems ending in a vowel add -yendo:
traer: trayendo • oír: oyendo • creer: creyendo • destruir: destruyendo
A. The gerund (-ing form in English) does not change in gender or number. It is used:
1. with the appropriate form of estar (or hay) to form the progressive tenses:
Lisa está estudiando. Estamos aprendiendo. No hay nadie hablando.
Notice that in Spanish this construction cannot be used to express the future, as it
frequently is in English; e.g. "I am leaving next week.‖
The simple present can be used in this sense: ―Salgo la próxima semana‖.
2. with the verbs seguir and continuar to mean "go on doing something" (Unlike
English, Spanish does not offer the option of using an infinitive after seguir or
continuar):
Continuó diciendo. He went on
Siguió cantando. She continued to sing
saying.
3. in connection with a main verb, with no preposition, to express "by doing" or "while
doing" something:
Llegó a la universidad estudiando
He got to the university by studying a lot.
mucho.
43
I fall asleep while counting sheep.
Me duermo contando ovejas.
Spanish does not use the gerund as an adjective. Clauses or different expressions have to
be used:
the answering machine
la máquina contestadora (el contestador)
un proceso que nunca termina
a never-ending process
(interminable)
los pasajeros que llevan una maleta
passengers carrying a big case...
grande...
B. The infinitive is the verb form that Spanish uses as a noun:
1. as the subject of a sentence or the object of a verb:
Seeing is believing.
Ver es creer.
I don't like studying.
No me gusta estudiar.
Decidir casi siempre es difícil.
Deciding is difficult most of the time.
Many verbs require no preposition before the infinitive: Quiero dormir. Some,
however, require different prepositions: Sueño con viajar; Ayudan a mejorar;
Tratamos de entender. (See Appendix D).
2. immediately after any preposition:
Voy a salir después de comer.
Es su manera de hablar.
Habla sin usar las manos.
I am going to leave after eating.
It's his/her way of speaking.
He speaks without using his hands.
Note that, unlike English, Spanish does not offer the option of using the gerund after
prepositions or as the subject of a sentence. The infinitive must be used in such cases.
Two useful idiomatic constructions involving the inifinitive are:
Al + infinitive indicates two actions occurring at the same time, as English at, on, in,
•
upon doing:
Be careful in crossing.
Cuidado al cruzar
Al entrar, cierra la puerta.
Close the door upon entering.
• Acabar de + infinitive is used to express having just done something:
Acabo de resolver el problema.
I have just solved the problem.
Acaban de salir.
They have just left.
PRÁCTICA 27A. Complete las oraciones con el artículo apropiado. Después, indique el
uso del gerundio (A) o del infinitivo (B) al que corresponde cada oración.
Ejemplo: Se reúnen para analizar _la_ situación. _B2_ (usar el infinitivo después de
preposición).
1. ¿Está mejorando ____ institución educativa? ______
2. Un grupo de expertos acaba de iniciar un congreso sobre ____ enseñanza. _____
44
3. Están reunidos para encontrar soluciones a ____ temas más urgentes. ______
4. Cambiar ____ sistema no es fácil, pero es posible. ______
5. Muchos profesores introducen ____ temas haciendo preguntas. ______
6. Hay instituciones que prefieren dar mucha libertad a ____ maestros. ______
7. ____ gobierno continúa estimulando iniciativas innovadoras. _______
8. Ningún cambio va a ser efectivo sin conocer bien a ____ juventud. _______
PRÁCTICA 27B. Use el infinitivo o el gerundio para completar las siguientes oraciones:
1. Estoy (writing) una carta para (expressing) lo que quiero (doing).
2. No me gusta (work). (Play) la guitarra es preferible.
3. Hago mi trabajo (watching) televisión, porque así es más fácil (thinking).
4. (Drinking) es mejor que (eating).
5. Es difícil (learn) esto, pero continúo (study).
6. Antes de (going) a Boston, estamos (preparing) las cosas.
7. Acabamos de (call) a la policía porque hay unos vecinos (make) mucho ruido.
8. Es posible (progressing) mucho (by doing) los ejercicios.
9. Siempre es buena idea (saying hello) a la profesora (upon entering) en la clase.
10. Es necesario (analize) el tema antes de (take) cualquier decisión.
11. Al (arriving) a casa, siempre me gusta (listening) música para (resting).
Repaso número cinco
A. Llene los espacios con las terminaciones necesarias, y elija las palabras que mejor
completan cada oración.
Ejemplo: El gobierno sigue (a buscar/buscando) alguna_ solución.
1. El congreso (es/está) (contemplar/contemplando) diferent___ perspectivas, porque
(resolver/resolviendo) est__ problema no (es/está) fácil.
45
2. L___ congresistas necesitan (tener/ser) mucho cuidado al (tomar/tomando)
(cualquier/cualquiera) decisión, porque esto va a (afectar/afectando) a much__ gente.
3. Vari___ sectores (son/están) (formar/formando) grupos de presión para
(demandar/demandando) lo que dese___.
4. Otros sectores están escéptic___, y no (querer/quieren) hacer (nada/algo) antes de
(tener/teniendo) pruebas tangibl___.
5. Por todas est___ razones, continúan (a discutir/discutiendo) este asunto, y nadie pued___
concluir (nada/algo).
6. (Ver/Viendo) tanta confusión, muchos creen que va a (ser/estar) imposible (llegar/llegando) a
un acuerdo adecuad___. 7. Sin embargo, el presidente puede (terminar/terminando) l__
discusiones en (cualquier/ningún) momento, (declarar/declarando) un estado de emergencia.
B. Escriba en español:
1. I never read either his messages or hers. But they don't understand that.
2. They are talking to several friends of mine who want some money.
3. I am going to leave after eating, but I am coming back (volver) at 8:45 pm.
4. He speaks without knowing what he is saying.
5. By practicing frequently, we are going to improve our writing.
6. I like being from Panama a lot. I know my country well.
7. The clock is still new but it is broken already. I can‘t believe what I‘m hearing.
8. The article I have just read is boring. I'm not going to continue to read.
C. Produzca oraciones como en el modelo.
Modelo: Sr. Vélez/España/vivir/Perú: El señor Vélez es español, pero ahora está
viviendo en Perú.
1. Dra. López/Venezuela/trabajar/Uruguay.
2. Sra. Martínez/Perú/construir escuelas/Haití.
3. General Gutiérrez/Nicaragua/pedir asilo político/Costa Rica.
4. Activista Rigoberta Menchú/Guatemala/asistir a un congreso/Holanda.
5. Periodista Elena Poniatoska/México/oír testimonios/La Florida.
46
Práctica escrita
A. Escriba en español:
1. She is an architect. She is from Madrid and she is a good friend of mine. Her father is
in Brazil now, studying carnivals, but he is going back to Spain next week.
2. I am a very good lawyer, but today I am sad. Our secretary is very good but today he
is not well and is not here. I think I am not ready for (para) this.
3. The doors are closed and I am at home watching TV. It's Monday; it's late and it's
boring not to have any money. Drinking too much is not very intelligent.
4. Working after eating is bad. Reading all the time instead of (en vez de) going out is
not healthy either. My homework, however, is done and that is good.
5. They are writing letters. Two are written already. These are good letters but my
friends are very tired of thinking and of searching for words.
B. Primero, lea la versión en inglés. Después, complete el párrafo con las palabras y
terminaciones necesarias. Escriba X en los espacios que no necesitan nada.
Finalmente, escriba dos preguntas sobre el texto y respóndalas.
Nacid__ en La Mancha en 1951, ___ cineasta
__spañol Pedro Almodóvar ____ sin ___ duda
___ pionero de ___ innovación cinematográfica
en ___ _______. ____ películas ____ ___
mismo tiempo comedias y tragedias, reflej_____
___ sociedad español__ en tod__ ____
complejidad. A ____ edad de _________ años
decide __________ a Madrid para
____________ un futuro más interesante. En
47
Born in La Mancha in 1951,
Spanish movie maker Pedro
Almodóvar is, without a doubt, a
pioneer of cinematographic
innovation in his country. His
movies are at the same time
comedies and tragedies, reflecting
Spanish society in all its
complexity. At the age of sixteen,
he decides coming to Madrid to
build a more interesting future. In
1979 he produces his first film,
1979 ____ produce ____ primer__ película,
titula___ Pepi, Luci y otr___ chicas del montón.
Pero ___ en 1988 cuando alcanza fama
internacional con ___ otra película, Mujeres al
borde de ___ _taque de nervios, ______ fue
nomin____ para ____ "Óscar". Almodóvar
continúa _____________ un cine intrig____ __
innovador__ _____ fascina ___ millones de
espectadores de todo ___ mundo.
titled Pepi, Luci, and Other
Ordinary Girls. But it is in 1988
when he reaches international
fame with another movie, Women
on the Verge of a Nervous Attack,
which was nominated for an
Oscar. Almodóvar continues to
produce an intriguing and
innovative cinema that fascinates
millions of spectators all over the
world.
28. The Preterite Tense / El tiempo pretérito
A. REGULAR AND IRREGULAR PRETERITE FORMS
1. Regular:
ayudar
--> ayudé, ayudaste, ayudó, ayudamos, ayudasteis, ayudaron.
atender --> atendí, atendiste, atendió, atendimos, atendisteis, atendieron.
asistir
--> asistí, asististe, asistió, asistimos, asististeis, asistieron.
2. Most common irregular preterites:
dar
di
diste
dio
dimos
disteis
dieron
decir
dije
dijiste
dijo
dijimos
dijisteis
dijeron
estar
estuve
estuviste
estuvo
estuvimos
estuvisteis
estuvieron
hacer
hice
hiciste
hizo
hicimos
hicisteis
hicieron
poder
pude
pudiste
pudo
pudimos
pudisteis
pudieron
poner
puse
pusiste
puso
pusimos
pusisteis
pusieron
querer
quise
quisiste
quiso
quisimos
quisisteis
quisieron
saber
supe
supiste
supo
supimos
supisteis
supieron
ser and ir
fui
tener
tuve
traer
traje
venir
vine
48
fuiste
fue
fuimos
fuisteis
fueron
tuviste
tuvo
tuvimos
tuvisteis
tuvieron
trajiste
trajo
trajimos
trajisteis
trajeron
viniste
vino
vinimos
vinisteis
vinieron
3. Observations:
• The 3rd plural ending of decir and traer is -eron and not -ieron.
• Ver is regular, but its stem is only the letter v: vi, viste, vio, vimos, vieron.
• Verbs derived from irregular verbs are like them in every way. For example, the
preterite of obtener is obtuve. Other common examples: distraer, distraje;
satisfacer, satisfice; predecir, predije; proponer, propuse; sostener, sostuve;
intervenir, intervine.
• The 3rd singular and plural of -er and -ir verbs whose stems end in a vowel, will end
in -yó and -yeron: creyó, creyeron; cayó, cayeron; leyó, leyeron; destruyó,
destruyeron; construyó, construyeron; oyó, oyeron; influyó, influyeron.
• The slightly irregular forms of the preterite of reír (and sonreír) are as follows:
reí, reíste, rio, reímos, reísteis, rieron
• All -ir stem-changing verbs change -o to -u or -e to -i in the third person singular and
plural of the preterite :
dormir: durmió, durmieron • repetir: repitió, repitieron • divertir: divirtió,
divirtieron
• Regular verbs ending in -car, -gar, -guar, and -zar have a spelling change in the first
person singular to maintain the pronunciation of the stem according to Spanish
consistent system (see §1): buscar: busqué ; llegar: llegué ; averiguar: averigüé ;
empezar: empecé.
• Verbs ending in –ducir (conducir, deducir, inducir):
produje, produjiste, produjo, produjimos, produjisteis, produjeron
PRÁCTICA 28A. Utilice el verbo entre paréntesis para completar, en pretérito, las
preguntas con las formas de tú y las respuestas con las formas de yo.
Modelo: (vivir) ¿Cuánto tiempo _viviste_ en Madrid? (estar) _Estuve_ allá cinco meses.
1. (visitar) ¿Cuándo __________ Madrid? (llegar) __________ a la ciudad en agosto.
2. (ir) ¿Con quién ___________? (viajar) __________ con mis compañeros de
secundaria.
49
3. (ver) ¿Qué museos ___________? (visitar) ____________ El Prado y el Reina Sofía.
4. (comenzar) ¿Qué día ___________ las clases? (empezar) _____________ el lunes
quince.
5. (hacer) ¿___________ muchos amigos? Sí, (conocer) __________ a varios españoles.
6. (disfrutar) ¿__________ de la vida nocturna? Sí, (estar) ___________ en varias
discotecas.
7. (poder) ¿____________ mejorar tu español? Claro, para eso (ir) _________.
8. (traer) ¿Qué ___________ del viaje? (sacar) _________ muchas fotos, (comprar)
__________ bastantes regalos y me (divertir) __________ muchísimo.
PRÁCTICA 28B. Ahora narre en tercera persona los eventos descritos en la práctica
anterior.
Ejemplo: Este joven estuvo cinco meses en Madrid.
B. USES OF THE PRETERITE / USOS DEL PRETÉRITO
1. to express non-habitual events occurring once or several times in the past ("punctual"
events):
I came in and sat down.
Entré y me senté.
50
He assisted me several times.
Me ayudó varias veces.
Did you decide to attend the lecture?
¿Decidiste asistir a la conferencia?
2. to express actions that have clear beginnings or ends (framed within a finite time
period, seen as complete):
He lived there since 1958.
Vivió allí desde 1958.
Julius Caesar lived for 57 years.
Julio César vivió 57 años.
Bebieron hasta las cuatro de la mañana. They drank until four a.m.
PRÁCTICA 28C. Cambie al pretérito la siguiente narración en primera persona.
Observe los eventos "puntuales" o "completos" que describe cada oración.
No hago la tarea porque tengo que ver al médico. Tampoco puedo hacer la siesta, por la
misma razón. El médico me dice la causa de mis problemas, y me da una prescripción.
Entonces voy a la farmacia, donde estoy más de media hora esperando. Finalmente,
tengo que irme sin comprar nada, porque los farmaceutas no pueden encontrar la
medicina. Después, vengo a casa y quiero hacer la tarea, pero me distraigo viendo la
televisión un rato. No sé en qué momento me duermo, lo cierto es que dos horas después
abro los ojos y veo una mariposa amarilla volando a mi alrededor. Sonrío, apago el
televisor, pongo la radio, y definitivamente no quiero hacer la tarea. Por fin comienzo a
escribir cartas escuchando música y no sé qué pasa después.
Observe how the one-time (perfective) aspect of the preterite affects the meanings of
certain verbs. Tuve que, for example, means both that I had to do something and did it.
No pude hacer la siesta means I couldn't and in fact didn't take a nap. Quise means that I
wanted-to-at-one-moment and has very nearly the sense of English "I tried to do it". No
quise is very close to English "I refused to do it".
29. Números de 100 en adelante
cien
ciento un dálmatas
ciento setenta y cinco toneladas
doscientos
doscientas noventa y ocho peras
trescientos
cuatrocientos
quinientos
100
101 Dalmatians (masc.)
175 tons (fem.)
200
298 pears (fem.)
300
400
500
51
quinientas treinta una libras
seiscientos
setecientos
setecientos uno
ochocientos
novecientos
1.000 mil
mil novecientos cuarenta y dos
1.000.000 un millón
mil millones de pesetas
un billón de dólares
531 pounds (fem.)
600
700
701
800
900
1,000
1942
1,000,000, noun
a billion pesetas
a thousand billion dollars
• Ciento (100) shortens to cien when it comes directly before a noun and with thousands
and millions, but is ciento when part of a number including tens or units:
100 pages
cien páginas
100,000 inhabitants
cien mil habitantes
107
ciento siete
99%
el noventa y nueve por ciento
The conjunction y only joins tens (decenas) and units (unidades).
• Only multiples of 100 and the number one agree in gender with the noun they qualify:
503 pages
quinientas tres páginas
301 women
trescientas una mujeres
trescientas un mil doscientas treinta y una letras
• Spanish does not use multiples of a hundred over nine, i.e. fifteen hundred: mil
quinientos, etc.
• Notice the period to mark thousands. The comma marks the decimal point: pi=3,1416.
• The preposition de is needed to connect nouns with the plural of mil and with the word
millón/millones:
dos mil personas — 2,000 people (do not use gente with numbers)
thousands of people (not gente)
miles de personas
one million women
un millón de mujeres
$13M
trece millones de dólares
un millón quinientos cuarenta mil novecientos once habitantes
PRÁCTICA 29. Conteste las preguntas escribiendo los números en palabras, o
pronunciándolos en voz alta.
1. ¿En qué año llegó a América Cristóbal Colón?
2. ¿Cuántas personas hay en tu ciudad?
3. ¿Cuántos estudiantes hay en tu universidad?
52
4. ¿En qué año fue la Independencia de los Estados Unidos?
5. ¿Cuántos habitantes tiene tu país?
30. Dates, Seasons, Weather / Fechas, estaciones, clima
Los días:
Los meses:
Las estaciones:
el lunes
la primavera spring
el martes
enero
julio
el verano
summer
el
febrero
agosto
autumn, fall
miércoles
marzo
septiembre el otoño
el invierno
winter
el jueves
abril
octubre
el viernes
mayo
noviembre
el sábado
junio
diciembre
el domingo
last week
la semana pasada
next Sunday
el domingo que viene
I'm going to Boston on Friday.
Voy a Boston el viernes.
I don't work on Saturdays.
No trabajo los sábados.
on Tuesday morning
el martes por la mañana
Tenemos vacaciones en enero. We have a vacation in January.
Vamos a México el junio que
We're going to Mexico next June.
viene.
Fuimos a Madrid el verano
We went to Madrid last summer.
pasado.
• Remember that Spanish does not use capitals for days of the week, months or seasons.
• Referring to days, note the use of the definite article in the singular to express "on".
But: Hoy es domingo, mañana es lunes. Spanish never uses en with days of the
week.
• The definite article is not needed when referring to seasons in a generic way:
El invierno es frío en Maine. No trabajo en (el) verano.
En Chile es verano cuando en Canadá es invierno.
Es un bello día de otoño. Aquí no hay primavera.
Dates / Las fechas:
el primero de octubre de 1894 : 1/10/94
Nací el seis de enero de 1986.
Mi cumpleaños es el veintinueve de mayo.
¿Cuál es la fecha? Hoy es (el) quince de agosto.
La Habana, 15 de abril de 1898.
• After the first of the month, Spanish uses cardinal (regular) numbers for dates. The
definite article is needed except to state the current date, or when the date alone is
given, such as in a letter or in a school exercise.
53
Climate and weather / El clima y el tiempo:
• Some weather conditions are expressed with hace:
What's the weather like today?
¿Qué tiempo hace hoy?
It's sunny today.
Hoy hace sol.
It was windy yesterday.
Ayer hizo viento.
Hace buen tiempo.
The weather is good.
La semana pasada hizo mal tiempo. The weather was bad last week.
Hace mucho calor en verano.
It's very hot.
Está haciendo demasiado frío.
It's too cold.
Hizo fresco el miércoles.
It was cool on Wednesday.
Note that frío, calor, viento, etc. are nouns. "Very" will have to be expressed with the
adjective mucho/ mucha: Hizo mucho calor. Fue un día muy caliente (caluroso).
• Other weather expressions:
it rains, is raining
llueve, está lloviendo
it snows, is snowing
nieva, está nevando
It's cloudy.
Está nublado.
¿Cuál es el pronóstico del tiempo? What's the weather forecast?
Se espera lluvia mañana.
Rain is expected tomorrow.
Tenemos nieve esta semana.
We're having snow this week.
Tuvimos un día soleado el viernes. We had a sunny day on Friday.
PRÁCTICA 30. Responda detalladamente las siguientes preguntas:
1. ¿Qué días y a qué horas tienes clases de español este semestre?
2. ¿En qué fecha es tu cumpleaños? ¿Y el de tu mejor amigo/a?
3. ¿En qué fecha llegó Cristóbal Colón a América?
4. ¿Cuál es una fecha memorable para ti? ¿Qué pasó ese día?
5. ¿Hay estaciones en el lugar donde vive tu familia? ¿Qué tiempo hace?
6. ¿Qué tiempo hizo este fin de semana: el sábado y el domingo?
7. Inventa el pronóstico del tiempo para los dos días del fin de semana que viene.
8. ¿Qué haces normalmente los viernes por la noche? ¿Qué hiciste el viernes pasado?
54
31. Both, All, Every / Ambos, todos, cada
• Todo/a/s and ambos/as must agree in gender and number with the noun they modify
or replace, and are never followed by the preposition de.
Va a llover toda la primavera.
It's going to rain all (the whole) spring.
Todos los problemas son fáciles.
All (of the) problems are easy.
Todos cometemos errores.
All of us (we all) make mistakes.
Ambos días hizo sol.
It was sunny on both (of the) days.
Ambas fueron semanas de lluvia.
They were both rainy weeks.
• Notice that ambos is never accompanied by articles: ambas ciudades both of the
cities
• A common equivalent of ambos/as is los/las dos:
Ambos (Los dos) conocemos Panamá y Caracas, y las dos (ambas) son
calientes.
Both of us know Panama and Caracas, and both of them are hot.
• Subject pronouns can be used after todos (not after ambos or los dos):
Todas ellas saben nadar.
All of them know how to swim.
• Todos often translates to every in the sense of "all the":
every day
todos los días
all of the classes
todas las clases
I know everything (that) you did on
Sé todo lo que hiciste el domingo.
Sunday.
Hay nieve por todas partes.
There is snow everywhere.
• When stressing individuality (each), every translates into cada:
She knows every detail of the story.
Conoce cada detalle del cuento.
He called each one of them by their
Los llamó a cada uno por su nombre.
names.
―Every other‖ is expressed by cada dos: cada dos años, cada dos días.
• Both...and is tanto...como (no gender or number). Never use ambos in this context:
Tenemos clases tanto los lunes como los miércoles.
We have classes both on Mondays and on Wednesdays.
PRÁCTICA 31. Escriba en español:
1. All of the South American countries voted against (contra) the resolution last week.
2. Every representative expressed his/her own reasons, but all of them agreed (estar de
acuerdo).
3. Yesterday, however, both Argentina and Uruguay changed their votes.
4. Both of them decided to defend their mutual interests against the majority.
55
5. This was both surprising and irritating for all representatives.
6. Something similar occurs every year.
Repaso número seis
A. Utilice el pretérito para completar la siguiente narración. Después, observe qué
eventos "puntuales" y qué acciones "completas" describe cada oración.
A. Utilice el pretérito para completar la siguiente narración. Después, observe qué eventos "puntuales" y
qué acciones "completas" describe cada oración.
1. Ayer _______ (ir [yo]) a la biblioteca y _________ (decidir) leer un rato.
2. Entonces, _________ (buscar [yo]) un libro reservado para la clase de español, y _________
(continuar) mi lectura sobre la historia de España.
3. Según lo que __________ (leer [yo]) ayer, los musulmanes del norte de África _____________ (invadir)
Hispania en el año 711,
4. y en pocos años _____________ (conquistar) la mayor parte de la península.
5. Los visigodos cristianos ___________ (tener) que refugiarse en el norte, pero durante ocho siglos
____________ (mantener) la determinación de reconquistar el país.
6. Los musulmanes ____________ (poner) su centro administrativo en la ciudad de Córdoba, la cual se
___________ (convertir) en un centro intelectual de la cultura islámica.
7. Uno de los héroes de la reconquista cristiana _________ (ser) El Cid. Rodrigo Díaz de Vivar ________
(nacer) cerca de Burgos, y ___________ (morir) heroicamente en el año 1099.
8. Muchos trovadores medievales __________ (cantar) al heroísmo de este personaje, mostrando cómo un
ejército pequeño, en nombre de la hispanidad y del Dios cristiano, _________ (poder) vencer un imperio
con cientos de miles de guerreros.
9. Casi doscientos años después, en el siglo XII, _________ (aparecer) el "Cantar de Mío Cid", que
_________ (ser) uno de los primeros libros publicados en lengua española.
10. Esta lectura me _________ (parecer) interesante y me (dar) __________ mucho en
qué pensar.
B. Escriba en español:
1. Several friends of mine came to visit my family last week.
2. Did you have another chemistry exam? No, I had no exams on Monday.
3. Which car did you choose? I liked both of them, but I chose the red one.
4. Professor Vélez studied both tourism and immigration in modern Spain.
5. All of us went to Mexico on January 10, 1950 and I began to study Spanish a month
later.
56
6. I was reading all night, since it was impossible to abandon such a fascinating novel.
7. He tried to open the door but he couldn't. So, he decided to wait for me. But I couldn't
either.
Práctica escrita
A. Narre en el pretérito los sucesos (reales o imaginarios) sobre un viaje o una aventura,
incluyendo solamente eventos puntuales o completos (no describa circunstancias,
emociones, etc.). Utilice los siguientes elementos:
- varias personas (yo, nosotros, ella, etc.)
- un caso de "a personal"
- un caso del uso del infinitivo
- dos días de la semana, un mes, un año o una fecha, un número superior a cien.
- todos, ambos, lo que, por la mañana/tarde/noche
- una frase que no necesita el artículo indefinido en español (pero sí en inglés)
B. Primero, lea la versión en inglés. Después, complete el párrafo con las palabras y
letras necesarias. Finalmente, escriba dos preguntas sobre el texto y respóndalas.
El tango ____ producto y expresión de la ola de inmigración gaucha y _urope_ en ___ región del
Río de la Plata (____ incluye ________ a Buenos Aires _______ a Montevideo) desde fines del
siglo XIX hasta la década de 1920. _________ en ______ ciudades hacia 1880 y _________
fama interna_ional con ___ desarrollo de ___ radiodifusi_n en ____ años 20, particularmente
gracias ___ cantante Carlos Gardel (1890-1935), un famos_simo artist_ ______ llev_ las
canciones del tango a _uropa y a _______ Am_rica Latina. Curios_________, la danza hoy
asocia___ con ___ tango apareció separa___ del género m_sical mismo, y ____ incorpora___ a
él a comienzos del siglo ____. Del mism_ modo, la música surgi__ inicialmente sin letra. ____
instrumentación es variable, pudiendo incluir viol_n, flauta, guitarra, arpa y piano. Un
instrumento _aracter_stic_, que ____ introducid__ hacia 1890, es el bandoneón, una versión del
acordeón _lem_n particularmente dif_cil de interpretar. Como afirm_ el compositor _rgentino
Enrique Santos Discépolo, ___ tango “es un pensamiento triste que se baila”. En efecto, ___
letras represent___ ____________ experiencias de los proletarios urban___: pobreza, soledad,
violencia, machismo, y una actitud _________ reflexiva ________ desafiante frente a ___
adversidad.
Tango was a product and expression of the Gaucho and European immigration wave in
the Río de la Plata region (which includes both Buenos Aires and Montevideo), from the
end of the 19th century until the 1920s. It was born in both cities toward 1880 and it
gained international fame with the development of radio broadcasting in the twenties,
particularly thanks to singer Carlos Gardel (1890-1935), an extremely famous artist who
took Tango songs to Europe and all of Latin America. Curiously, the dance associated
with Tango today appeared separated from the musical genre itself, and it was
incorporated to the genre in the beginnings of the 20th century. In the same manner, the
music emerged initially with no words. Its instruments are variable, and they can include
violin, flute, guitar, harp and piano. A characteristic instrument, which was introduced
57
toward 1890, is the bandoneon, a version of German accordion that is particularly
difficult to play. As Argentine composer Enrique Santos Discépolo declared, "Tango is a
sad thought that is danced." Indeed, Tango words represent all of the urban proletariat
experiences: poverty, loneliness, violence, machismo, and an attitude toward adversity
that is both reflective and daring.
32. The Imperfect / El imperfecto
A. REGULAR AND IRREGULAR IMPERFECTS
1. Regular:
ayudar --> ayudaba , ayudabas, ayudaba, ayudábamos, ayudabais, ayudaban
atender --> atendía, atendías, atendía, atendíamos, atendíais, atendían
asistir --> asistía, asistías, asistía, asistíamos, asistíais, asistían
2. Only three verbs have irregular forms in the imperfect:
ir
iba
ibas
iba
íbamos
ibais
iban
ser era
eras
era
éramos
erais
eran
ver veía
veías
veía
veíamos
veíais
veían
B. USES OF THE IMPERFECT / USOS DEL IMPERFECTO
1. to relate descriptions in the past: the background or setting, situations, conditions, and
actions that were in progress (was/were ...ing):
Eran las seis. Tenía doce años. Estaba It was six. I was twelve. I was sick and was
enfermo y leía en cama. Afuera llovía. reading in bed. It was raining outside.
Suddenly...
De pronto...
2. to relate habitual, customary actions, or things that used to happen in the past without
specific reference to a beginning or end (imperfective, incomplete) (used to, would):
They used to go to the movies every
Iban al cine todos los sábados.
Saturday.
I would to go the park a lot when I was
Iba mucho al parque cuando era joven.
young.
Estaba informada porque leía mucho. She was informed because she read a lot.
This means that time expressions conveying repetition are often clues for selecting the
imperfect: con frecuencia, todos los días, cada primavera, generalmente, siempre, a
veces, etc.
Tip: Whenever you could say used to or was/were...ing in English, you need the
58
imperfect in Spanish.
PRÁCTICA 32A. Cambie al pasado la siguiente descripción. Solamente los dos últimos verbos deben estar
en pretérito (son los únicos que reportan una acción "puntual"). Explique por qué todas las otras
oraciones deben estar en imperfecto.
Es ____________ el martes doce de noviembre. Son____________ las seis de la mañana. Poca
gente camina____________ por las calles. Algunos peatones van____________ hacia el trabajo,
todavía medio dormidos. Hace frío, pero no llueve____________ ni nieva____________. Hay
algunos autos, pero el tráfico no es____________ pesado. Ya estamos____________ en
invierno. Desde mi ventana veo____________ pasar a la gente. Generalmente, cuando
tomo____________ el café matutino, estoy ____________ sereno/a, contento/a, alerta. Nada me
molesta ____________. Hay ____________ magia en el aire fresco. Tanto la gente como la
naturaleza parecen____________ un poco perezosas por las mañanas. El cielo está
____________ gris, pero no siento____________ melancolía. En ese momento, me
siento____________ inspirado/a y escribo____________ un poema.
PRÁCTICA 32B. ¿Cómo era tu vida antes de venir a la preparatoria?
¿Dónde y con quién vivías?
¿Dónde estudiabas?
¿Cómo era tu escuela?
¿Qué hacías todos los días?
¿Quiénes y cómo eran tus amigos?
¿Qué hacías los fines de semana?
¿Con qué soñabas?
¿Qué deseabas?
33. Imperfect and Preterite Contrasted
The preterite is a "perfect" tense because it reports events viewed as completed within a
finite time frame. An "imperfect" tense conveys duration, progression, incompleteness:
• Los indígenas no aceptaban a los europeos. Algunos nunca los aceptaron.
The natives would not accept Europeans (for some time). Some (definitely) never did.
El sábado nevó todo el día. vs. El sábado nevaba y hacía frío. (as a general
•
description)
It snowed all day on Saturday (time frame) vs. It was snowing on Saturday, and it
was cold.
El año pasado asistí a muchos conciertos. vs. El año pasado asistía mucho a
•
conciertos.
Last year I attended many concerts. vs. Last year I used to attend concerts a lot
59
(often).
When narrating, each tense has a distinct function:
A. The imperfect gives background information and describes what was happening
(duration).
The preterite reports punctual actions and tells what happened (completion):
• When I woke up, it was raining. (Act of waking up [once], raining in progress)
Cuando me desperté, estaba
o: Cuando me desperté, llovía.
lloviendo.
Our friend was waiting for us when we arrived at the theater. (Act of arriving, waiting
•
in progress)
Nuestro amigo nos esperaba cuando llegamos al teatro.
He said (that) he was tired. (Act of reporting [complete] on an ongoing condition or
•
plan)
Dijo que estaba cansado.
No les dije que planeaba estudiar en Maine.
B. The imperfect describes a state or condition (duration).
The preterite reports a punctual change in condition (completion).
They didn't want to go but she insisted. (Their state of unwillingness vs. her act of
•
assertiveness)
No querían ir, pero ella insistió.
• I was going to leave earlier, but I decided to wait for you.
Iba a salir antes, pero decidí esperarte. (ongoing intention vs. a change of plans)
• I was feeling cheerful; but, while watching the news, I got scared.
Me sentía contento; pero, al ver las noticias, tuve miedo.
PRÁCTICA 33A. Complete con el pretérito o el imperfecto del verbo entre paréntesis.
1. El sábado, mientras _________ (leer [yo]) el periódico tranquilamente, ________ (sonar) el
teléfono. 2. _____ (ser) María, mi vecina. Me _______ (decir [ella]) que _________ (necesitar)
un favor. 3. Al escuchar su voz, __________ (sentir [yo]) impaciencia, porque __________
(querer) seguir leyendo sin interrupciones. 4. Pero ___________ (escuchar [yo]) con paciencia
su larga historia. 5. Cuando finalmente ___________ (colgar [yo]) el teléfono, ya ________
(ser) las diez de la mañana. 6. Entonces _________ (decidir [yo]) dar un paseo, porque
__________ (hacer) sol. 7. ________ (caminar) durante casi dos horas y, cuando _________
(volver) a casa, ya _____ (ser) hora del almuerzo. 8. Por eso no ____________ (terminar) el
artículo tan interesante que __________ (estar) leyendo cuando me __________ (llamar) María,
la vecina inoportuna.
C. As was explained in §28, a few verbs have slightly different meanings in the preterite.
All of them, however, follow the general idea of duration for the imperfect and of
completeness for the preterite:
Imperfect
Preterite
could, had the ability to
managed to (or failed to if negative)
poder
wanted, had the intention to
tried to (or refused if negative)
querer
conocer previously knew for some time met, got to know for the first time
60
saber
had knowledge for some time
learned, found out at a specific
point
See § 22 to review the difference between conocer and saber
Los conocía antes de viajar.
I knew them before traveling.
¿Qué sabía Bush del ataque?
What did Bush know of the
attack?
No sabía que eras chilena.
I didn´t know you were Chilean.
De niño podía jugar todo el día.
Los conocí al viajar.
I met them while traveling
¿Cuándo supo Bush del ataque?
When did Bush find out about the attack?
Ayer supe que eras de Chile.
Yesterday I learned you were from Chile.
También pude hacer muchos amigos.
I also had the opportunity to make many
As a child, I could play all day.
friends.
Como no podíamos llamarlo, ... no pudimos saber qué pensaba del asunto.
we couldn‘t find out what he thought of the
Since we couldn‘t call him,
matter.
Queríamos ir de compras, ...
pero mi madre no quiso darnos dinero.
but my mother refused to give us some
We wanted to go shopping,
money.
PRÁCTICA 33B. Subraye la forma más apropiada de cada verbo.
Antes yo no (1. sabía/supe) nada sobre Colombia, pero el diciembre pasado (2. viajaba/viajé) a
Bogotá y (3. aprendía/aprendí) mucho sobre el país. Durante mi visita, (4. conocía/conocí)
varios centros comerciales, discotecas, universidades y parques inmensos. También (5.
pude/podía) ver las obras originales de Fernando Botero, el famoso pintor de figuras
voluminosas. El último día, un guía turístico me (6. decía/dijo) que yo no (7. podía/pude) irme sin
ver el Museo del Oro, e inmediatamente (8. quería/quise) visitarlo. Afortunadamente, esa misma
tarde (9. podíamos/pudimos) ir al museo. Allí (10. había/hubo) impresionantes piezas de oro
elaboradas por la cultura Muisca, los indígenas de esa región. También (11. sabíamos/supimos)
que los muiscas nunca (12. querían/quisieron) revelar el secreto del Dorado -la legendaria
ciudad de oro- a los españoles, quienes no (13. podían/pudieron) encontrarla jamás.
Paradójicamente, los colombianos (14. daban/dieron) el nombre de "Eldorado" al aeropuerto
internacional de la capital. Gracias a este viaje, (15. sabía/supe) que Bogotá es una ciudad muy
moderna con una rica historia.
34. Hace with Time Expressions
A. Hace with the preterite tense is equivalent to English "ago":
I arrived long time ago.
Llegué hace mucho tiempo.
B. Hace/Hacía used with the present or the imperfect express
"have been/had been doing something for...":
We have been waiting for two hours.
Hace dos horas que esperamos.
We had been sleeping for two hours.
Hacía dos horas que dormíamos.
While these word orders are the most frequently used, you will see other usages, notably
61
Hace mucho tiempo que llegué as a variant of A.
PRÁCTICA 34.
¿Cuánto hace que vives en los Estados Unidos?
¿Cuánto hace que estudias español?
¿Cuánto tiempo hace que llegaste a esta preparatoria?
¿Cuánto hace que tuviste tus últimas vacaciones?
¿Cuántos años hace que llegó Colón a América?
¿Cuántas décadas hace que terminó la segunda guerra mundial (1945)?
¿Aproximadamente cuántos años hacía que los ingleses estaban en Norteamérica cuando los Estados
Unidos se independizaron?
¿Cuánto tiempo hacía que Kennedy era presidente cuando murió?
35. Ordinal Numbers / Los números ordinales
primero/a
segundo/a
tercero/a
cuarto/a
1o, 1a
2o, 2a
3o, 3a
4o, 4a
quinto/a
5o, 5a
sexto/a
6o, 6a
7o, 7a
8o, 8a
9o, 9a
10o, 10a
11o, 11a (*rarely
undécimo/a*
used)
12o, 12a (*rarely
duodécimo/a*
used)
séptimo/a
octavo/a
noveno/a
décimo/a
• Beyond 10th, cardinal numbers are preferred. Thus, the fifteenth floor would be el piso
quince.
• Primero and tercero shorten to primer and tercer before masculine singular nouns:
el primer capítulo, el tercer verso • la primera versión, la tercera calle
• Ordinal numbers are used less frequently in Spanish than in English. For example, as
stated before (§ 30), Spanish uses an ordinal number only for the first of the month,
and cardinal numbers after that: El primero de mayo, el dos de agosto.
• Titles of kings and Popes are written in Roman numerals and said in ordinal numbers:
(read Isabel cuarta, Juan Pablo segundo)
Isabel IV, Juan Pablo II
Notice that no article is used in this context.
• Century numbers are written in Roman numerals, but said in cardinal numbers after
the tenth century:
Desde el siglo XV hasta el
(read quince and diecinueve)
XIX
Desde el siglo VIII hasta el (read octavo and doce)
62
XII
PRÁCTICA 35. Escriba las oraciones, y exprese lo mismo utilizando "hace".
Ejemplo: China adquirió su nombre en el siglo III a.C. (siglo tercero antes de Cristo)
Hace veinticuatro siglos que China adquirió su nombre.
1. Juan Carlos I fue coronado rey de España en 1975.
2. Alfonso X, El Sabio, subió al trono de Castilla y León en el siglo XIII.
3. Thomas Jefferson se posesionó como 3er. presidente de la unión norteamericana en
1801.
4. Enrique VII subió al poder como 1er. rey de la dinastía Tudor en el siglo XV.
5. En 1978, Juan Pablo II se convirtió en el 1er. Papa no italiano desde 1522.
6. Julio César invadió a Bretaña en el siglo I a.C.
Repaso número siete
A. Utilice el pretérito o el imperfecto para completar la siguiente narración. Después,
observe los eventos "puntuales" o completos (pretérito) vs. los verbos "circunstanciales"
o en progreso (imperfecto).
1. Ayer, cuando _________ (llegar [yo]) a la biblioteca, todavía _______ (ser) temprano.
2. Entonces __________ (decidir [yo]) leer un rato, porque ________ (tener) mucho tiempo.
3. _________ (buscar [yo]) el libro que _____________ (estar) reservado para la clase de
español, y _________ (comenzar) a leer sobre El Cid.
4. Rodrigo Díaz de Vivar _________ (nacer) en el siglo XI, una época en la que los musulmanes
_________ (controlar) mucha parte de la península Ibérica.
5. Estos __________ (mantener) un régimen de tolerancia con los judíos y cristianos, pero los
visigodos _____________ (querer) recuperar su control de España.
6. El Cid ________ (ganar) fama por sus campañas contra los árabes, aunque también
________ (luchar) al servicio de los reyes moros de Zaragoza durante un breve periodo.
7. En el año 1081, el rey Alfonso VI _________ (expulsar) de su reino al Cid, porque _________
(tener) miedo y no __________ (querer) la competencia de un joven héroe.
8. Rodrigo __________ (morir) en el año 1099, mientras __________ (combatir) contra los
moros.
9. En el siglo XII, _________ (aparecer) el "Cantar de Mío Cid", el cual _________ (narrar) las
hazañas heroicas de este personaje.
B. Traduzca al español:
63
1. That morning I had to go to the city because I needed to buy some things. I left the
house early because I hoped to arrive before noon. I was wearing summer clothes when I
went out because it was hot, but in the afternoon it began to rain and from three o'clock
(on) it was cold all afternoon. When I arrived home, I was all wet. It was 7pm. I was
hungry and tired. When I was a child I used to love rain, but that afternoon it was
(definitely) not so nice.
2. I have been studying Spanish for about three years.
3. They had been waiting for an hour when I called them.
4. I arrived an hour ago, but other people continued to arrive later.
5. They were talking to another friend of mine who wanted some money.
6. Dr. Vélez was going to leave after eating, but he couldn't.
7. The clock was still new but it was broken already. I couldn‘t believe what I was
hearing.
8. Six decades ago, King George VI promised he was going to help the war victims.
Práctica escrita
A. Sustituya las palabras subrayadas por las que están entre paréntesis, cambiando
el tiempo verbal si es necesario.
1. Con frecuencia tenían razón. (solamente ayer)
2. Continué leyendo el martes. (hace una semana)
3. El lunes pasado llegaste a tiempo. (casi siempre)
4. Nunca cometía ningún error. (esta vez)
5. Mis padres veían a mi abuela regularmente. (el año pasado)
6. Lo que dijimos esta mañana no fue muy inteligente. (por lo general)
64
7. Fui filósofo varios años, pero cambié de profesión. (hace mucho tiempo)
B. Primero, lea la versión en inglés. Después, complete el párrafo con las palabras y
letras necesarias. Finalmente, escriba dos preguntas sobre el texto y respóndalas.
La civilización maya aparec___ ______ más de dos mil años, en las monta_as de Guatemala.
Con el tiempo, ocu___ tanto ___ península de Yucatán _______ algun_s partes de lo que hoy son
Belice y Honduras. ______ social como pol__________, los mayas _______ organizad____ en
extremo. También alcanza____ un impresionante desarrollo en las matem_ticas y ___
astronom_a. Se sabe, por ejemplo, que su complej__ sistema de numeración inclu___ el cero y
que crea____ un calendario más exacto que el que utilizaban ___ _uropeos. Además, t______
una ar__itectura muy sofisticad__ que expres____ un diálogo continuo con el cosmos a través de
palacios majestuos___. Para _____ mayas, ___ ar__itectura era un vehículo de comunicaci_n
social y religios__, y cada elemento t_____ u__ significación particular. Por eso constru_______
p____mides que manifest______ tant_ la jerar__ía espiritual de su cultura ______ ___
tra_ectoria cíclica de ____ estaciones y las estrellas. Hasta _____ poco tiempo, los antrop_logos
pensa______ que los mayas _______ un pueblo pacífico, dedicad_ a la ciencia, ___ arte y a la
agricultura. Pero vari__ descubrimientos recientes, ___ mejorar ___ interpretación de ___
escritura jeroglífic_, revela____ que las ciudad__ mayas t_______ guerras
___________________. Est_ podría ser una de l_s causas de su misterios_ desaparición.
The Mayan civilization appeared more than two thousand years ago, in the mountains of
Guatemala. In time, it occupied both the Yucatan peninsula and some parts of what is
Belize and Honduras today. Both socially and politically, the Mayans were extremely
organized. They also achieved an impressive development in mathematics and
astronomy. It is known, for example, that their complex number system included the
zero, and that they created a more exact calendar than the one Europeans used. They also
used to have a very sophisticated architecture that expressed an ongoing dialog with the
Cosmos through majestic palaces. To the Mayans, architecture was a vehicle for social
and religious communication, and each element had a particular significance. This is why
they built pyramids that manifested both their culture's spiritual hierarchy and the cyclic
trajectory of seasons and stars. Until not long ago, anthropologists used to think that the
Mayans were a peaceful people, completely devoted to science, art and agriculture. But
several recent discoveries, upon improving the interpretation of hieroglyphic writing,
revealed that Mayan cities had wars frequently. This could be one of the causes for their
mysterious disappearance.
36. Comparison / Las comparaciones
A. COMPARATIVES AND SUPERLATIVES
Spanish distinguishes between comparative and superlative by context: más means more
65
or most and menos means less or least. Thus el libro más largo may mean the longer
book (of two) or the longest (of a group). Other examples:
Esta casa era más bonita y era la más nueva del barrio.
This house used to be prettier and it was the newest (one) in the neighborhood.
Esta lectura es menos interesante, y es la más larga de todas.
This reading is less interesting, and it is the longest of (them) all.
Teníamos el problema más complicado del mundo.
We had the most complicated problem in the world.
Los poemas breves de Neruda muchas veces son los más interesantes.
Neruda's short poems are often the most interesting.
Note the use of de after superlatives (del barrio, del mundo), and the placement of the
adjective (complicado) after the noun (el problema).
PRÁCTICA 36A. Escriba en español:
1. The longer book is less boring.
2. The oldest city in the U.S.
3. The newest bicycle in the house.
4. The most serious dilemma in life.
5. A less complicated meal.
6. The richest country in the world.
7. The most beautiful images are often the least complicated.
8. This is the cheapest lamp in the store.
B. IRREGULAR COMPARATIVES: MEJOR, PEOR, MAYOR, MENOR
MEJOR better, best
PEOR worse, worst
Hablaste mejor que yo.
Son los peores sistemas que conozco.
MAYOR bigger, biggest
MENOR smaller,
smallest
Ese fue uno de nuestros mayores problemas.
La inflación es menor este año.
These irregular forms do not replace the regular, but rather convey slightly different
ideas:
• mejor/peor are used just like English better/worse to indicate a degree of excellence.
The less common más bueno/malo tend to convey moral qualities:
He was the kindest man in town.
Era el hombre más bueno del pueblo.
• mayor/menor indicate degree of importance, whereas más grande/pequeño are
66
preferred for physical size:
¿Cuál es tu mayor preocupación?
What's your biggest concern?
She asked for the largest ice cream of
Pidió el helado más grande de todos.
all.
• mayor/menor also mean older/younger, only referring to people (especially siblings):
Mi hermana mayor es más pequeña que
My older sister is shorter than I.
yo.
The youngest son had the oldest house.
El hijo menor tenía la casa más vieja.
PRÁCTICA 36B.
¿Cuál es tu mayor aspiración?
¿Tienes hermanos o hermanas menores que tú?
¿Y mayores?
¿Cuál es el peor equipo de béisbol de los Estados Unidos?
¿Cuál es el mejor deporte de tu preparatoria?
¿Cuál es el mayor defecto de tu mejor amigo/a?
C. MORE OR LESS
Más... que more... than
Menos... que less... than
to compare nouns: Tengo menos amigos que ella.
adjectives: Las máquinas son hoy más rápidas que nunca.
and adverbs: Ella habló más rápido que el profesor.
Trabajamos más que cualquiera de ellos.
Más... de
more... than
Menos... de less... than
more or less than a given quantity of something (numbers):
Tengo menos de cinco libros.
Recibía más de la mitad de los telegramas.
Contestó más del cincuenta por ciento.
The preposition de is also used when the comparison involves a clause (a phrase with a
new conjugated verb, usually introduced by el, la(s), lo(s) que):
You speak more than (what) you should.
Hablas más de lo que debes.
He brought more things than (the ones) he
Trajo más cosas de las que
needed.
necesitaba.
I spend less money than I earn.
Gasto menos dinero del que gano.
There are less people than I thought.
Hay menos gente de la que creí.
I had more fun than I expected.
Me divertí más de lo que esperaba.
PRÁCTICA 36C. Complete con que o con de según sea necesario.
1. Curiosamente, la grabadora barata era mejor ___ la cara. Y la barata valía menos ___ cien dólares.
67
2. Más ___ quinientas personas asistieron a la conferencia, muchas más ___ las que esperábamos.
3. Hace más ___ media hora que estoy sentado aquí, pero no tengo nada mejor ___ hacer.
4. Hubo más víctimas ___ las que informó la prensa y había menos periodistas ___ el año pasado.
5. Hacía menos frío el mes pasado ____ este. Pero el mes pasado cayó más nieve ___ nunca.
6. Este libro es más interesante ____ lo que pensé. Ya leí más ___ la mitad.
7. Mi hermana menor conoce a más gente ___ yo. Tiene más ___ doscientas amigas.
8. Más ___ la mitad de la población combinada gana menos ____ la minoría más poderosa.
D. AS...AS
Tan...como
as.....as
Tanto/a...como as much....as
Tantos/as...como as many...as
to equate adjectives or adverbs:
Es tan lista como siempre.
Come tan lentamente como su hermana.
alone or with a noun:
Yo no estudio tanto como él.
Los peruanos comen tanta carne como nosotros.
No teníamos tantos problemas como antes.
• Remember that tanto...como (with no feminine or plural) also means both...and (see §
31):
tanto las ciudades grandes como las
both large and small cities
pequeñas
• as...as possible has different variants in Spanish. A common equivalent is lo
más...posible:
El examen va a ser lo más fácil posible.
Llegué lo más pronto posible.
PRÁCTICA 36D. Complete con tan, tanto/a/s, como y de, según sea necesario.
1. Mi hermano mayor hizo _______ viajes en su juventud _______ yo ahora. Hizo más ___ diez.
2. El hijo menor de José no juega con _______ frecuencia ______ el mayor.
3. Ella habla _______ idiomas ______ su hermano menor. Saben más ___ cuatro lenguas.
4. Vivía _______ tranquila y plácidamente ______ cualquiera podría desear.
5. Me divertí _______ _______ esperaba. También bailé mucho más ___ lo que normalmente bailo.
6. _______ social _______ políticamente, es el país más estable ___ la región.
7. Nuestras manzanas son ______ sabrosas ______ las ___ tu país. Pero no producimos _______.
68
8. Las obras de Hemingway no son _______ sencillas _______ mucha gente imagina.
E. COMMON EXPRESSIONS USING MORE, MOST, LESS:
• The more...the more has different variants in Spanish. A common equivalent,
particularly in America, is Mientras más..más:
Mientras más estudio, más aprendo. The more I study, the more I learn.
Mientras menos gastas, mejor vives. The less you spend, the better you live.
• Most (of) is expressed by la mayor parte (de). An equivalent expression, for
countable nouns only, is la mayoría de. These expressions use the verb in the third
person singular:
La mayoría de nosotros conoce la mayor parte del país.
Most of us know most of the country.
¿Conoces a tus compañeros? Conozco a la mayor parte.
Do you know your classmates? I know most of them.
Less
and
less: cada vez menos
More and more: cada vez más
•
Cada vez mejor, cada vez peor, cada vez mayor, cada vez menor.
Viajar es cada vez más fácil, pero las distancias culturales son cada vez
mayores.
Traveling is easier and easier, but cultural distances are ever larger.
Más y más is mostly colloquial and less frequently used.
Even worse: todavía/aún peor
• Even more: todavía más / aún más
El servicio era agradable, y la comida todavía mejor.
The service was nice, the food even better.
PRÁCTICA 36E. Exprese en español.
1. I needed most of the reports (los informes) as soon as possible. The sooner, the better.
2. Life is more and more difficult in most of the country. It's even worse in rural areas.
3. I'm speaking better and better. The more I practice, the better I speak. I'm even better on Sunday
mornings, when I am inspired most of the time (el tiempo).
4. Most cars are more and more expensive every year. The more they cost, the less they last (durar).
PRÁCTICA 36F. Compare en inglés:
1. El clima de hoy con el de ayer.
2. Su niñez con la de sus padres.
69
3. Su país con un país hispánico.
4. La vida del siglo XX con la del siglo XV.
37. Pronouns After a Preposition
Pronombres después de preposición
para mí
sin ti
a usted
con él
por ella
contra
hacia
detrás de
sobre
entre
nosotros/as
vosotros/as
ustedes
ellos
ellas
• Note that, with the exception of mí and ti (no accent mark), these are identical to the
subject pronouns: Voy sin ti. ¡La carta es para nosotros! No quiso salir con ella.
• After a preposition, the English pronoun "it" can be expressed by eso when it refers to
an idea, and by él or ella when referring to a specific object whose gender is clear
(never use lo or la in this context):
Quería hablar con usted sobre eso. (about it)
Necesita su sombrero: no puede vivir sin él. (without it)
• The reflexive form is sí, referring back to the subject:
(among themselves)
Están luchando entre sí.
• Con followed by mí, ti or sí forms conmigo, contigo, and consigo:
Está jugando conmigo.
Hablaba consigo (con él mismo).
¿No quiso ir contigo?
PRÁCTICA 37. Exprese en español.
1. Life is more beautiful with you. It is better without worries, too. We can live without them.
2. There was a more difficult problem before (ante) us, but we didn't talk about it.
3. He wanted to talk with me about it, but I refused. I was too mad at (enojado/a con) him.
4. All of them are fighting among themselves for (por) her.
38. Object Pronouns / Los pronombres de complemento
70
Direct
English
Indirect
me
me
me
te
you
te
lo/la
him, her, it, you (Ud. form)
le
nos
us
nos
os
you (plural) [in Spain]
os
los/las
them, you (Uds. form)
les
Direct objects: what? or whom?
Indirect objects: to/for whom?
I did...(what?) I did my homework. I
did it.
Hice la tarea. La hice.
I did (for
whom?)
I did her homework for her.
Le hice la tarea (a ella).
I
I saw the doctor. I saw
saw...(whom?) him.
Vi al doctor. Lo vi.
I said (to
whom?)
I told that to Juan. I told him.
Le dije eso a Juan. Le dije.
• Note that direct and indirect pronouns only differ in the third person (singular and
plural).
• Spanish places these pronouns immediately before a conjugated verb. The object
pronoun eliminates the need for the personal a:
¿Conoces al abogado? Sí, lo conozco, pero no le debo nada. ¡No te creo!
When
these pronouns are the object of an infinitive or a gerund, they are joined to the
•
end:
Al verla, supe que la amaba.
Upon seeing her, I realized I loved her.
By writing to them, they'll understand you
Escribiéndoles, te entenderán mejor.
better.
• Thus, object pronouns may precede the conjugated verb or be attached at the end of
the infinitive or gerund (but they can never come in between):
No me quieres creer.
-->
No quieres creerme.
Siguieron buscándolo.
-->
Lo siguieron buscando.
Le voy a dar un regalo. -->
Voy a darle un regalo.
• When used together, indirect object pronouns precede direct object pronouns (the
exact opposite of the usual English order):
Nos los entregó a tiempo (los
She delivered them to us on time.
mensajes).
Quiero mandártelo/Te lo quiero
I want to send it to you.
mandar.
• When the first pronoun of the two would be le or les, it changes to se (singular or
plural).
I want to explain it to her before showing it
Se lo quiero explicar antes de
to her.
enseñárselo.
As
there
frequently
is
ambiguity
with
the
third person, clarify with a él, a ellas, a
•
71
usted(es), etc.:
Ella le escribe a él; yo le escribo a
She is writing to him; I am writing to her.
ella.
No le quiero pedir un favor a usted. I don't want to ask you a favor.
No quiero pedírselo a ustedes
I don't want to ask it from you guys, either.
tampoco.
I sent it to him first.
Se la mandé a él primero.
• Spanish prefers to use the indirect object pronouns even when the indirect object is
mentioned:
Le estoy escribiendo una carta al jefe. I am writing a letter to the boss.
¿Por qué no le pides dinero a tu hija? Why don't you ask your daughter for money?
• Note the following verbs, which take certain prepositions in English, but not in
Spanish (except for the personal a when needed):
Miré al jugador y miré el campo. Los I looked at the player and at the field. I
looked at them.
miré.
They're looking for the lawyer. They're
Buscan a la abogada. La buscan.
looking for her.
Escuchamos el concierto. Lo
We listened to the concert. We listened to it.
escuchamos.
I asked for two paellas. I requested them to
Pedí dos paellas. Se las pedí al mesero.
the waiter.
They stole the money from her. They stole it
Le robaron el dinero. Se lo robaron.
from her.
Le voy a quitar el trabajo. Voy a
I'm going to take his job away from him.
quitárselo.
PRÁCTICA 38A. Sustituya los complementos directos por su pronombre correspondiente:
1. Mandó las fotos.
2. Está leyendo la novela nueva.
3. Quería ver el programa otra vez.
4. Va a perder las elecciones.
5. Pidieron la cuenta.
6. Sigo dando dinero a los pobres.
7. Estaban mirando el atardecer.
8. Continuó escuchando música.
9. Busco al profesor Pérez.
PRÁCTICA 38B. Sustituya los complementos indirectos por su pronombre correspondiente:
72
1. Mandó las fotos a Ud.
2. Va a contestar al presidente.
3. Está escribiendo a su abogado.
4 . Tuvo que vender su coche a sus padres.
5. Continué dando dinero a los pobres.
PRÁCTICA 38C. Sustituya todos los complementos posibles por sus pronombres correspondientes:
1. Está escribiendo la carta a su abogado.
2. Va a contestar el mensaje al presidente.
3. Tiene que vender su coche a sus padres.
4. Mandó las fotos a Ud.
5. Sigo dando dinero a los pobres.
6. Les pedí un favor a mis vecinos.
7. Le están robando el coche.
8. Conocimos al candidato.
9. Le puedo mandar una carta a María el lunes.
39. Gustar and Other Verbs Used with Indirect Objects
There is a group of verbs that are most frequently used with indirect objects. The most
common is gustar, which is the equivalent of to like, but functions very differently. In
English, the expression "I like the book" makes me the subject and the book the object. In
Me gusta el libro, the book is the subject of the sentence and I the indirect object. It is as
if we were to say in English "The book is pleasing to me": El libro me gusta (note that
the article is needed). Hence:
Indirect object
Verb (and
adverb)
Subject
Nos
gusta mucho
el fútbol.
We like soccer a lot.
A mí no me
gustaban
los periódicos.
I did not like newspapers.
Al profesor le
gustó
la clase.
The teacher liked his class.
¿Te
gusta
leer?
Do you like reading?
73
Me
gustas
I like you (romantically).
tú.
• A sentence beginning "Peter likes..." will have to begin A Pedro le gusta...:
A la mayoría de la gente le gusta dormir. Most people like to sleep.
• Where English says "I don't like it" or "I like them", it or them is the subject of the
verb that Spanish uses. Spanish omits these subject pronouns:
Do you like poetry? Yes, I like it.
¿Te gusta la poesía? Sí, me gusta.
Do not try to express it or them with gustar. If you must, it will have to be with the
appropriate demonstrative pronoun: Me gusta eso, me gusta esta.
• Adverbs such as mucho, poco, and un poco are usually placed right next to gustar
and similar verbs, since the adjectives will be next to nouns:
They like drinking water a lot.
Les gusta mucho tomar agua.
They like drinking lots of water.
Les gusta tomar mucha agua.
• For clarity or contrast, a + prepositional pronoun can be used:
A ella le gustan los viajes, pero a él no. She likes trips, but he doesn't.
A mí me gustó el concierto, pero a ti no. I liked the concert, but you didn't.
• To like better is expressed by gustar más:
We liked this reading better than the
Nos gustó más esta lectura que la otra.
other.
PRÁCTICA 39A. Exprese en español.
1. I like the theater.
2. He likes skating.
3. We like it a lot, too.
4. Did you like the movie?
5. Did you like movies a lot when you were a child?
6. My oldest sister didn't like trips.
7. My parents didn't like them much, either.
8. Most people don't like problems very much.
9. Most of the country liked the first candidate better than the second.
10. I liked them both.
11. Children like to play.
12. As a child, I used to like playing, too.
13. Mary likes Joe.
74
Other verbs that express personal reactions and function like gustar are the following:
agradar
caer
bien/mal
disgustar
doler
encantar
to like (not romantically)
to (dis)like a person (not
romantically)
to annoy
to hurt
to like a lot (love)
fascinar
importar
to fascinate
to care about
interesar to interest
molestar to bother
preocupar to concern,
worry
to irritate
enojar
sorprender to surprise
I love baseball.
Me encanta el béisbol.
A los murciélagos les molesta la luz. Light bothers bats.
Insects fascinate us.
Nos fascinan los insectos.
Noise annoys José.
A José le disgusta el ruido.
Do you like your neighbor?
¿Te agrada el vecino?
I don‘t like this professor.
Me cae mal este profesor.
I don't care about the news.
No me importan las noticias.
We were not interested in it.
No nos interesaba.
My arm hurts (me). It hurts.
Me duele el brazo. Me duele.
The neighbor is worried about her
A la vecina le preocupan sus flores.
flowers.
The verbs faltar, quedar and parecer are used like gustar, but they also appear without
an indirect object in impersonal statements:
This cake is lacking (missing) more
A este pastel le falta más sabor.
flavor.
More education is lacking (needed) in
Falta más educación en la sociedad.
society.
Poetry seems strange to me.
A mí me parece extraña la poesía.
Poetry seems strange.
La poesía parece extraña.
Te queda poco tiempo para terminar. You have little time left to finish.
There's little time left before
Queda poco tiempo para las
vacation.
vacaciones.
A common idiomatic expression for How did you like...:
What did you think of the movie?
¿Qué les pareció la película?
We found it interesting.
Nos pareció interesante.
What did you think of the poems?
¿Qué te parecieron los poemas?
I loved them.
Me encantaron.
PRÁCTICA 39B.
1. Comente su viaje más reciente con su familia o sus amigos:
¿Qué les encantó?
¿Qué les interesó?
75
¿A quién(es) les gustó más el viaje?
¿Por qué? ¿Qué les faltó hacer?
¿Cómo les pareció cada cosa?
2. Comente sus gustos y los de su familia y amigos durante su infancia:
¿Qué les parecía divertido?
¿Qué les enojaba, molestaba, importaba o preocupaba?
¿A quién no le gustaban los deportes?
¿Qué les encantaba?
¿Quién les caía mal?
¿Por qué?
Repaso número ocho
A. Produzca oraciones usando los elementos dados y haciendo los cambios necesarios.
Ejemplo: toda la gente / encantar / música. —> A toda la gente le encanta la música.
1. mayoría de la gente / interesar / diversiones.
2. médicos / faltar / entender mejor el alma.
3. tú / quedar / poco dinero en el banco.
4. yo / doler / músculos / después de correr.
5. gobierno / preocupar / problemas sociales.
6. artistas / interesar / misterio.
7. sociedad / importar / apariencias.
8. tus amigos / parecer / cómico tu padre.
9. perros / fascinar / tocino.
10. profesor López / molestar / errores.
B. Exprese en español:
76
1. He refused to go out with me. I am tired of calling him.
2. My older sister's house used to be bigger than his. Both of them were comfortable.
3. I have been sitting here for more than an hour. I can't continue to wait for him.
4. I liked her lecture far better than his. But he didn't like it at all.
5. I know your boyfriend better than you. He is not going to speak with you about it.
6. You didn‘t tell us when (was it that) you met your husband.
7. This is the worst restaurant in town, but there is another nearby that is better.
8. My first problem was finding a less expensive house. I saw a good one two days ago.
9. Fortunately, nobody drank as much beer as doctor know-it-all. He loves it.
10. Pedro said yesterday he wanted to assist us with everything we had to do.
Práctica escrita
A. Conteste las preguntas utilizando pronombres de complemento. Use la información entre paréntesis
para añadir un comentario a cada respuesta. Ejemplos:
¿Viste la última película de Almodóvar? (anoche, interesar, + , anterior)
_Sí, la vi anoche, y me interesó más que la anterior._
¿Le traías manzanas a tu profesor de la escuela primaria? (con frecuencia, agradar, = , buenas notas)
_Sí, se las traía con frecuencia, y le agradaban tanto como las buenas notas._
1 . ¿Escuchaste el último disco de Shakira? (hace una semana, sorprender, =, primero)
2. ¿Les contabas tus secretos a tus amigas cuando eras niño/a? (a veces, interesar, - , ahora)
3. ¿Conociste a las peruanas que vinieron anoche? (a todas, agradar, + , la semana pasada)
4. ¿Les mostrabas tus tareas a tus padres en tu infancia? (todas las noches, preocupar, +, notas)
5. ¿Hiciste los ejercicios del repaso número siete? (la semana pasada, aburrir, = , otros)
77
B. Primero, lea la versión en inglés. Después, complete el párrafo con las palabras y
letras necesarias. Finalmente, escriba dos preguntas sobre el texto y respóndalas.
Am_rica Latin_ es mucho menos ind_gena ___ lo que piensa la mayoría de los
_orteamerican___, qui_____ ___ conoc__ sol_mente por ___ publicidad o por los estudios
an_ropol_gico_. Claro que ___ mayor _______ de los _atinoamericanos de hoy ________
orgullos___ de su herencia precolombin_, y hoy se hacen ____ esfuerzos ____ nunca para
respetar los derechos indígen___. Pero ________ muchos extranjeros no saben, ______ que
hubo una _______ inmigración _urope_ en el Cono Sur, ________ en el siglo XIX como en el
____. Además, durante el periodo colonial, los _spañoles y portu__eses se mezcl_______ con los
indígenas y con ___ población de ascendencia _frican_. ____ combinarse entre ____,
produ_______ la raza hoy llamad__ "mestiza", que constituye la mayor_a de la po________
_atinoamerican_. No hay que olvidar _________ la influencia asiátic_, ___ cual no es ____
masiv__ como en Estados Unidos, pero sí tiene ____ presencia bien conocid_ en Bra_il, Cuba y
Per_, entre otros. __ muchas personas ____ sorprende saber que solamente Guatemala y _olivia
tie_____ ma________ ind_gena. Lo cierto es que ________________ es u__ de l_s regi_nes m_s
divers_s ______ mundo, tant_ racial _______ cultural________. Est_ ____ su m_____r riqueza.
Latin America is much less indigenous than most North Americans think, who know it only through
publicity or from anthropological studies. Of course, most Latin Americans today are proud of their preColumbian heritage, and today, more efforts than ever are made to respect indigenous rights. But what
many foreigners do not know is that there was a large European immigration to the Southern Cone, both in
the 19th century and the 20th. Furthermore, during the colonial period, Spaniards and Portuguese got mixed
with the indigenous people and with the population of African descent. In combining among themselves,
they produced the race called "mestizo" today, which constitutes the majority of the Latin American
population. Asian influence should not be forgotten either. It is not as massive as in the United States, but
has a well-known presence in Brazil, Cuba and Peru, among others. Many people are surprised to learn that
only Guatemala and Bolivia have an indigenous majority. The truth fo the matter is that Latin America is
one of the most diverse regions in the world, both racially and culturally. This is its greatest asset.
40. Reflexives / Los reflexivos
A. USE OF REFLEXIVE PRONOUNS / USO DE LOS PRONOMBRES
REFLEXIVOS
singular:
me
te
se
plural:
nos
os
se
Me encontré en un problema.
Ella se escucha.
Nos estamos vistiendo.
Estamos vistiéndonos.
No se quisieron ver.
No quisieron verse.
78
I found myself in trouble.
She listens to herself.
We are dressing (ourselves).
They refused to see each other (or
themselves).
• Reflexives are placed in the same position as other object pronouns: immediately before
a conjugated verb, or joined to the end of a gerund or infinitive.
• The plural reflexive pronouns may mean either themselves or each other. Logic will
normally indicate which is intended; a sentence such as Se escriben is more likely to
mean "they write to each other" than "they write to themselves". If it is important to
make clear which is meant, insert the adverb mutuamente or variations of entre sí,
entre vosotros, entre nosotros, uno al otro, una a la otra, unas a las otras, etc.:
They remind themselves.
Se recuerdan a sí mismos.
They remember one another.
Se recuerdan uno al otro.
They remember/remind each
Se recuerdan entre sí.
other.
• Spanish uses reflexive pronouns more frequently than English does. Whereas, for
example, English says "I dressed"' Spanish must say "I dressed myself" making it clear
that the action of the verb is directed back at the subject. Here is a list of some common
verbs used frequently with a reflexive pronoun. Nearly all of these verbs may be used
as transitive verbs in sentences such as "I dressed the children", "I woke them up", etc.
to go to bed, to lie down
acostarse (ue)
to get married (to)
casarse (con)
to wake up
despertarse (ie)
to have fun, enjoy oneself
divertirse (ie)
to get up
levantarse
to sit down
sentarse (ie)
to get dressed
vestirse (i)
I got up early.
Me levanté temprano.
I put the children to bed at ten.
Acosté a los niños a las diez.
You amused your brother with your
Divertiste a tu hermano con tus chistes.
jokes.
She bathed the dog before bathing
Bañó al perro antes de bañarse ella.
herself.
•Verbs with a reflexive pronoun are not to be confused with verbs like gustar, which take
indirect object pronouns and usually appear in the third person:
Verbs using reflexive
Verbs like gustar (indirect object pronoun):
pronouns:
Me gustó la fiesta.
Me divertí en la fiesta.
Nos encanta la mañana.
Nos levantamos por la mañana.
Se acostaba bajo las
Le fascinaban las estrellas.
estrellas.
Se sientan a estudiar
Les interesa la astronomía.
astronomía.
PRÁCTICA 40A. Termine las oraciones con la forma reflexiva, e incluya el pronombre
al final para enfatizar el contraste con la primera parte de la oración. Añada también
una expresión con gustar.
Ejemplo: Primero vestía a los niños y después... _se vestía él. No le gustaba vestirse._
5. Ciertamente divirtieron al niño, pero
también...
1. Hoy desperté a mi hermano, pero primero...
79
2. Vistió a su amo (master) y después...
6. Bañasteis al gato y al mismo tiempo...
3. El ministro casó a su hermana hace un mes, y ayer... 7. Acostamos a los niños antes de...
4. Debes sentar primero a los invitados y luego...
8. Nunca acuesto a mis hijos sin...
B. VERBS USED REFLEXIVELY / VERBOS PRONOMINALES
There is a considerable number of verbs whose meanings change when used reflexively,
although many can be used as transitive verbs, too. An interesting case is sentir, to feel,
which takes a reflexive pronoun with adjectives and adverbs (se siente bien), but not
otherwise (siente alegría). Here are some other verbs for reference:
comunicarse (con)
to communicate (with) (comunicar: to communicate
something)
dormirse (ue)
to go to sleep (dormir: to sleep)
enamorarse (de)
to fall in love (with) (enamorar: to seduce)
esforzarse (por) (ue)
to make an effort to (only used reflexively)
irse
to go away (ir: to go somewhere)
llevarse
to carry off something (llevar: to carry somewhere)
preocuparse (por)
to be worried (about) (preocupar: to worry someone)
quedarse
to stay (quedar: to remain, to be or have left).
quejarse (de)
to complain (about) (only used reflexively)
reunirse (con)
to get together in a meeting (reunir: to gather something)
They want to get in touch with you.
Are you leaving without
complaining?
They fell in love with the little dog.
We were making an effort to
understand.
I didn't sleep much this week.
They are going to stay in San José
because they don't have any time
left.
Se quieren comunicar contigo.
¿Vas a irte sin quejarte?
Se enamoraron del perrito.
Nos esforzábamos por entender.
Dormí poco esta semana.
Se van a quedar en San José
porque no les queda tiempo.
80
PRÁCTICA 40B.
1. Complete las oraciones con los reflexivos y las terminaciones apropiadas del pretérito
o el imperfecto.
Escriba una X en los espacios que no necesitan nada.
a. Su esposo ___ sent___ cansado con frecuencia porque no ___ pod___ dormir bien,
pero ese día ___ sint___ alegría al verla.
b. Gloria y Esteban ____ divorciar___ hace un año, porque no ___ pod____ comunicar____
bien.
c. Después, Esteban ____ enamor___ a una vecina, y Gloria ____ enamor__ del jefe (boss).
d. Gloria ___ fue de la casa y ___ llev_ a los niños. Esteban __ qued___ en la misma casa hasta
hoy.
e. Ella y yo siempre ____ esforz_______ por resolver___ los problemas pacíficamente.
f. Tú nunca quis____ ni quejar___ ni preocupar___ por la situación en que est________.
g. Cuando ___ fui del país, el gobierno continu____ esforzándo___ por encontrar___ soluciones.
2. Exprese en español:
a. He looks at himself while getting dressed. He feels tired. b. They found each other a
year ago and they immediately fell in love. c. I never worry about the news. There's
nothing new. d. Complaining about the food was not going to solve anything. e. He
stayed here (for) a few weeks. He left (went away) on Thursday. f. She felt asleep
quickly, but didn't sleep well. g. People need to communicate more often.
C. TO BECOME: HACERSE, PONERSE, VOLVERSE...
verb
used with
suggests
hacerse
mostly nouns
effort for status or gradual transformation
ponerse
adjectives only
temporary change in emotional or physical
state
volverse
adjectives or
nouns
a more lasting change in emotional or physical
state
convertirse (ie)
nouns only
en
transformarse en
a lasting, objective transformation
Se hizo abogado. Su vida se hizo intensa.
Se pusieron pálidos y furiosos. Me puse
triste.
El tema se puso / se volvió difícil.
La vida se volvió una pesadilla.
Su casa se convirtió en un hotel.
Los problemas se convierten en
oportunidades.
81
He became a lawyer. His life got
intense.
They became pale and angry. I got sad.
The topic became difficult.
Life turned into a nightmare.
Their house became a hotel.
Problems become opportunities.
El nombre de ―Hispania‖ se transformó en ―España‖.
The Spanish equivalent of to become or to get + adjective is often a reflexive verb.
Some common examples:
to become glad, happy
alegrarse
to get tired
cansarse
to realize (become aware of)
darse cuenta (de)
to get angry
enojarse
to get sick
enfermar(se)
to become (get) independent
independizarse
to get prepared, get ready
prepararse
to go mad
volverse loco
PRÁCTICA 40C.
1. Complete las oraciones con el equivalente apropiado de become.
a. Esta ciudad está _______________ en un centro turístico.
b. Después de una dura campaña política, el candidato liberal _______________ presidente.
c. No fue fácil para mí ________________ rico. Tuve que esforzarme mucho.
d. La clase __________________ una fiesta después de que llegaron los músicos.
e. Cuando recibieron la noticia, ________________ contentos, pero yo ___________ triste.
f. Las antiguas colonias inglesas __________________ en los Estados Unidos de América.
g. Las condiciones de vida _________________ imposibles poco a poco.
2. Responda las preguntas de manera lógica, traduciendo adecuadamente el verbo entre paréntesis.
a. ¿Cómo te pones cuando hace sol? (get glad)
b. ¿Cómo te sentiste cuando te insultó Gabriel? (got angry)
c. ¿Por qué Cuba ya no es parte de España? (got independent)
d. ¿Por qué Nietzsche no siguió escribiendo? (went mad)
e. ¿Cómo es que Manuela no vino contigo? (got sick)
f. ¿Por qué no siguieron Uds. jugando? (got tired)
g. ¿Por qué no te casaste con Pedro? (realized didn't love him)
h. ¿Cómo aprobaste el examen de conducción? (got well prepared)
i. ¿Cómo te sientes cuando termina el semestre? (get very glad)
D. REFLEXIVE FOR POSSESSION
Spanish avoids the use of possessive adjectives with parts of the body, clothing, and
personal items. Indirect object pronouns are often used to point in the direction of the
person concerned:
They took her (Carmen's) purse away.
Le quitaron la bolsa (a Carmen).
82
She is bringing her car. She's bringing
it.
Her head hurts. It hurts.
Le duele la cabeza. Le duele.
Where the action is directed toward oneself, a reflexive pronoun is used:
I washed my hands. I washed them a while
Me lavé las manos. Me las lavé hace
ago.
rato.
They took off their glasses.
Se quitaron los anteojos.
She hit her leg. She hit it while
Se golpeó la pierna. Se la golpeó al
entering.
entrar.
Va a traer el carro. Lo va a traer.
PRÁCTICA 40D. Exprese en español:
1. My pinkie hurts.
2. He took off his pants.
3. They robbed her heart.
4. They robbed it from her.
5. He continues to scratch his (own) head.
6. He continued to scratch her head.
7. He washed his hands.
8. He washed them (his hands) while washing the dishes.
9. I hit my eyelash and it hurts.
10. His leg hurts because he hit it while entering.
11. He's bringing his jacket.
12. That took her peace away.
41. The Passive Voice / La voz pasiva
A. SER + PAST PARTICIPLE
The passive voice is formed in Spanish in the same way as in English, with the
appropriate form of the verb ser and the past participle (which, as an adjective, must
agree in gender and number with the subject):
This book is read by everyone.
Este libro es leído por todo el mundo.
Las rosas fueron compradas por la niña. The roses were bought by the girl.
The window was broken by the police.
La ventana fue rota por la policía.
The agent of the verb, if mentioned, comes after the preposition por.
83
For use of estar with past participle (resulting condition).
PRÁCTICA 41A. Complete las oraciones con el verbo ser y el participio del verbo entre
paréntesis.
1. América _____________________________ por Cristóbal Colón en 1492. (explorar)
2. Este ____ un problema muy ______________ hoy en el país. (discutir)
3. En aquella época, los africanos muchas veces ____________________ como esclavos.
(vender)
4. El secreto no _____________________ hasta hace poco tiempo. (revelar)
5. Nuestra casa _____________________ por un famoso arquitecto mexicano. (construir)
6. Todos nosotros ____________________ por la policía después del accidente. (interrogar)
7. La ciudad de Lima _____________________ en 1535 por Francisco Pizarro. (fundar)
8. Las minas de plata de Potosí, en Bolivia, _________________ en 1545. (descubrir)
9. Paraguay _________________ por tropas brasileñas entre 1870 y 1876. (ocupar)
B. PASSIVE CONSTRUCTIONS USING SE
• Spanish avoids the passive with ser when the agent of the action is unknown or
irrelevant. The most common way of expressing a passive idea in Spanish –that
something happens, but without expressing who in particular does it–, is by using se
with a verb in the third person:
Spanish is spoken here.
Aquí se habla español.
You need to be patient.
Se necesita tener paciencia.
One eats / People eat well in France.
Se come bien en Francia.
Is it possible to travel without a visa?
¿Se puede viajar sin visa?
That model is not used (in use)
Ese modelo ya no se usa.
anymore.
• When the subject (model in the above example) is plural, the verb must be plural:
Those models are not used anymore.
Esos modelos ya no se usan.
Shirts are sold in this store.
Se venden camisas en esta tienda.
En América no se conocían los caballos. Horses were not known in America.
Similar to English, Spanish can also use an impersonal They..., without the subject
• pronoun:
Allá no respetan los derechos humanos. They don't respect human rights there.
They signed the agreement this
Firmaron el acuerdo esta mañana.
morning.
The agreement was signed this
O: El acuerdo se firmó esta mañana.
morning.
PRÁCTICA 41B.
1. Complete con se y el pretérito (puntual) o el imperfecto (duración) del verbo indicado.
a. En la época colonial española no siempre (obedecer) ________________ las leyes.
b. En el siglo XVIII (producir) ________________ varios conflictos por esta causa.
c. Por ejemplo, oficialmente sólo (poder) ________________ comerciar con Madrid,
d. pero con frecuencia (traficar) ______________ esclavos y productos con los británicos.
e. Por eso, en la década de 1770, (decidir) _____________ liberalizar el comercio.
f. Así (iniciar) ______________ un proceso de más comunicación con Europa.
g. Pero los conflictos legales no (resolver) _________________ en ese momento.
84
2. Narre o invente algunas noticias recientes utilizando expresiones impersonales. Por ejemplo:
Pronostican un día de sol el sábado. Se aconseja ir a la playa. Se aprobó la ley a favor de los
desempleados. Se informó que la gente puede cobrar un seguro de desempleo. Etcétera.
C. SE + INDIRECT OBJECT PRONOUNS (SE ME, SE LES, ETC.)
• The passive se may be followed by an indirect object pronoun indicating the person
who undergoes the action of the verb (to whom?):
Flowers were sent to her. (She was sent
Se le mandaron flores (a ella).
flowers)
Se les ofreció el trabajo (a ellos). The job was offered to them.
That wasn't told to me. (I wasn't told that).
No se me dijo eso (a mí).
These examples express the common "false passive" in English, that is, a construction
that uses the indirect object (to whom?) as the subject: "She was sent flowers"; "They
were offered...";―I was told‖. But ser won't work for this type of constructions.
• The combination of se + indirect object pronoun (se me, se te, se le[s], etc.) is also
used to express the ideas of losing, breaking, forgetting and dropping, when not
deliberate, underlying that something happened by accident. Contrast the following
examples:
Se le rompieron dos costillas.
Rompió su promesa.
He broke two ribs (by accident). He broke his promise.
Se les perdió la llave.
Perdieron el partido.
They lost their key.
They lost the game (match).
Se me cayó el libro.
Caí en una trampa.
I dropped the book.
I fell into a trap.
Se te olvidó mi dirección.
Olvidaste tus malos recuerdos.
You forgot my address.
You forgot your bad memories.
Se nos acabó la gasolina.
Acabamos el trabajo.
We ran out of gas.
We finished the paper (or the job).
Notice that, in the ―accident‖ construction, the object in English is the subject in
Spanish (llave, libro, etc.). Thus, the verb uses the third person singular or plural to
agree with the thing(s) lost, broken, forgotten, etc. This is similar to, but not to be
confused with, the constructions used for verbs like gustar, which do not normally use
se.
PRÁCTICA 41C. Exprese en español:
1. She wasn't told why.
2. I was given the necessary directions.
3. He broke the cup (by accident).
4. We forgot to do our homework.
85
5. Did you drop your wallet? Yes, and I lost it.
6. They ran out of time for (para) the exam.
7. He got angry and broke all of the dishes (intentionally). I forgot why.
8. As a child, I was told that adults were always right.
9. They forgot their own names.
10. I forgot all of the words during the presentation.
42. Obligation: Se debe, hay que
• A general sense of obligation is often expressed impersonally. Where English says you
should, one ought to, etc., Spanish would typically use an expression with se:
One (You) should follow directions
Se debe seguir las instrucciones con
carefully.
cuidado.
One (You) ought to have experience.
Se necesita tener experiencia.
Se tiene que ser experto para ser
One has to be an expert to be admitted.
admitido.
• Hay que is a common way to express obligation, both formally and informally. It can
be used in all tenses, and is always followed by the infinitive:
One should know what happens in the
Hay que saber qué pasa en el mundo.
world.
You had to go across the region on foot.
Había que atravesar la región a pie.
We'll have to wait until tomorrow.
Habrá que esperar hasta mañana.
One ought to keep history in mind.
Hay que tener en cuenta la historia.
PRÁCTICA 42. Describa qué se debe, se necesita o hay que hacer en su actividad
favorita. Por ejemplo: "Para tocar bien el piano se necesita mucho tiempo y sensibilidad.
Hay que practicar todos los días", etc.
86
43. Por/Para
These prepositions are very common in Spanish and, unfortunately, both often
correspond to English "for". The following is an attempt to summarize the main uses of
the two:
A. PARA (en general indica destino o propósito):
Direction toward a
Salgo mañana para Madrid.
destination or goal
Vamos para la oficina.
1.
2.Purpose, use, goal taza para café coffee cup
or destination
Había espacio para todos.
toward a recipient.
Estudié medicina para ayudar.
Practican para mejorar.
Compró estos libros para ti.
Trabaja para la OTAN.
I am leaving tomorrow for
Madrid.
We're going to the office.
llantas para la nieve snowtires
There was room for everybody.
I studied medicine in order to
help.
They practice to improve.
He bought these books for you.
She works for NATO.
3.Time limit or
deadline:
Hay que hacerlo para el lunes.
It needs to be done by/for
Monday.
4.Comparison
against
the normal
expectation
Tiene una gran población para
un país tan pequeño.
Para ser extranjero, habla muy
bien el idioma.
It has a large population for such
a small country.
For a foreigner, he speaks the
language very well.
5.Opinion: "To
me..."
Para mí, la política es
interesante.
La vida es valiosa para
cualquiera.
To me, politics is interesting.
Life is valuable to any one.
A helpful tip: Whenever you could use in order to (even though you are saying just "to"
or "for") in English, you need to use para + infinitive in Spanish:
(in order) To improve, people have to do Para mejorar, hay que hacer
this.
esto.
I do it (in order) to have fun.
Lo hago para divertirme.
I do it for improving. (in order to
Lo hago para mejorar.
improve)
BUT:It's not easy to improve this.
No es fácil mejorar esto.
PRÁCTICA 43A. Identifique el uso de para que ilustra cada oración.
Ejemplo: Neruda es un símbolo nacional para los chilenos. _A5: Opinión_.
87
1. Íbamos a salir para la ciudad esa tarde, pero no pudimos. _____________
2. Necesitábamos llegar para el anochecer, pero no lo hicimos. _______________
3. Queríamos ir a la ciudad para asistir a la reunión del sindicato. ______________
4. Pero no había condiciones climáticas para un viaje tan largo. ______________
5. Estaba nevando demasiado para ser octubre, y no esperábamos eso. _____________
6. Era una reunión muy importante para nosotros, y lamentamos no llegar. ______________
7. La reunión era para debatir las estrategias del próximo mes. _______________
8. Hay que estar preparados para defender nuestros derechos. ______________
9. Para el profesor este tema es muy fácil de entender. ______________
10. Estas manos son para darte amor. _______________
B. POR (en general indica motivación o sustitución):
1. Exchange, substitution,
rate, correspondence (per)
Te doy cinco dólares por el I'll give you five dollars for
the book. Thanks for
libro. Gracias por todo.
everything.
Don't take me for an idiot.
No me tomes por idiota.
a veinte kilómetros por hora, el diez por ciento, $3 por
galón.
He deserves it for (because
2. Cause, reason, or motive
Lo merece por su
of an action (because of, to dedicación.
of) his dedication.
fetch)
We went to the store for
Fuimos al mercado por
bread.
pan.
Te quería por tu honestidad. I loved you for your
honesty.
No jugaron por la lluvia.
They didn‘t play because of
the rain.
On behalf of, for the sake Lo hice por el dinero.
I did it for the money.
of,
I'm not doing it on your
No lo hago por ti.
in favor of, out of
account.
They worry about me.
Se preocupan por mí.
They fight for
Luchan por la
independence.
independencia.
She worked for world
Trabajaba por la paz
peace.
mundial.
A menudo me pregunta por
He often asks about you.
ti.
3. Duration in time (often
omitted)
During the morning,
evening,
or afternoon
4. Movement through a place
Estuvo en México (por) seis He was (stayed) in Mexico
for six weeks.
semanas.
He used to work in the
Trabajaba por la mañana.
morning.
Iba por la calle cuando...
88
I was going down the
street...
5. Agent in a passive phrase
(by)
6. Means of transportation
or communication (by)
Vamos a pasear por el
parque.
Pasa por mi oficina.
We are going to take a walk
through/by way of the park.
Drop by my office.
Fue escrito por ella.
It was written by her.
Está afectada por la noticia. She's affected by the news.
El paquete llegó por avión. The package arrived by
plane.
Call me on the telephone.
Llámame por teléfono.
7. In a large number of idiomatic expressions:
por fin
por ejemplo
por desgracia
por supuesto
por eso
por aquí
finally, at last
for example
unfortunately
of course
that's why
around here
por lo menos
por favor
por primera vez
por casualidad
por otra parte
por lo tanto
at least
please
for the first time
by (any) chance
on the other hand
therefore
A helpful tip: The questions ¿para qué? (for what purpose?) and ¿por qué? (for what
reason or motive?) can give you clues on choosing the appropriate preposition. Both
prepositions will often be grammatically correct, but they would convey different
meanings:
Recibimos dinero por la investigación. Recibimos dinero para la investigación.
I got money from the research (done).
I got money for the research (to do). [in
[exchange]
order to]
Compré esto por ti.
Compré esto para ti.
I bought this because you wanted me to.
I bought this to give it to you. [destination]
[motive]
PRÁCTICA 43B. Identifique el uso de por que ilustra cada oración.
Ejemplo: Los chilenos admiran a Neruda por su poesía. _B2: Motivo_.
1. Maine es famoso por su belleza natural. _____________
2. Los Andes estaban habitados por los incas. ____________
3. Hay un médico por cada mil habitantes. _____________
4. Salieron por la puerta de atrás. ____________
5. Los africanos fueron esclavizados por más de trescientos años. ___________
6. No vale la pena probar narcóticos ni por curiosidad. _____________
7. Lo llevaron a la cárcel por robar. _________________
8. Hay que trabajar por el bien común. _______________
9. Nos esforzamos por crear un futuro mejor. ______________
10. Mándame un mensaje por correo electrónico. ______________
11. No hay que quejarse sólo por quejarse, hay que hacer algo. ______________
89
PRÁCTICA 43C. Complete con por o para:
1. Voy a comunicarme ______ fax ______ informarles rápidamente qué pasó.
2. Pude entrar sin problemas porque me tomaron ______ periodista. Asistir a la conferencia fue
útil ______ mis estudios.
3. El gobierno propuso un plan ______ controlar la inflación. ______ supuesto, el senado lo
rechazó.
4. Gloria no come postres ______ no engordar. Se preocupa ______ su salud.
5. ______ fin hicieron una fiesta ______ celebrar el cumpleaños de su abuela.
6. Muchos animales están muriendo ______ la contaminación. ______ lo tanto, hay especies en
peligro.
7. ______ varios años trabajó ______ una corporación multinacional.
8. ______ mí, lo mejor de ser estudiante son las vacaciones. Hay oportunidades ______ todo.
9. Muchas ciudades del Caribe han sufrido desastres ______ los huracanes.
10. Ya no me interesan esos juegos ______ niños. Pero estoy loco ______ los videojuegos.
11. ¿______ cuándo es la presentación oral? Es ______ el próximo miércoles.
12. Necesitamos prepararnos ______ esa presentación ______ lo menos desde el lunes.
13. ______ ir a Nueva York desde Boston ______ tierra, hay que pasar ______ Connecticut.
14. Estudiando, se preparan ______ un futuro mejor. Y no van a desanimarse ______ ningún
motivo.
15. ______ una universidad pequeña, hay muchísimas clases interesantes. ______ eso me gusta.
Repaso número nueve
A. Utilice el infinitivo o el gerundio de los verbos entre paréntesis. Hágalos reflexivos
cuando sea necesario. Observe también los usos de por y para en cada oración.
Ejemplo: (quedar) No les iba a _quedar_ suficiente comida para el día siguiente, porque
varios refugiados iban a continuar _quedándose_ en su casa esa noche.
1. (comunicar) ______________ bien es una habilidad útil en la vida. Muchas personas tienen
serios problemas por no ______________ sus sentimientos o ideas claramente.
2. (hacer) No siguió ______________ más fiestas para poder concentrarse en sus estudios
y para así ______________ un buen profesional.
3. (preparar) ______________ para cualquier eventualidad, no iban a tener preocupaciones.
Ahora podían ______________ su viaje por Suramérica sin ningún obstáculo.
4. (poner) No tiene sentido ______________ nerviosos por los exámenes. ______________
atención en clase, se puede tener éxito sin demasiado esfuerzo.
5. (reunir) El aprendizaje no consiste simplemente en ______________ información. También
es ______________ con otros para entender mejor las cosas y actuar con más eficacia.
B. Exprese en español:
1. This is another problem that must be postponed for next Monday.
2. The flowers were received by my mother on time. She loved them.
90
3. I realized that one had to make a lot of effort to succeed in the first year.
4. By travelling, I found out that in Mexico they work harder than I thought (used to
believe).
5. Is Spanish spoken in Brazil? No, everybody knows that they speak Portuguese there.
6. I broke my leg for the second time. I got very mad. It hurts a lot.
7. I am finished with this job, which was assigned to me long time ago. I got pretty
happy.
8. These concepts are used, both medically and industrially, for controlling people.
9. It is known that forests are destroyed in large quantities all over the world.
10. They were sent invitations for the wedding more than two weeks ago.
11. They were supposed to (had to) be here earlier to attend the lecture, but they couldn‘t
arrive on time because of the traffic. They felt frustrated.
12. He put on his hat and went away. He hit his head on leaving. He was furious.
13. You're not marrying Juan, are you? Are you going crazy?
Práctica escrita
A. Escriba un párrafo narrando un episodio de la historia de su país o de su región.
Incluya:
- por lo menos dos frases utilizando por y para
- varias oraciones en voz pasiva con ser y con se
- por lo menos un verbo reflexivo
- la expresión tanto...como (both... and)
- una comparación (más, menos, tan, etc.)
- una oración con lo cual
Tenga en cuenta (keep in mind) el uso del pretérito, del imperfecto, de ser y de estar.
91
B. Primero, lea la versión en inglés. Después, complete el párrafo con las palabras y
letras necesarias. Finalmente, escriba dos preguntas sobre el texto y respóndalas.
La ______________ los territorios _spañoles en Am_rica ___ independiza___ durante la segunda
década del siglo ____. Sin embargo, en Cuba y Puerto Rico se reforz__ ___ sistema colonial ____
miedo de las rebe_iones ___ esclavos, como ____ que o_urrieron en _ait_. Además, las dos islas
___ consideraban fundamenta___ para _spaña, ________ ____ su ubicaci_n militar estratégic_
como ____ su produc____ de azúcar. Fue ____ es_s raz_nes, entre otr_s, que _____ siguieron
_______ parte de España hasta fines del siglo _____. Durante __ segunda mitad ___ siglo, ___
industria azucarera _uban_ ___ convirtió __ la más importante ____ mundo. En _uba ___
producía m_s ___ la tercer_ parte de todo ___ azúcar mundial. Pero ____ tensiones intern___ se
__________ _______ ____ más fuert___ y en 1868 comenz__ la primer__ guerra por la
independencia _ubana, que duró diez a_os. ______ recuperar su control de __ isla, España
promet__ hacer reformas. En la práctica, los cambios que __ implementa___ fueron inefica____,
y ___ intensific__ la represi_n ______ evitar nuev___ levantamientos. Mil___ de cubanos ____
lucharon ____ la independencia fueron encarcela____ o exilia____. Así, en 1895 estalló de nuevo
___ guerra, que termin__ en 1898 con la intervenci_n militar _stadounidense. Cuba fue ocupa___
____ Estados Unidos hasta 1902, cuando ___ establec___ la primera Rep_blica. Puerto _ico, ___
otro lado, contin_a _______ territorio estadounidense hasta ___ presente.
Most Spanish territories in America became independent during the second decade of the 19th century.
However, in Cuba and Puerto Rico, the colonial system was reinforced out of fear from slave rebellions,
like the ones that occurred in Haiti. Furthermore, both islands were considered fundamental to Spain, both
because of their strategic military location and because of their sugar production. It was for these reasons,
among others, that both of them continued to be part of Spain until the end of the 19th century. During the
second half of the century, Cuban sugar industry became the most important in the world. More than a
third of all the sugar in the world was produced in Cuba. But internal tensions became stronger and
stronger, and in 1868 the first war for Cuban independence started, lasting for ten years. To regain its
control over the island, Spain promised to make reforms. In practice, the changes that were implemented
were ineffectual, and repression was intensified to avoid new upheavals. Thousands of Cubans who fought
for independence were either jailed or exiled. So, in 1895, war exploded again, ending in 1898 with the US
military intervention. Cuba was occupied by the US until 1902, when the first Republic was established.
Puerto Rico, on the other hand, continues to be a US territory until the present.
92
44. The Forms of the Present Subjunctive
Las formas del presente del subjuntivo
ayudar
poner
seguir
present
indicative
yo ayud-ø
yo pong-ø
yo sig-ø
-->
present subjunctive
--> ayude, ayudes, ayude, ayudemos, ayudéis, ayuden
--> ponga, pongas, ponga, pongamos, pongáis, pongan
--> siga, sigas, siga, sigamos, sigáis, sigan
• In order to form the present subjunctive, use the first person singular of the present
indicative –the yo form–, remove the -o, and add endings in -e for -ar verbs, and
endings in -a for -er and -ir verbs. This change of vowel may involve some spelling
changes:
pag-ø --> pague
busc-ø --> busque
empiez-ø --> empiece
• Stem-changing verbs in -ar and -er change their stems in the same way and in the same
cases as in the present indicative:
piense pienses piense
pensemos penséis piensen
vuelva vuelva
vuelvas
volvamos volváis
vuelvan
• Stem-changing verbs in -ir, whatever the change that occurs in the present indicative,
change the stem of the nosotros form of the present subjunctive from e to i and from o
to u.
muera
mueras muera
muramos muráis
mueran
sienta
sientas sienta
sintamos
sintáis
sientan
pida
pidas
pida
pidamos
pidáis
pidan
• There are only six verbs whose forms in the present subjunctive do not follow the rules
above:
ir --> vaya
ser --> sea
estar --> esté
saber --> sepa
haber --> haya
dar --> dé
(subjunctive for hay) (accent in yo and Ud. forms)
All the verbs we have studied so far, have been in the Indicative Mood because they
indicate states or actions that are presented as factual. The Subjunctive Mood is used,
mostly in dependent clauses, after verbs that suggest non-factual events, such as indirect
commands, doubts, etc. Compare the following two groups of sentences:
Presented as factual (indicative):
Desired, doubted (subjunctive):
I insist that he is here. Inisito en que está I insist that he be here. Insisto en que esté
aquí.
aquí.
I believe he did it. Creo que lo hizo.
I demand that he do it. Exijo que lo haga.
I know he is here. Sé que está aquí.
I doubt he'll be here. Dudo que esté aquí.
PRÁCTICA 44. Exprese qué quieren o dudan los otros, para lo cual se necesita el
subjuntivo. Cambie los complementos por pronombres.
Ejemplos:
¿estudias? Quieren que estudies. ¿pedimos un favor? Dudan que lo
93
pidamos.
1. ¿trabajas?
2. ¿entiendes el ejercicio?
3. ¿vuelves?
4. ¿construyes una casa?
5. ¿pensamos?
6. ¿bebemos vino?
7. ¿nos morimos?
8. ¿repites la pregunta?
9. ¿sabes?
10. ¿nos comunicamos?
11. ¿te vas?
12. ¿eres ingeniero?
45. Commands / Mandatos
A. FORMS OF THE IMPERATIVE / FORMAS DEL IMPERATIVO
All commands –except the affirmative tú and vosotros forms– use part of the present
subjunctive in what is called the imperative mood (el imperativo):
afirmativo
negativo
use present subjunctive
use present subjunctive
usted(es)
hable(n), coma(n), viva(n)
no hable(n), no coma(n), no viva(n)
ponga(n), vaya(n), sea(n)
no ponga(n), no vaya(n), no sea(n)
use 3rd person present indicative
use present subjunctive
tú
habla, come, vive
no hables, no comas, no vivas
trae, juega, oye
no pongas, no vayas, no seas
-ad, -ed, -id
use present subjunctive
vosotros
hablad, comed, vivid
no habléis, no comáis, no viváis
poned, id, sed
no pongáis, no vayáis, no seáis
• Usted(es) commands use third person of present subjunctive: vivan; no hagan; vuelva; etc.
The pronoun Ud(s). is sometimes used as a way of softening the imperative:
Pasen ustedes, por favor.
Please come in.
Please don't be so difficult.
No sea usted tan difícil.
Ustedes is the only plural form used in America, whereas in Spain it is reserved for formal
94
situations.
• Affirmative tú commands use the third person singular of the present indicative: vuelve;
piensa;
The following verbs are exceptions:
decir --> di
salir --> sal
poner --> pon
hacer --> haz ser
--> sé
venir --> ven
ir
--> ve
tener --> ten
• Negative tú commands use the tú form of the present subjunctive: no tengas miedo; no
salgas.
• Vosotros commands, used only in Spain now, simply change the –r from the infinite into a
–d ending for the affirmative (poner --> poned), and use the vosotros form of the present
subjunctive for the negative (no pongáis).
• Object pronouns are attached to the end of the affirmative command forms. This almost
always involves the placing of a written accent:
Escríbale usted; cómelo; dígamelo; mándamelos, etc. But hazlo; dime; etc.
• Object pronouns are placed immediately before the negative command forms:
No me hables; ni me lo digas; tampoco se lo pidas; nunca te levantes;
jamás se duerma usted; no se te olvide; etc.
PRÁCTICA 45A.
1. Transforme las oraciones en mandatos. Ejemplos:
No quiero que me cantes esa canción: ¡No me la cantes!
Insisto en que salgáis:
¡Salid!
13. Insisto en que leáis el libro.
1. No quiero que me des problemas.
2. Quiero que Ud. abra la puerta.
14. Insisto en que juegues conmigo.
3. No quiero que veas ese programa.
15. Quiero que estudiéis.
4. Insisto en que Uds. traigan a sus amigos.
16. Deseo que te prepares.
5. No quiero que se molesten.
17. No quiero que os preocupéis.
6. Necesito que me hagas un favor.
18. No quiero que te vayas.
7. Insisto en que Ud. me diga la verdad.
19. No quiero que trabajéis.
8. Espero que me creas.
20. No deseo que te duermas.
95
9. Espero que estudie bien el tema.
21. Quiero que sigas hablando.
10. No quiero que me paguen la deuda.
22. Espero que le mandes el telegrama.
11. Quiero que penséis.
23. Espero que tengas cuidado.
12. Ojalá vengas.
24. No quiero que les presten la moto.
2. Transforme cada afirmación como en el modelo.
MODELO: Pedro cierra la ventana. (tú)
Pedro closes the door.
Pedro, close the door.
Pedro, cierra la ventana.
Don't close the door.
No cierres la ventana.
Close
it. Don't close it.
Ciérrala. No la cierres.
a. María abre la puerta. (tú)
d. La señora Moreno dice la verdad. (Ud.)
b. Mi amiga sigue al senador. (tú)
e. El doctor Pérez se lleva a mi hermana.
(Ud.)
c. El mesero me trae la cuenta. (tú)
f. Los compañeros nos devuelven los mapas.
(Uds.)
B. OTHER EXHORTATIONS/ OTRAS EXHORTACIONES
The present subjunctive is also used in the following expressions:
1.
Comamos.
Cantemos otra cosa.
Sigamos durmiendo.
Let's eat.
Let's sing something else.
Let‘s go on sleeping.
• Common exception: vamos let's go.
• Personal pronouns follow their usual pattern of being joined to the end.
Escribámosle. Let's write to him.
Prestémoselo. Let's lend it to him/her.
• When a reflexive pronoun is used, the verb drops the final –s:
Sentémonos. Let's sit down.
Vámonos. Let's leave.
2.
Que vaya Pedro.
Let Pedro go (not me)
3.Wishes:
Que lo hagan ellos.
Let them do it, have them do it (I don't want to)
¡Que te diviertas!
Que disfruten (de) la película.
96
Have fun!
Enjoy the movie. (you all)
Get better (health).
May you do well / Be well.
Que se mejore.
Que te vaya bien.
PRÁCTICA 45B. Exprese en español.
1. Let's go to the movies on Friday.
2. Let's not work today. Let's have fun.
3. Let's listen to music.
4. Let's continue to write to her.
5. Let's send him a message via e-mail.
6. Let's ask him for money.
7. I don't want to get tired; let John go.
8. Have your employees do the work for (instead of) you.
9. He lost the address; let him go crazy looking for the house.
10. May you be happy!
11. May all of your dreams become true!
12. May life be perfect!
46. The Present Subjunctive in Noun Clauses
A noun clause is a group of words (containing a verb) which is the subject or object of
the verb of the main clause. In the sentence "I demand that he be here", the words "that
he be here" are the noun clause, object of the main verb "demand". The subjunctive is
used in a noun clause when the main verb expresses an indirect command, an emotional
attitude, doubt, disbelief or denial.
A. INDIRECT COMMANDS / MANDATOS INDIRECTOS
After verbs expressing want, desire, hope, request, insistence, command, suggestion, and
similar ideas, what is in the clause is like a command. The verb in such clauses must be
in the subjunctive.
Sugiero que lo hagas ahora.
Insisto en que esté presente.
I suggest that you do it now.
I insist that he be present.
• English frequently uses a phrase with the infinitive to express indirect commands: "I
want him to do it" etc. Spanish must say instead Quiero que lo haga ("I want that he
do it").
Me pide que la escuche.
Le rogamos que se vaya.
She asks me to listen to her.
We beg you to go away.
97
• If there is no change in subject there is no need for a subjunctive clause:
Quiero ir, ella insiste en hablar, espero terminar, etc.
• If the verb in the main clause simply reports the situation described in the subordinate
clause, there is no need for a subjunctive:
Insisten en que no lo van a visitar.
Les digo que está lloviendo.
They insist that they are not going to visit
I tell them that it is raining.
him.
• Other common verbs and expressions introducing indirect commands:
aconsejar, esperar, evitar, exigir, hacer, lograr, mandar, preferir, prohibir,
recomendar.
es esencial que, es importante que, es mejor que, es necesario que, es urgente que
• Ojalá, whose form is invariable, meant originally something like "O to Allah" and it is
used to express a general wish or hope. It is always followed by the present
subjunctive:
I hope they come tomorrow.
Ojalá (que) vengan mañana.
I hope it won't rain.
Ojalá no llueva.
Ojalá may be used with or without que with no change in meaning.
PRÁCTICA 46A. Conteste las siguientes preguntas
1. ¿Qué desean los padres que hagan sus hijos?
¿Qué espera un estudiante de sus profesores?
¿Qué expectativas tienen los ciudadanos sobre un gobierno? (to expect: esperar)
¿Cómo deseas que sea tu fin de semana? (usa ojalá)
¿Qué le pide la gente a Dios?
¿Cuáles son algunas sugerencias de los propietarios (landlords) para sus inquilinos
(tenants)?
¿Qué buscan los ecologistas para el planeta?
2. Complete con el infinitivo, el presente de indicativo o el presente del subjuntivo del verbo entre
paréntesis, según sea apropiado en el contexto. Utilice "que" si es necesario.
a. (ir) Me ruegan __________ a visitarlos.
b. (hacer) Les digo __________ frío.
c. (organizar) Necesitan __________ mi
cuarto.
d. (sentirse) Espero que ella __________
mejor.
e. (sentarse) Quiero __________.
f. (buscar) Les digo __________ mis cosas.
g. (empezar) Ojalá __________ el concierto.
h. (decorar) Necesito __________ mi cuarto.
3. Combine los dos verbos dados según los modelos:
Modelo 1: pedir/ ir : me pide que vaya
a. sugerir / dormir
98
b. rogar / entender
c. decir / salir
Modelo 2: esperar / callarse : espero que (él) se calle.
d. preferir / quedarse
e. querer / comunicarse
f. insistir en / preocuparse
Modelo 3: lograr / interesar : no logran que me interese
g. recomendar / cansar
h. hacer / molestar
i. lograr / sorprender
B. EMOTIONAL ATTITUDE, DOUBT OR DENIAL
When the main verb expresses any subjective attitudes (sorrow, surprise, anger,
indifference, concern, hope), doubt, disbelief or denial, the verb in the noun clause must
be in the subjunctive.
It surprises me that you do not know
Attitude:
Me sorprende que Ud. no lo sepa.
it.
It's sad that the sun isn't shining.
Es triste que no haga sol.
I am sorry (that) they don't know
Siento que no te conozcan.
you.
It's not possible that they are so
Doubt:
No es posible que sean tan tontos.
foolish.
I doubt/don't believe that it is true.
Dudo/no creo que sea verdad.
It's hard to believe you're leaving
Es increíble que te vayas ya.
now.
I am not saying (that) he is stupid.
Denial:
No digo que sea tonto.
Es falso que me moleste el ruido. It is false that noise disturbs me.
No es cierto que haya marcianos. It's not true that there are Martians.
• Spanish frequently uses the present subjunctive with a future meaning. There is no
longer a future subjunctive in daily use:
We doubt he'll say no.
Dudamos que diga que no.
Me alegra que haya clase
I'm glad there will be class tomorrow.
mañana.
• In general, creer, no dudar, ser verdad, and other expressions of belief or conviction
imply certainty and therefore will not require a subjunctive in the clause that follows,
99
while no creer, dudar, no ser cierto, etc., will. In questions, the clause may not involve
a subjunctive, depending on the doubt in the mind of the speaker: ¿Crees que va a
llover? or ¿Crees que llueva?
• Spanish may not omit the relative que as English often omits ―that‖.
I don't think (that) I'll arrive on time.
No creo que llegue a tiempo.
Some
common
verbs
and
expressions
implying
subjective attitudes:
•
enojar, gustar, molestar, sorprender, temer, tener miedo, interesar, importar,
lamentar, parecer raro/bonito/interesante que, ser normal/bueno/natural/curioso
que
• tal vez, quizás, quizá, which all mean perhaps, are followed by the subjunctive if the
speaker wants to convey a considerable degree of doubt (que is not used here):
Perhaps he will arrive soon. (probably
Tal vez llegue pronto.
not)
Maybe we'll go to the movies. (very
Quizás vamos al cine.
possible)
PRÁCTICA 46B.
1. Complete con la forma apropiada del verbo entre paréntesis
(algunas oraciones no requieren subjuntivo):
a. Nos encanta que ella (pagar) __
la cuenta de vez en cuando.
b. Espero que tú nos (visitar) ___
pronto para celebrar tu victoria.
c. Es triste que él no (tener)
____ amigos en el trabajo.
d. Es verdad que los españoles no (comer) ____ mucha carne.
e. No nos importa que ellos (parecer)
_____ estúpidos.
f. Diles que no (sentirse)
____ obligados a venir.
g. Yo les digo que (deber)
_____ irse ahora mismo.
h. Dudo que (ellos: llegar)
_____ a tiempo para hablar contigo.
i. Es posible que ellos lo (escribir) _____
para la fecha indicada.
j. Prefieren que yo no (irme) ____
para Europa todavía.
2. Comience cada oración con la expresión entre paréntesis, haciendo los cambios
necesarios. Algunas no requieren subjuntivo.
Ejemplo: Van a la estación. (Espero
Espero que vayan a la estación.
que)
-->
a. Hace sol esta tarde. (Ojalá que)
g. Está nevando. (Les digo que)
b. Todos los libros están en la biblioteca.
(Dudo que)
h. Se casa con mi hermano. (Creo que)
c. La preparamos pronto. (Nos piden que)
i. No saben leer francés. (No es posible
que)
d. Tenemos dinero en el banco. (Me alegra
que)
j. No es verdad. (No digo que)
100
e. No conocen al candidato. (No me importa k. No vienen esta semana. (Es triste que)
que)
f. No quieren pagar. (Me molesta que)
l. Continúas preocupándote. (Es natural
que)
Repaso número diez
A. Produzca cinco oraciones por cada elemento, como en el modelo.
1) Dame algo. 2) No me des nada. 3) Espero que me
MODELO: darme algo (tú):
des algo.
4) No creo que me des nada. 5) Creo que me vas a dar
algo.
traerme algo (tú)
1.
seguir quejándose de algo (tú)
6.
escuchar a alguien (tú)
2.
escoger algunas clases (tú)
7.
decírselo a alguien (tú)
3.
preocuparse por algo (Ud.)
8.
enamorarse de alguien (Ud.)
4.
entender algún problema (tú)
9.
llevárselas (las cosas) (Uds.)
5.
comunicarse con alguien (Uds.)
10.
B. Exprese en español:
1. I want you to send him a message via e-mail. Perhaps he'll like it (strong doubt).
2. Ask him for money to stay for one more week. Don‘t worry; he's going to say yes.
101
3. Let's ask him where he is going tonight. I hope he knows. I doubt he'll tell us.
4. We were sent a letter: let‘s answer it. I don't think it's too difficult to do.
5. Let‘s sit down and wait for her until 9:45, which seems logical to me.
6. If he is offered the job, let him accept it. It is terrible that there are no other options.
7. They suggest that we communicate frequently. I don't care that you don't care.
8. It is not true that the world population is growing more and more rapidly.
9. It is true that the country has a lot of problems because of its geographic location.
10. It is impossible for him to see you (that he see you). I hope he feels better by Friday.
Práctica escrita
A.Escriba sobre algunos de los siguientes temas:
1. ¿Qué opinan los extranjeros sobre tu país, tu región, tu preparatoria, tu familia?
(A los extranjeros les gusta que..., les molesta que, les sorprende que, les parece bonito
que, les parece curioso que, no pueden creer que...)
2. Haz algunas predicciones (tal vez, es posible, se espera, puede ser) sobre:
a. el futuro de la humanidad
c. el clima este fin de semana
b. la tecnología actual
d. lo que tal vez hagas después de graduarte
102
B. Primero, lea la versión en inglés. Después, complete el párrafo con las palabras y
letras necesarias. Finalmente, escriba dos preguntas sobre el texto y respóndalas.
El Distrito Federal de México, nombre oficial de es_ famos_ capital, __ una de las metrópolis
m_s grandes ____ mundo, con una po___ción de más ___ veinticinco mill_____ ___ habitantes.
Como ______ ubicada en un valle, las montañas no permit__ que el viento dispers__ los gases
produ______ ____ las f_bricas y los millones ___ vehículos que hay en ___ ciudad. Por ello ___
inevitable que est___ gases nocivos ___ qued___ atrapad___, convirtiendo a esta urbe en una de
las más contamin______ ____ planeta. ____ sorprendente que, a pesar de est_ desafortunad_
_en_meno, la ciudad contin___ atra_______ a millones de visitantes de tod_ el mundo. Aunque
__ sus habitantes ____ parece natural que ___ aire no sea ___ limpi_ ______ en otros lugares,
numerosos individu___ se enferman y much__ gente ___ quej__ de dolores de cabeza
fre_uent___. En su esfuerzo ____ mejorar la situaci_n, el go_ierno prohíb__ que los vehículos
sal____ a la calle diari_mente. Solo se permit__ que circul___ los automóviles marca____ con
calcomanías de cierto color para días espec_fic___ de la semana. Tambi_n __ organizan
campañas para fomentar mayor conciencia ecol_gic_ y lograr que l_s empresas cambi___ sus
hábitos contamina_____. Es poco probable que la contaminación desapare____ complet_mente,
pero se pueden tomar medidas eficaces ______ evitar que las circu_stancias empeor___. Es
interesante que precis_mente un _ientífico nacid_ en ___ ciudad de México, ___ profesor Mario
J. Molina, ____ uno de los expertos más importantes ____ mundo en el campo de los estudios
eco___________. A Molina se __ otorgó el premio Nóbel en 1975 _____ su trabajo en ____mica
atmos__ric_, y hace pocos años h_____ estudios específic___ _______ disminuir la
contaminación de su c__dad. Los mexicanos esper___ que su capital, ______ historia, belleza y
riqueza cultural ___ motivo de orgullo ______ todos, encuentr__ ___ solu_i_n satisfactori_ para
est___ problemas en un futuro cercan_. Tal vez todo sea cuestión de buena voluntad __
imaginación.
The Federal District of Mexico, official name of this famous capital, is one of the largest cities in the world,
with a population of more than twenty-five million inhabitants. Because it is located in a valley, the
mountains do not allow the wind to disperse the gases produced by factories and millions of vehicles (that
there are) in the city. That is why it is inevitable for those harmful gases to stay trapped, making this urban
area one of the most polluted in the planet. It is surprising that, despite this unfortunate phenomenon, the
city continues to attract millions of visitors from all over the world. Although it seems natural to its
inhabitants that the air is not as clean as in other places, numerous individuals get sick and many people
complain about frequent headaches. In its effort to improve the situation, the government prohibits vehicles
to go out in the streets daily. Only automobiles marked with stickers of a certain color for specific days of
the week are allowed to circulate. Educational campaigns are also organized to foment a bigger ecological
conscience and make enterprises change their polluting habits. It is not probable that pollution will
disappear completely, but effective measures can be taken to prevent circumstances from worsening. It is
interesting that precisely a scientist born in Mexico City, professor Mario J. Molina, is one of the most
important experts in the world, in the field of ecological studies. Molina was awarded the Nobel Prize in
1975 for his work in atmospheric chemistry, and not long ago he made specific studies to reduce pollution
in his city. Mexicans hope that their capital city, whose history, beauty and cultural richness is a motive for
pride to every one, will find a satisfactory solution to these problems in the near future. Perhaps it is all a
matter of good will and some imagination.
103
47. Subjunctive in Adjective Clauses
El subjuntivo en cláusulas adjetivas
An adjective clause is a group of words –containing a verb– which tells something about
a noun. This noun to which the clause refers is called its antecedent:
I admire
the woman
who teaches my children.
antecedent
adjective clause
The subjunctive is used in an adjective clause when the antecedent is someone or
something whose existence is unknown, hypothetical or uncertain from the point of view
of the speaker:
I am looking for a secretary who knows
Busco un secretario que sepa francés.
French.
(no personal a: it's not a definite person)
Necesito un libro que me explique esto. I need a (some) book that explains this.
There are no novels that may interest me.
No hay novelas que me interesen.
Is there anyone who may have any
¿Hay alguien que tenga sugerencias?
suggestions?
Choose whichever class you like best.
Escoge la clase que más te guste.
• If the antecedent is known to exist, the indicative is used:
I am looking for the secretary who knows
Busco al secretario que sabe francés.
French.
She needs the book that explains this.
Necesita el libro que explica esto.
There are some novels that interest me.
Hay novelas que me interesan.
Aquí hay alguien que necesita ayuda. There's someone who needs assistance here.
Escojo las clases que más me gustan. I choose those classes I like best (after
checking them out).
• When the antecedent is not mentioned, adjective clauses are introduced by lo que
(what...). Use the subjunctive when the reference is hypothetical or unknown (often in
the future):
Do what(ever) you (may) want.
Haz lo que quieras.
I do not care what they (may) say. (whatever
No me importa lo que digan.
it is)
• When lo que refers to something definite, it is followed by the indicative:
You do exactly what (you know) you want.
Haces exactamente lo que quieres.
I do not care about what they're saying (I
No me importa lo que dicen.
know what).
• Indefiniteness or lack of restriction is often stressed by repeating the verb in
subjunctive:
no matter what (whatever) they say
digan lo que digan
no matter what (whatever) you do
hagas lo que hagas
PRÁCTICA 47A. Produzca oraciones describiendo referentes primero hipotéticos y
después reales.
104
Modelo: yo / buscar / candidato / saber italiano / tenemos / no hablar otros idiomas.
Busco un candidato que sepa italiano, porque los que tenemos no hablan otros
idiomas.
yo / necesitar / libro / hablar sobre África / hay aquí / solamente hablar sobre Europa.
1.
gente / pedir / gobierno / hacer reformas / hay ahora / no servir para nada.
2.
necesitarse / sociedad / ser justa / tenemos / no ser igualitaria.
3.
no haber / empleados / tener experiencia / hay ahora / estar aquí hace poco tiempo.
4.
necesitarse / compañías / entender la ecología / hay ahora / no cuidar el medio ambiente.
5.
yo / buscar / trabajo / pagar buen sueldo / tengo / no ser suficiente para sobrevivir.
6.
PRÁCTICA 47B. Exprese en español.
1. Do what(ever) you want.
4. Eat what(ever) you like.
2. Choose any clothes you like.
5. She doesn't care whatever they do.
3. They're going to do it, whatever you say.
6. No matter what we do, he doesn't like it.
48. The Past Subjunctive / El imperfecto del subjuntivo
soñar
ellos preterite --> -ra: past imperfect subjunctive
soñaron
--> soñara, soñaras, soñara, soñáramos, soñarais, soñaran
ver
vieron
--> viera, vieras, viera, viéramos, vierais, vieran
oír
oyeron
--> oyera, oyeras, oyera, oyéramos, oyerais, oyeran
• The past subjunctive of all verbs is formed by changing the ending of the third person
plural (ellos) of the preterite from -ron to –ra. Note that the first person plural gets an
accent mark.
• Since the third person plural of the preterite of -ir stem-changing verbs has a special
stem-change, all forms of the past subjunctive will also have this stem.
durmiera, sintieras, pidiera, etc.
105
• Past subjunctive forms ending in -se exist, but are not frequently used.
• The imperfect subjunctive is used in subordinate clauses referring to past events in the
cases mentioned previously (§46, 47):
Indirect commands: Me pidió que fuera a visitarlo.
He asked me to go visit him.
Attitude:
We liked that he would come
Nos gustaba que viniera a
often.
menudo.
Doubt or denial: No es que estuviera triste.
It's not that I was sad.
Uncertain antecedent:
He wanted a book that
Quería un libro que lo explicara.
explained it.
• Following ojalá, the past subjunctive signals a hypothetical future situation:
Ojalá (que) tuviera un millón de
I wish I had a million dollars.
dólares.
Similar conjectures introduced with "It would be..." may use the past subjunctive:
It would be good for you to arrive on time.
Sería bueno que llegaras a tiempo.
It would be desirable if they could avoid
Sería deseable que evitaran la
inflation.
inflación.
PRÁCTICA 48A. Termine las oraciones de manera lógica. No todas requieren el
subjuntivo.
La gente quería un gobierno que...
6. A mis amiguitos les sorprendía que...
Sugirieron que...
7. Ojalá...
Los senadores dijeron que...
8. Sería deseable que...
A mis padres les gustaba que...
9. Nos parecía raro que...
Buscaba desesperadamente personas que...
10. Las noticias informaron que...
1.
2.
3.
4.
5.
PRÁCTICA 48B. Exprese en español:
1. It's not that he didn't have any options.
4. It's not that fighting solved anything.
2. It would be convenient for us to live here.
5. I wish we could live without eating.
3. It would be wonderful if you could go away.
6. I wish everybody understood me.
106
49. Pero/sino/no solo...sino
• When but follows a negation and means something like "but on the contrary" or "but
rather," use sino rather than pero:
No use "pero" sino "sino".
No es tonto, sino demasiado inteligente.
Don't use pero but sino.
He is not stupid, but too intelligent.
• If "but on the contrary" separates two conjugated verbs, sino que must be used:
I am not shouting but (rather) speaking
No grito, sino que hablo claramente.
clearly.
• When the second phrase does not exclude the first, pero is used:
He is not intelligent, but he does know a
No es inteligente, pero sabe mucho.
lot.
• Note that a comma is generally used before (not after) pero or sino.
• The Spanish equivalent of "not only...but (also)" is no solo...sino (también), and que
must be used when sino introduces a clause with a conjugated verb. Observe some
optional variations of this construction in the following examples:
El sistema no solo es eficaz, sino también agradable.
The system is not only efficient, but also nice.
No solamente oyeron música, sino que además bailaron.
Not only did they listen to music, but they also danced.
No querían únicamente la independencia política, sino también la económica.
They wanted, not just political independence, but also economic independence.
Manejo automóvil no sólo por comodidad, sino por necesidad.
I drive a car not only for comfort, but also out of necessity.
• También and además can be used interchangeably for this construction, and both of
them can be omitted. The adverbs solo, solamente and únicamente are also synonyms
in this context.
• The adverb solo –only– takes on a written accent when the context does not provide
enough clues to avoid confusion with the adjective solo –alone–, as in the fourth
example:
Manejo solo / sólo por necesidad.
I drive alone / only out of necessity.
PRÁCTICA 49. Complete con pero, sino, o sino que.
1. No te dije que trajeras las frutas, ___________ las llevaras a la casa de María.
2. Trabajar puede ser bueno para la salud, ___________ prefiero dormir.
3. No les importa que sigas estudiando, ___________ escuchando música tan ruidosa.
4. Cuba era importante para España no solo política ___________ económicamente.
5. No les gusta ir al gimnasio, ___________ tienen que hacerlo por su salud.
6. No creíamos que pudiera llegar a tiempo, ___________ esperábamos pacientemente.
7. No es que tenga muchos problemas, ___________ no resuelvo los pocos que tengo.
8. No es que saliera todos los días, ___________ sí lo hacía con frecuencia.
9. Tampoco es que fuera bohemio, ___________ le gustaba escribir toda la noche.
10. No dije que hubiera más, ___________ menos de diez millones de habitantes en Uruguay.
107
Repaso número once.
A. Aclaraciones. Describa las situaciones como en cada modelo.
Modelo 1: señor Gómez / gustar / autos / costar mucho / ser confiable
Al señor Gómez no le gustaba que los autos costaran mucho, sino que fueran
confiables.
1. el jefe / importar / empleados / tener mucha experiencia / ser listo
2. yo / agradar / clases / ser difícil / activo e interesante
3. la gente / preocupar / políticos / decir estupideces / destruir el país
4. nosotros / interesar / personas / saber mucho / ser comprensivo y estimulante
5. esos individuos / parecer bien / trabajos / exigir mucho tiempo / producir dinero
6. ministro de economía / doler / gente / estar en problemas / criticarlo
Modelo 2: conocer / nadie / ser millonario / alguien / saber vivir bien
No conocía a nadie que fuera millonario, pero sí a alguien que sabía vivir bien.
7. hablar con gente / ver mucha televisión / personas / leer bastante
8. conocer / nadie / saber otros idiomas / alguien / comunicarse bien
9. soñar con escenas / tener mucha acción / situaciones / relacionarse con su vida diaria.
10. asistir / conferencias / ser muy largo / presentaciones / ofrecer información útil.
B. Exprese en español:
1. Is there anyone here who knows her better? I don‘t know.
2. No, there is nobody who lives with her or near her house, which is sad.
3. There was no book in his library that was not boring. What a pity!
4. They told me to leave, but I refused. I didn't care what they might say.
5. It is true that it was raining that afternoon when we met.
6. It was not cold but hot. Not only was it hot, but also humid.
7. Evita wanted Argentina to love her, whatever she did.
108
8. What did you think of the movie I recommended? It was silly, but funny.
9. My boss demanded that we washed our hands before returning to work.
10. It would be silly for us to get angry with him before knowing what's happening.
Práctica escrita
A. Escriba un párrafo sobre lo que, en su opinión, es necesario que ocurra en el mundo.
¿Qué tipo de personas y/o acciones se necesitan? ¿Qué sería deseable que hicieran los
políticos, los científicos, la gente común, las compañías multinacionales, los educadores,
los niños? ¿Cómo sería posible que hubiera paz, que se lograra un mejor futuro? Incluya
en su párrafo:
- por lo menos una frase con lo que
- por lo menos una frase con por y otra con para
- por lo menos una frase con sino, otra con pero, y otra con no solo...sino
B. Primero, lea la versión en inglés. Después, complete el párrafo con las palabras y
letras necesarias. Finalmente, escriba dos preguntas sobre el texto y respóndalas.
La novelista, poeta, cuentista, periodista y guionista de películas bonaerense Ana María Shua
nac___ en 1951. ____ su prim___ libro, El _ol y _o, publica___ cuando t______ diecis_is años,
se ___ otorgaron dos premios de poesía. Estud___ en la Universidad de Buenos Aires, donde ___
especializó en Artes y Literatura. En 1976, ___ dictadura militar oblig__ a que su familia se
divid______ ____ el exilio. Ana María decidió __________ ____ algún tiempo en Francia con su
esposo. ____ regres____ a Argentina, su prim____ novela, Soy paciente, recib___ el prestigios_
Premio de narrativa Editorial Losada. Con más ___ quince libros, que inclu____ no solo todos los
géneros tradicionales ______ tambi_n l_s cuent_s brevísimos y la literatura infantil, est__ autora
continúa __________ extraordinari___ obras de imaginación, humor __ ironía. No es
sorprendente que sus libros _________ reconoci___ y premia___ en Argentina, Estados Unidos,
Venezuela y Alemania. En 1992 public__ un nuevo libro de historias brevísim___, Casa de
Geishas, en _________ páginas puede leer___ el siguiente microcuento:
Teóloga
En el siglo ____ despu_s de Cristo, un grupo de te_logos discute ________ el sexo de los
_ngeles. Obvi_mente, no se admite que ____ mujeres (entonces ni siquiera era seguro que
t_________ alma) ______ capaces de discutir materias teologales. Sin embargo, uno de ellos es
una mujer hábilmente disfraz____. Afirma con mucha energía que los ángeles sólo pued___
pertenec___ al sexo masculino. _________, ______ no ___ dice, que entre ellos ha__ mujeres
disfrazadas.
Casa de Geishas, de Ana María Shua. © 1992 Editorial Sudamericana
Buenos Aires' novelist, poet, short-story writer, journalist, and film writer Ana María
Shua was born in 1951. For her first book, The Sun and I, published when she was
sixteen, she was awarded two poetry prizes. She studied in the University of Buenos
Aires, where she majored in Arts and Literature. In 1976, the military dictatorship forced
her family to divide because of exile. Ana decided to stay for some time in France with
her husband. Upon returning to Argentina, her first novel, I'm Patient, received the
109
prestigious Narrative Prize from Losada Publishers. With more than fifteen books,
including not only all traditional genres but also short-short stories and children
literature, this author continues to produce extraordinary works of imagination, humor
and irony. It is not surprising that her books were recognized and awarded prizes in
Argentina, the United States, Venezuela and Germany. In 1992 she published a new book
of short-short stories, House of Geishas, in whose pages the following microstory can be
read:
A Woman Theologian
In the 7th century A.D., a group of theologians argue on the sex of angels. Obviously, it is
not admissible that women (then it was not even certain that they had a soul) be capable
of arguing on theological matters. However, one of them is a skillfully disguised woman.
She states energetically that angels can only belong in the masculine sex. She knows, but
does not say it, that there are disguised women among them.
50 Subjunctive in Adverb Clauses
El subjuntivo en cláusulas adverbiales
Adverb clauses modify the verb in the main clauses. They say something about how,
when, where or why that action occurs, and are always introduced by a conjunction:
(indicative)
main clause
conjunction
(indicative or subjunctive)
adverbial clause
Siempre me siento feliz
I always feel happy
cuando
when
me llamas.
you call me.
Hoy preparé la salsa
Today I made the sauce
como
the way
a ti te gusta.
you like it.
The subjunctive is used in adverb clauses when they convey provision: the action
described in the adverb clause is only anticipated and therefore it is or was hypothetical,
it implies a reservation, a condition not yet met or a mere intention.
A. CONJUNCTIONS OF PROVISION OR PURPOSE
An adverb clause always requires the subjunctive after the following conjunctions
because they presuppose a condition not met yet:
a menos que
con tal de que
en caso de que
unless
provided that
in case
antes de que
para que
sin que
110
before
so that
without
No puedes ganar a menos que te apoyen.
You can't win unless they support you.
We're going provided that the weather is
Vamos con tal que haga buen tiempo.
good.
Trajimos sombrero en caso de que haga sol. We brought a hat in case it is sunny.
Se fue antes de que yo pudiera hablar con
She went away before I could talk to her.
ella.
No puedes irte sin que hablemos primero. You cannot leave without us talking first.
Trabaja para que su hijo pueda viajar.
She works so that her son can travel.
When the provision does not involve a second subject, use the infinitive (que is not
used):
He works in order to be able to travel.
Trabaja para poder viajar.
She took a shower before leaving.
Se bañó antes de irse.
• a menos que always precedes a verb in the subjunctive, even if there is no change of
subject:
No puedo dormir a menos que me
I can't sleep unless I relax.
relaje.
• para que has a variety of equivalent expressions that convey the idea of ―so that‖, ―in
order that‖:
a fin de que, con el objeto/propósito de que, con la intención de que.
• sin que and a clause must be used in Spanish where English often uses a phrase such as
"without us seeing him." Spanish must say: "sin que lo veamos / viéramos."
•
PRÁCTICA 50A.
1. Complete con una conjunción o preposición lógica. Después, observe por qué se usa
el subjuntivo o el infinitivo en cada oración.
a. Lleva un paraguas ______________
d. Un cuchillo sirve __________ cortar
llueva.
cosas.
b. Debes lavarte las manos __________
e. Vamos a la playa ______________
comer.
haga sol.
c. No puede llover ______________ haya f. No puedo oírte ______________ hables
nubes.
fuerte.
2. Complete con el presente del subjuntivo o con el infinitivo, según el contexto:
a.(organizar) Nos vamos a reunir para _________________ una fiesta sorpresa para
María.
b.(dar) Vamos a hacerla en casa de Pedro, con tal de que sus padres nos ________
permiso.
c. (saber) Queremos hacer la fiesta sin que ella lo _________________.
d. (llegar) Vamos a preparar todo antes de que _________________ los invitados.
e. (ir) Pero no vamos a tener nada a menos que _________________ a comprar todo
pronto.
f. (necesitar) Tenemos que llevar la tarjeta de crédito en caso de
_________________la.
3. Exprese en español: She used to work so that her husband could study. But, without
her knowing it, he always went fishing (pescar) before going to class, unless it was
111
snowing.
B. CONJUNCTIONS OF TIME
en cuanto = tan pronto como
mientras (que)
cuando
después (de) que
hasta que
siempre que
as soon as
while, as long as
when
after
until
whenever, as long as
• The subjunctive follows time conjunctions to convey a future or anticipated event
(which has not or had not yet occurred, hypothetical at that moment):
Voy a regresar en cuanto caiga la
I am going back as soon as the night falls.
noche.
Cuando llegues, vas a sentirte mejor. When you arrive, you'll feel better.
No quería esperar hasta que
He didn't want to wait until you were done.
terminaras.
Cierre la puerta después de que salga. Close the door after you leave.
Mientras estés en Bolivia el año que viene, tienes que ir a Sucre.
While you are in Bolivia next year, you have to go to Sucre.
Dijo que iba a llamarnos siempre que se sintiera solo.
He said he would call us whenever (as long as) he felt lonely.
Note that there is a command (direct or indirect) or a future state involved.
• The indicative follows time conjunctions to describe an action as already completed, in
progress, or a known fact or habit:
Regresé tan pronto como cayó la
I came back as soon as the night fell.
noche.
Esperó hasta que terminaste.
He waited until you were done.
Siempre cierro la puerta cuando salgo. I always close the door when I leave.
Está viendo la película mientras
He is watching the movie while we eat.
comemos.
He (in fact) calls us whenever he feels
Nos llama siempre que se siente solo.
lonely.
PRÁCTICA 50B.
1. Complete con una conjunción de tiempo adecuada según el contexto. Observe también
por qué se usa el indicativo o el subjuntivo en cada oración:
a. Para poder votar, debes esperar _________________ tengas dieciocho años.
b. Siempre se enfermaba _________________ empezaba el invierno.
c. Trato de aprender esto bien _________________ estoy en la universidad,
porque, _________________ salga al mundo laboral, voy a necesitarlo.
d. Pensé que ibas a sentirte feliz _________________ nos besáramos.
112
2. Complete con el subjuntivo o con el indicativo según el contexto:
(terminar) Jorge y Gloria pensaban ir al cine cuando _________________ de
a.
estudiar.
(estudiar) Pedro, el novio de ella, los espiaba por la ventana mientras
b.
_________________.
c. (salir) Quería sorprenderlos tan pronto como _________________ de la casa.
d. (ver) De hecho, se sentía celoso siempre que los _________________ juntos.
e. (llegar) Al salir, Pedro quiso seguirlos hasta que _________________ al teatro.
f. (entrar) Pero los perdió después de que ellos _________________ en la autopista.
3. Exprese en español: You should visit the museum as soon as you can. While you are
there, don't forget to see the Egyptian exhibition. They brought it when I worked there,
and I liked seeing it whenever I had a minute. I used to see it very frequently, until
they fired (despedir) me.
C. AUNQUE
although, even though, even if
• The indicative follows aunque to acknowledge a fact:
Vamos a pescar aunque está
We're going fishing even though it's raining.
lloviendo.
No te creyó aunque decías la
He didn't believe you although you were telling
verdad.
the truth.
• The subjunctive follows aunque to present the circumstances as hypothetical or
irrelevant:
Vamos a pescar aunque llueva
We're going fishing even if it may rain
mañana.
tomorrow.
No iba a creerte aunque dijeras la
He wouldn't believe you even if you told the
verdad.
truth.
• a pesar de que (in spite of, despite that) is followed by the indicative or the
subjunctive under the same circumstances as aunque:
Vamos a pescar a pesar de que está lloviendo hoy y a pesar de que llueva mañana.
PRÁCTICA 50C.
1. Complete con el subjuntivo o con el indicativo según el contexto:
(estar) Aunque Brasil __________ en Suramérica, allí no se habla español, sino
a.
portugués.
(hacer) Queríamos ir aunque no __________ buen tiempo al día siguiente. No
b.
importaba.
c. (llegar) Por favor llámame, aunque (tú) __________ tarde el lunes próximo.
(ser) Aunque Stalin __________ ruso, su política no fue beneficiosa para muchos
d.
rusos.
e. (sentirse) Voy a ver el programa esta noche, aunque no __________ bien después.
113
D. COMO, DONDE, SEGÚN
• The indicative follows como (the way, as), según (according to) and donde (where) to
acknowledge a known place or way to do something:
Hazlo como te indiqué.
Do it the (specific) way I showed you.
Lo hice según indican las
I did it according to what the directions
instrucciones.
indicate.
Vimos el campo donde luchó
We saw the field where Bolívar fought.
Bolívar.
• The subjunctive follows como (in any way, as), según (as, according to) and donde
(wherever) to convey an unknown or nonspecific place or way to do something:
Hazlo como quieras.
Do it any way you want (as you please).
Puedes hacerlo según prefieras. You can do it as you prefer.
Podía jugar donde quisiera.
I could play wherever I wanted.
• Note that the conjunctions como and donde, unlike question words, do not have
accents.
• de la manera que, del modo que, are common equivalents of como, especially in
writing:
Hazlo de la manera que te indiqué, o del modo que quieras.
PRÁCTICA 50D. Dé respuestas abiertas con la conjunción y el verbo entre paréntesis:
Ejemplo: ¿Salimos ahora o más tarde? (cuando / tú / decidir) Cuando tú
decidas.
a. ¿Nos reunimos hoy o mañana? (cuando / el jefe / decirnos)
b. ¿Nos vemos en mi casa o en la tuya? (donde / tú / preferir)
c. ¿Te llamo por el celular o por el teléfono fijo? (según / te sea conveniente)
d. ¿Esperamos a Juan o vamos sin él? (como / ustedes / querer)
e. ¿Les mando el mensaje a la casa o a la oficina? (donde / ellos / recibirlo más
fácilmente)
f. ¿Cómo hago el postre? (de la manera / parecerte mejor)
114
Summary / Resumen
Conjunction
a menos que
con tal que
en caso de que
antes de que
para que
sin que
tan pronto como/en cuanto
mientras (que)
siempre que
cuando
después (de) que
hasta que
aunque
a pesar de que
como, según
donde
Followed by the subjunctive:
always followed by the
subjunctive
unless
provided that
in case
before
so that, in order that
without
as soon as
while, as long as
whenever, as long as
when
after
until
although, even if
in spite of, despite that
as, in any way
where, wherever
always followed by the
subjunctive
(but use infinitive when there is no
change of subject)
followed by the subjunctive when
referring to anticipated
circumstances
(hypothetical or unknown at the
moment)
Repaso número doce
A. Vida de pareja. Complete las oraciones con la forma verbal apropiada para el
infinitivo que se da entre paréntesis, añadiendo que cuando sea necesario:
MODELO: Hay varias reglas importantes para una relación (funcionar).
Hay varias reglas importantes para que una relación funcione.
1. Lávate los dientes antes de (llegar) tu pareja.
2. No discutas con tu pareja a menos que a ambos (interesarles) discutir.
3. No hables del clima para (cambiar) de tema.
4. Sal a dar un paseo siempre (estar) de mal humor.
5. No compres nada sin (aprobarlo) tu pareja.
6. Haz siempre la comida según (gustarle) a tu pareja.
115
7. Ten buenos amigos para quedarte en su casa en caso de (enojarse) tu pareja.
B. Cambie al pasado.
MODELO: Van a ir al cine después de que terminen la tarea.
Fueron al cine después de que terminaron la tarea. (They did go)
Iban a ir al cine después de que terminaran la tarea. (They were planning to go)
1. Se van a alegrar cuando les des la buena noticia.
2. Voy a comprar los ingredientes para que puedas cocinar bien.
3. Vamos a poner las cosas donde el dueño de la casa quiera.
4. ¿Llevas la tarjeta de crédito en caso de que la necesitemos?
5. Nunca hace nada sin que su familia lo apruebe.
6. No compro nada sin verlo primero.
7. Vas a ir a clase aunque no te sientas bien.
8. A mi familia le gusta hacer planes para aprovechar bien el tiempo.
C. Exprese en español:
1. He is studying in this country so that he can look for a better job.
2. I am going to buy you the house provided that the bank approves the loan.
3. Without him knowing it, we want to plan a party for his birthday.
4. Before he gets here, let's start singing. After he goes away, we can have fun.
5. As soon as you get there, call me. I really want you to call me.
6. Even though he was lying to her, and she knew it, the queen still liked her son.
7. Even if he continues to travel all night, he will not arrive on time.
8. I don't mind staying here until it stops raining.
9. One has the right to do whatever is needed in order to be happy.
10. I don‘t care how you do it, but do it any way you can.
116
11. They don't like the way you speak, and they don't want to speak that way, either.
Práctica escrita
Primero, lea la versión en inglés. Después, complete el párrafo con las palabras y letras
necesarias.
En caso de que Ud. qu____ tener un encuentro cercan_ con ___ naturaleza tropical, hay
una naci_n _entroamerican_ ____ tiene mucho que ofrecer. Para que l_s distint_s
ecos_stemas no _______ destru_____, el gobierno de Costa Rica cre__ más ___ treinta
par__es naturales, l_s cuales cubr__ el dieciséis ____ ciento del área nacional. ___
diversidad _ostarric_____ es asombros_ dondequiera que se _______. La __________
___ los visitantes ___ sorprende de que _______, por ejemplo, ______ 850 especies de
aves en e__ país, más ___ las 800 que _____ en los Estados Unidos, a pesar de t_____ un
área un poco más pequeñ_ ____ West Virginia: cin_uenta y un mil millas cuadra____.
Así que, en cuanto tom__ la decisi_n, Costa Rica le ofrecerá la o_ortunidad de
_________ tucanes y quetzales en ___ hábitat natural. Una vez en ___ pa_s, no querrá
_______ hasta que enc_______ un guía local, para que él le m_______ un cocodrilo en
algun_ ribera o un delfín arqueándose en ___ mar. El ecoturismo, sin embargo, está
estrict_mente control____ ____ el Servicio de Parques Nacionales, y nadie p______
internarse en las zonas proteg_____ sin que l_s oficinas de San José escrib___ el permiso
correspond______. Despu_s de conoc____ est_s datos, no __ parecerá extraño que en la
_ltima década el n_mero de visitantes haya aumentado de modo dramátic_, hasta casi un
millón ___ turistas el año pas____. Aunque Ud. se qued____ vari_s meses en ___ región,
no terminaría de apreci____ las innumerables especies de mamíferos, plantas y reptiles
que hay allí.
In case you want to have a close encounter with tropical nature, there is a Central
American nation that has a lot to offer. So that the different ecosystems were not
destroyed, the Costa Rican government created more than thirty Natural Parks, which
cover 16% of the national area. Costa Rican diversity is amazing wherever one goes.
Most visitors are surprised that there are, for example, some 850 bird species in that
country, more than the 800 there are in the United States, in spite of having an area
slightly smaller than West Virginia: 51,000 square miles. So, as soon as you make your
decision, Costa Rica will offer you the opportunity to get to know toucans and quetzals in
their natural habitat. Once in the country, you won't want to leave until you find a local
guide, so that he (can) show you a crocodile in some riverbank or a dolphin arching in
the sea. Ecotourism, however, is strictly controlled by the National Park Service, and no
one can get inside the protected areas without the San José offices writing the
corresponding permit. After learning these data, you won't find strange that, in the last
decade, the number of visitors has increased in a dramatic way, to almost a million
tourists last year. Even if you spent several months in the region, you wouldn't finish
appreciating the innumerable species of mammals, plants and reptiles there.
117
51. The Future and Conditional Tenses
Los tiempos futuro y condicional
Future (will)
hablaré
viviré
hablarás
vivirás
hablará
vivirá
hablaremos viviremos
hablaréis
viviréis
hablarán
vivirán
Conditional (would)
iría
sería
irías
serías
iría
sería
iríamos
seríamos
iríais
seríais
irían
serían
• The future and conditional have the same endings for all verbs.
• The stem is generally the infinitive, but some verbs (and their compounds) differ
slightly:
decir: diré, dirías
poder: podremos, podría
salir: saldrá, saldríais
caber: cabrá, cabrían
poner: pondrá, pondría
tener: tendré, tendría
haber: habrá, habría
querer: querrás, querríais
valer: valdrá, valdrían
hacer: harás, haría
saber: sabrás, sabría
venir: vendrá, vendríamos
THE FUTURE is used as it is in English:
Esta mesa no cabrá en el auto.
Haremos la comida cuando llegues.
Habrá mejores oportunidades algún
día.
This table won't fit in the car.
We'll prepare dinner when you get here.
There will be better opportunities some
day.
• Spanish also uses the future to express some conjectures (conjeturas):
He won't answer the phone--perhaps he‘s
No contesta el teléfono: estará fuera.
out.
Maybe something's happening to him/her.
¿Le pasará algo?
• Sometimes, English will or won't does not refer to the future but rather implies want, in
which case it must be expressed with the appropriate verb (usually querer) in Spanish:
He says he won't go with me.
Dice que no quiere ir conmigo.
• Remember that the present subjunctive is used to express future, hypothetical events in
subordinate clauses (as seen in sections 44-50):
No creemos que tenga ningún
We don't think he'll have any problems.
problema.
Ojalá que no haya examen mañana. I hope there won't be an exam tomorrow.
PRÁCTICA 51A. ¿Cómo será la sociedad del futuro?
¿Qué hará la gente?
118
¿Qué tipo de tecnología habrá?
PRÁCTICA 51B. ¿Futuro o subjuntivo presente? Complete con la forma apropiada.
1. (tener) No creemos que ella ___________ ningún problema. Yo creo que sí los ___________.
2. (encontrar) Ojalá que ellos ___________ la felicidad. Oh, sé que la ___________ algún día.
3. (decir) Quiero que tú le ___________ qué debe hacer. Sí, prometo que se lo ___________.
4. (querer) Tendré jugo en caso de que ellos no ___________ licor. Algunos no ___________.
5. (ponerse) Te compraré un helado para que no ___________ triste. ¡Oh, yo ___________ feliz!
6. (informar) Es normal que se nos ___________ sobre la guerra, y seguramente se nos ...
7. (haber) No ____________ clase mañana a menos que ___________ suficientes estudiantes.
THE CONDITIONAL is used as it is in English:
¡No diría eso nunca!
No me molestaría darte un beso.
Sería maravilloso saber bailar.
I/he/she/you would never say that!
I wouldn't mind giving you a kiss.
It would be wonderful to know how to
dance.
• Spanish also uses the conditional to express conjectures (conjeturas) about past
situations:
She didn't come. She was probably sick.
No vino ayer: estaría enferma.
• English frequently uses the word would to express past habitual actions. Spanish does
not use the conditional, but rather the imperfect in such cases:
As a child, I would go to the movies all the
De niño, iba al cine todo el tiempo.
time.
FOR POLITE REQUESTS:
• Quisiera is often used for I would like instead of the conditional (Querría is generally
avoided).
Would you like (want) to go with me?
¿Quisieras acompañarme?
Quisiera ir al Perú algún día.
I would like to go to Peru some day.
Me gustaría ir al Perú.
• Would you...?, Could you...? involve either the conditional or the past subjunctive of
poder:
¿Me pudiera Ud. dar información?
(formal)
Could/Would you give me some
¿Me podrías dar información? (just information?
polite)
PRÁCTICA 51C. Entrevista al profesor del año. Forme preguntas como en el modelo.
cómo / mejorar el sistema educativo: ¿Cómo mejoraría Ud. el sistema educativo?
1. cómo / apoyar el arte y la creatividad.
2. qué / hacer frente al analfabetismo
119
3 . de qué manera / prevenir el crimen
4. qué / decirles a los jóvenes del país
5. de qué modo / comunicarse con la nación
6. qué / tener como prioridad en el gobierno
PRÁCTICA 51D. Exprese en español:
1. Could you tell me what you'll do?
2. I would love it.
3. We'd have to do this faster.
4. Many people would not take a bath (bañarse) very frequently in the 19th century.
5. I'd like to go fishing.
6. Could you help me, sir?
7. He was probably listening to music.
52. If-Clauses / Frases con "si"
The present subjunctive is never used in si-clauses. Very much like English, either some
Indicative tense or the Past Subjunctive will be used according to how likely the event is
perceived.
A. USE THE INDICATIVE:
• For events likely to take place:
Si suben el precio, no lo compraremos. If the price is raised, we won't buy it.
No tienes que hacerlo si no hay tiempo. You don't have to do it if there's no time.
I don't know if (whether) he is home or
No sé si él está en casa o no.
not.
• After por si (acaso)... just in case:
Traigo paraguas por si acaso (llueve).
Llevó el celular por si se perdía.
I'm bringing an umbrella just in case (it
rains).
He took his cell phone in case he got lost.
B. USE THE PAST SUBJUNCTIVE:
• For unlikely or contrary-to-fact events, the si-clause is in the past subjunctive, and the
result clause in the conditional:
120
Si tuviera un millón de dólares, sería
rico.
No haría eso si fuera tú.
If he had a million dollars, he'd be rich.
I wouldn't do that if I were you.
• After como si... as if:
Me trataba como si yo fuera una niña. He treated me as if I were a child.
She always acts as if nothing were the
Siempre actúa como si no pasara nada.
matter.
• After ojalá and aunque, when they refer to conditions contrary to fact:
If only there were peace on this planet.
Ojalá hubiera paz en este planeta.
I'd go to the beach even if it snowed in
Iría a la playa aunque nevara en
summer.
verano.
• When whether...or (not) conveys hypotheses, Spanish uses subjunctive constructions
(not si-clauses):
We'll get married whether they like it or
Nos casaremos (sea que) les guste o no.
not.
PRÁCTICA 52A. Sustituya las siguientes condiciones hipotéticas por posibilidades
reales.
Modelo: Si vinieras, me pondría contento.
Si vienes, me pondré contento.
-->
1. Si se me olvidara el paraguas, me mojaría.
4. Si le gustara la casa, se la vendería.
2. Si se quitara el sombrero, podríamos verle la cara. 5. No ocurriría nada si ella no se diera cuenta.
3. Si trajeras más muebles, no cabríamos en la casa.
6. Me haría empleado público si el gobierno
pagara mejor.
PRÁCTICA 52B. Termine las oraciones de manera lógica.
1. Me pondría contento si...
5. Mis padres se enojarán si...
2. Si el gobierno sube los impuestos...
6. Ojalá todo...
3. Obtendré un buen trabajo si...
7. Viviría aquí aunque (even if)...
4. Si todo fuera perfecto...
8. Se puso ropa de invierno por si...
5. Se nos olvidarían los problemas si...
9. Algunos gobiernos nos tratan como si...
121
Repaso número trece
A.
Comente los problemas con tres posibilidades hipotéticas como en el modelo.
Modelo: Se me acaba la energía. Sería mejor si no se te acabara.
Vivirás mejor si no se te acaba.
Todo será más fácil cuando no se te acabe.
1. Se me olvidan las fechas.
3. Se me olvida respirar.
2. Se me pierden las cosas.
4. Se me acaban las ganas.
B. Termine las oraciones de manera gramaticalmente correcta.
1. Me pondré feliz cuando...
5. El planeta tendrá mejor futuro después de que...
2. Me volvería loco si...
6. Mis padres se pondrán contentos cuando...
3. La gente se hace deportista si...
7. No habrá igualdad social a menos que...
4. Me gustaría conocer personas que... 8. Me graduaré con tal de que...
C. Exprese en español:
1. I wish it were true; I'd like it a lot. But I don't think it will be possible.
2. He will be given the necessary instructions, even if he doesn't need them.
3. I wish we didn't have a final exam for this class; it would be great.
4. Although it's raining today, I still want to go. I'll take an umbrella just in case.
5. Could you lend me $10 just in case I run out of money?
6. If they are having fun, why ask them to do something else?
122
7. There are people who act as if they were going to die tomorrow.
8. Actually, I would help her if she realized she needs help.
9. If the government invested more in education, there would be less crime.
10. I'd like us to communicate (be in touch) more often.
11. There will be fewer problems if we help each other.
Práctica escrita
Primero, lea la versión en inglés. Después, complete el párrafo con las palabras y letras
necesarias.
La Declaración de Independencia firm___ ___ cuatro de julio ___ 1776, estipuló ____ consenso unánime
que las trece colonias se llamar___ de ahí en adelante los Estados Unidos de _m_rica, que ten_____ un
gobierno federal independiente ___ Inglaterra, y que __ proteg______ los derechos inalienabl___ de cada
individu__. La nuev_ naci_n decid___ adoptar ___ sistema democr_tic__ ______ que es_s derechos --a
___ Vida, ___ Libertad y la búsqueda de ___ Felicidad, entre otros-- ________ respetad___ _____ todos
los estados. Tambi_n estableció que, si cualquier forma de gobierno ___ h________ destructiv_ para est_s
fines, el pueblo ten_______ derecho a alterarla o abol______. La prudencia de hecho dicta____ que los
gobiernos bien establecidos no ___ cambia____ ____ causas ligeras y transitori___. ___ muy posible que el
Rey Jorge III no se imagin____ que est_ pequeñ_ nación ____ extend_____
__________________________ Norteamérica y que, dos siglos despu_s, llegaría a ser un_ potencia
mundial. Est_ Declaración ____ hoy un símbolo de l_s ideales democrátic_s, l_s cuales sin duda
_______________________ a much_s otr_s naciones para constituir sociedades just___. ____ lamentable
que los gobiernos posteriores de los Estados Unidos hayan negado l_s mism_s derechos a otros pueblos del
mundo, apoy_______ a veces dictaduras militar____ en diversos países, e invadi__________ ilegalmente.
________ deseable que algunos gobiernos rec__________ que, en el curso de los acontecimientos
humanos, tarde o temprano _____________ necesario que un pueblo asum___ entre los poderes terrenales
la condición de i______dad a la que tiene derecho.
The Declaration of Independence signed on July 4th, 1776, stipulated by unanimous
consensus that the thirteen colonies would be called from then on the United States of
America, that they would have a federal government independent from England, and that
the inalienable rights of each individual would be protected. The new nation decided to
adopt a democratic government so that those rights --to Life, Liberty and the pursuit of
Happiness, among others-- be respected by all of the states. It also established that, if any
form of government became destructive of these ends, the People would have the right to
alter or to abolish it. Prudence, indeed, would dictate that well-established governments
were not changed for light and transient causes. It is very possible that King George III
did not imagine that this little nation would extend itself through most of North America
and that, two centuries later, it would come to be a world power. This Declaration is
today a symbol for democratic ideals, which, without a doubt, will continue to inspire
many other nations in constituting just societies. It is regrettable that subsequent
governments of the United States have denied the same rights to other peoples in the
world, by supporting at times military dictatorships in various countries, and by invading
them illegally. It would be desirable if some governments remembered that in the course
123
of human events, sooner or later it becomes necessary for one people to assume among
the powers of the earth, the equal station to which it is entitled.
53. Compound Tenses with Haber
Los tiempos compuestos con haber
COMPOUND INDICATIVE TENSES USING haber:
Present Perfect
Pluperfect
Future Perfect
Conditional Perfect
(I have given, etc.)
(I had read, etc.)
(I will have
(I would have said...)
returned...)
he dado
había leído
habré vuelto
habría dicho
has dado
habías leído
habrás vuelto
habrías dicho
ha dado
había leído
habrá vuelto
habría dicho
hemos dado
habíamos leído
habremos vuelto
habríamos dicho
habéis dado
habíais leído
habréis vuelto
habríais dicho
han dado
habían leído
habrán vuelto
habrían dicho
Nunca hemos estado en París.
Ya me lo habían mencionado.
Lo habré terminado para el sábado.
Yo no me habría sentido así.
We have never been to Paris.
They had already mentioned it to me.
I will have finished it by Saturday.
I would not have felt that way.
• Spanish forms compound tenses with the auxiliary haber and the past participle in
much the same way that English does with have.
• Following haber, the past participle is invariable; it always ends in -o. (Following ser
or estar, the past participle agrees with the noun in gender and number as an
adjective).
• Reflexive and object pronouns must precede the conjugated form of haber.
• Hay is expressed in any of these tenses with the past participle of haber preceded by
the auxiliary (ha, había, habrá, habría) in the 3rd person singular:
There haven't been any problems in the
No ha habido problemas en el país.
country.
PRÁCTICA 53A. Exprese en español:
1. I don't know if she has seen the movie.
2. I didn't know if she had seen the movie.
3. She will have seen it by next week.
124
4. They asked me if she had been to Rome.
5. He told me he hadn't put the hat on.
6. He says he hasn't put it on.
7. I would have opened the door for you.
8. It would have been better that way.
COMPOUND SUBJUNCTIVE TENSES USING haber:
Perfect Subjunctive
...(that) I have helped, etc.
Pluperfect Subjunctive
...(that) I had attended, etc.
haya asistido
hubiera ayudado
hayas asistido
hubieras ayudado
haya asistido
hubiera ayudado
hayamos asistido
hubiéramos ayudado
hayáis asistido
hubierais ayudado
hayan asistido
hubieran ayudado
Lamento que la haya tratado tan mal.
Dudo que la hayan visto todavía.
Si se me hubiera dicho eso, no habría ido.
Nunca salía antes de que su esposa
se hubiera levantado.
I am sorry that he (has) treated her so badly.
I doubt that they have seen it yet.
Had I been told that, I wouldn't have gone.
He never went out before his wife
had gotten up.
• Compound subjunctive tenses are ruled by the general principles for the use of the
subjunctive.
• A common context for using the pluperfect subjunctive (hubiera) is in si-clauses
referring to the past. The perfect subjunctive (haya) is never used after si:
I would have called you if I had been able
Te habría llamado si hubiera podido.
to.
Si hubiera habido tiempo, nos
Had there been some time, we would have
habríamos divertido.
had fun.
PRÁCTICA 53B. Cambie al pasado las oraciones, como en el modelo.
MODELO: Nos encantaría que vinieran mañana. Nos habría encantado que hubieran
venido ayer.
1. No les molestaría que trajeras al perro esta noche.
2. ¿Te parecería raro que se me olvidara tu cumpleaños este mes?
125
3. Si estuvieran tristes, no querrían comer hoy.
4. Si se hiciera una encuesta, se sabría qué piensa el público sobre este tema.
5. Aunque no hubiera fantasmas, tendría miedo de estar solo mientras veo esta película.
6. No sé qué haría si se nos acabara el dinero en este momento.
7. A tus amigos les gustaría que te hicieras abogado.
54. Expressing Present Attitudes Toward Past Events
Cómo expresar una actitud presente frente a un evento pasado
•When an attitude is expressed in the present (I'm glad) toward an action completed in the
past (she left), the perfect subjunctive is normally used in the dependent clause:
Main clause: Present attitude
Dependent clause: Complete past event
conjunction
Present indicative
Perfect subjunctive
Me alegra
I'm glad
que
(that)
se haya ido.
she (has) left.
(Es una) lástima
It's a pity
que
that
no haya llovido el verano pasado.
it didn't rain last summer.
No pueden creer
They can't believe
que
that
no hayas continuado escribiendo.
you didn't continue to write.
Ojalá
I hope
que
hayas tenido un buen viaje.
you had a good trip.
•Only when a present attitude refers to things that used to happen or descriptions, the
[imperfect] past subjunctive may be used in the dependent clause:
No es sorprendente que Colón viajara [haya viajado] con frecuencia.
It's not surprising that Columbus used to travel [had traveled] frequently.
PRÁCTICA 54. Exprese sus reacciones presentes a los eventos descritos.
Ejemplos: Hubo una explosión en Bogotá :: Siento mucho que haya habido una
explosión allá.
Me gustó la exposición :: Me parece normal que te haya gustado (la
exposición).
1. No dormí nada anoche.
2. Me cansé de hacer deporte.
126
3. Hizo sol en la playa.
4. Ya almorcé.
5. Hubo un accidente.
6. Había revoluciones frecuentes en el siglo XIX.
7. No ocurrió nada interesante hoy.
8. Obtuve una A en mi examen de biología.
9. Nos gustaron las pinturas de Goya.
10. Se preocuparon porque no llamaste.
55. Reported Speech: Use of the Pluperfect
Cómo relatar lo dicho en el pasado: El uso del pluscuamperfecto
• When telling what someone said, the imperfect is generally used to report on stated
ongoing conditions, intentions, habits or actions in progress:
He told me that he had lived in
Me contó que hacía muchos años que vivía (ongoing
a small town for many years,
condition) en un pueblo pequeño, y que quería
and that he wanted to study in
(intention) estudiar en una gran ciudad. Dijo que
a large city. He said that for
por eso estudiaba (habit) mucho siempre, porque se
that reason he always studied a
estaba preparando (action in progress) para un
lot, because he was preparing
ambiente competitivo.
for a competitive environment.
Note that hace muchos años is also a description reported in the imperfect (hacía).
• The pluperfect is used to report on a past statement that describes a previous
occurrence:
Dijo: "Salí de mi pueblo hace varios años, y ahora vivo en Chicago".
Dijo que había salido de su pueblo hacía varios años, y que ahora vivía en Chicago.
He said he (had) left his town several years ago, and he now lived in Chicago.
• Again, surrounding circumstances are reported in the imperfect:
Dijo: "Me fui cuando tenía veintidós años"
Dijo que se había ido cuando tenía veintidós años.
He said he (had) finished his studies when he was twenty-two and then he came to
this city.
• Unlike colloquial English, in this context the pluperfect is widely used in Spanish.
Compare:
Dijeron que se habían divertido mucho. They said they had a great time.
Me contó que había ido a Perú hacía dos He told me he went (had been) to Perú
años.
two years earlier (ago).
127
PRÁCTICA 55. Relate qué dijeron las noticias de ayer.
Ejemplo: "El ministro dio un discurso". Dijeron que el ministro había dado un discurso.
1. Eligieron nuevo presidente.
2. Estaba lloviendo mucho mientras la gente votaba.
3. Los resultados se publicaron por la noche, pero ya todo el mundo sabía quién ganaría.
4. Hubo un accidente en el centro.
5. Seguía lloviendo cuando ocurrió el accidente.
6. Los Piratas ganaron el campeonato de fútbol.
7. Dejó de llover cuando los Piratas jugaban.
8. Terminó la cumbre (summit) internacional.
9. Ya no llovía cuando salieron los embajadores.
56. Todavía, aún, ya (no)
• The adverbs todavía and aún both mean still/yet, and can be placed before or after the
verb:
Le parecía imposible que estuviera lloviendo todavía (aún).
It seemed impossible to him/her that it was still raining.
Aún (todavía) no había comenzado a nevar cuando ocurrió el accidente.
It hadn‘t started snowing yet when the accident happened.
• Only before or after comparisons (más, menor, peor, etc.), can even be translated as
todavía or aún: Es aún mejor de lo que pensaba. It‘s even better than I thought.
• Keep in mind that English still and yet are also conjunctions meaning however or
but. In these cases other Spanish expressions must be used, such as sin embargo,
con todo, pero, mas:
She's smart, yet selfish (Es lista, pero egoísta).
• Ya has several uses, but its most common meanings are already and now for
affirmative sentences, and not anymore for negative sentences. It's usually placed
before the verb:
I‘ll be right back; I'm coming back right now.
Ya vuelvo / regreso.
They will (may) have forgotten by now.
Ya se les habrá olvidado.
They have returned the books already.
Ya han devuelto los libros.
Ya almorcé / Ya he almorzado. I(‗ve) already had lunch.
They don't like dancing anymore.
Ya no les gusta bailar.
128
Ya no nos quejaremos (más).
We won‘t complain anymore.
PRÁCTICA 56. Complete con todavía o ya, según el contexto.
1. ¿__________ leíste el libro? No, __________ no he comenzado a leerlo, pero seguramente para el
martes __________ lo habré terminado. ¿De qué se trata?
2. Es sobre la vida de los aztecas cuando los españoles __________ no habían llegado a América. Es más
interesante __________ que el texto de la semana pasada. __________ he leído más de la mitad. Pero es
bastante largo, y __________ no tenemos mucho tiempo.
3. Bueno, pero __________ nos quedan tres días antes de la clase. Como __________ estoy preparada
para las otras clases, creo que __________ puedo terminarlo a tiempo.
4. Pero si hubieras comenzado la semana pasada, __________ no tendrías que preocuparte.
Repaso número catorce
A. Exprese en inglés:
Se da cuenta de que tenemos que resolver el problema.
En ese momento se dio cuenta que teníamos que resolver el problema.
Por su desinterés, no se daba cuenta que teníamos que resolver el problema.
Todavía no se ha dado cuenta que tenemos que resolver el problema.
Aún no se había dado cuenta que teníamos que resolver el problema.
Si se lo dicen, se dará cuenta que tenemos que resolver el problema.
Si se lo dijeran, se daría cuenta que tenemos que resolver el problema.
Aunque se lo digan, no se dará cuenta que tenemos que resolverlo.
Aunque se lo dijeran, no se daría cuenta que lo tenemos que resolver.
Quieren que se dé cuenta que tenemos que resolverlo.
Les habría gustado que se diera cuenta que teníamos que resolverlo.
Si se lo hubieran dicho, se habría dado cuenta que teníamos que resolverlo.
No creo que se haya dado cuenta que lo tenemos que resolver.
B. Complete con el participio pasivo (-ado, -ido) del verbo entre paréntesis. Tenga en
cuenta que en algunas oraciones es necesario cambiar la terminación según el
género y el número del sustantivo.
1. (hacer) Todavía no he _________ las papas, pero estarán _________ para antes de las siete.
2. (nominar) Borges había sido _________ varias veces para el Premio Nóbel, pero no se le otorgó.
3. (apoyar) No creo que hubiera _________ esta causa si no supiera que fue _________ por otros.
129
4. (inventar) Galileo había _________ el termómetro más de cien años antes de que la escala de grados
centígrados fuera _________ por Celsius.
5. (escribir) La vida nueva fue _________ por Dante mucho antes de haber _________ la Divina
Comedia.
6. (devolver) Los libros no están _________todavía, pero ya los habremos _________ para el lunes.
C. Exprese en español:
1. Haven't you read the poem yet? I can't believe you haven't read it.
2. When we arrived, we realized they hadn't waited for us.
3. Has he returned her money? He will not have solved the problem by Thursday.
4. It is sad that they didn't have more time for their lecture.
5. I hope they are finished with the exam already.
6. He said he didn't know anyone who had been to his country.
7. I didn't want to take him to the airport before he had seen her.
8. They said you enjoyed yourself in Canada. If I were you, I'd go again.
9. I would have never finished it without her telling me how it was done.
10. He's always known there is a better system to achieve results.
Práctica escrita
Primero, lea la versión en inglés. Después, complete el párrafo con las palabras y letras
necesarias.
___ activista indígena quiché Rigoberta Menchú Tum _______ treinta y tres años cuando en 1992 ___
convirtió en la n________ mujer que recibió el Premio Nóbel de la Paz, cuatrocientos años despu_s de que
los _uropeos llega____ _l continente _merican_. A Rigoberta se ___ dio el premio _____ su defensa de l_s
derechos human_s para l_s indígenas _uatemaltec_s. Aunque esta no fue __ prime__ vez que a alguien de
Guatemala se ___ otorgó el Nóbel --Miguel Ángel Asturias _______ gan___ el de literatura en 1967--, el
acontecimiento t_____ gran impacto en la vida del país, que ______________ en una cruenta guerra civil
durante los años ochent___, habiendo sufr____ bajo vari_s dictad_r_s. ____ eso ___ significativo que
Menchú, en su discurso del Nóbel, ______ afirmado que el premio ____ un tributo, no sólo a ella y a los
pueblos indígen_s de _m_rica, _______ también al "pueblo _entroamericano, que ______________ su
estabilidad, la organización de su futuro, y el camino ______ su desarrollo __ integración, basados en la
democracia civil y el respe__ mutuo". Est_ campesin_ maya-quiché ___________ famos_ ____ la
publicación del libro Me llamo Rigoberta Menchú en 1983, que comien___ con el siguiente epígra_e del
Popol Vuh, el libro sagrado más importante ___ la literatura _aya: "Siempre ____mos viv____ aquí: es
justo que continu_mos ___________ donde nos place y donde qu________ morir. Sólo aquí pod___mos
resucitar; en otr_s partes jamás volv____mos a encontrarnos completos y nuestro dolor ______ etern__".
130
Es interesante que est_s palabras, escri_____ hace más ____ trescientos años, _________ ten____ tant_
relevancia ______ la situación _______ de l_s indígenas american_s.
Indigenous quiché activist Rigoberta Menchú was 33 when in 1992 she became the ninth
woman who received the Nobel Peace Prize, four hundred years after the Europeans
arrived in the American continent. Rigoberta was given the prize for her defense of
human rights for indigenous Guatemalans. Although this was not the first time in which
someone from Guatemala was awarded the Nobel --Miguel Ángel Asturias had won the
one for literature in 1967--, the event had a great impact in the life of the country, which
had been in a cruel civil war during the 1980s, having suffered under several
dictatorships. This is why it is significant that Menchú, in her Nobel address, stated that
the prize was a tribute, not only to her and to the indigenous peoples from Guatemala,
but also to the "Central-American people who are still searching for their stability, for
the structuring of their future, and the path for their development and integration, based
on civil democracy and mutual respect." This Mayan-quiché peasant woman became
famous for the publication of the book I, Rigoberta Menchú in 1983, which begins with
the following epigraph from the Popol Vuh, the most important sacred book in Mayan
literature: "We have always lives here--it is fair for us to continue to live where we
please and where we want to die. Only here will we be able to come back to life;
somewhere else we would never find ourselves whole again, and our pain would be
eternal." It is interesting that these words, written more than three hundred years ago,
still have so much relevance for the current situation of Native Americans.
131

Documentos relacionados